You are on page 1of 345
Ecuaciones diferenciales son igualdades que envuelven derivadas de funciones desconocidas. Por ejemplo: y tay =3 y!" + By! + 6y = cos(x) Pi di 1 Lag t Rat Gi = Bwcos(ut) (2c? + )dx — 2aydy = FV, PV oe OF (1) (12) Cuando una ecuacién envuelve una o més derivadas con respecto a una variable par- ticular, esa variable es Ilamada variable independiente. Una variable es llamada dependiente si en la ecuacién hay alguna derivada de esa variable. En las ecuaciones (1-1) y (1.2) la variable dependiente es y y hay una tinica variable independiente que es x. En la ecuacién (1.3) la variable dependiente es é y tes la variable independiente. L, R,C,£, y w son constantes Hamadas parametros. La ecuaci6n (1.5) tiene una variable dependiente V y dos variables independientes rey. Como la ecuacién (1.4) puede ser escrita 24 dy — 2xy—= =0 r de — —2ry =0, podemos considerar cualquiera de las variables como dependiente, pasando a ser la otra independiente. En las ecuaciones (1.1)-(1.4), tenemos solo una variable independiente y por lo tanto todas las derivadas que aparecen son ordinarias. Tales ecuaciones reciben el nombre de ecuaciones diferenciales ordinarias. Cuando la variable dependiente depende de mas de una variable independiente. como en la ecuacidn (1.5), la ecuacién se llama ecuacién diferencial parcial. Los primeros capftulos de este libro estaran dedicados al estudio de ecuaciones diferenciales ordinarias, abordéndose el tema de las ecuaciones diferenciales parciales en la tiltima parte. Se define el orden de una ecuacién diferencial como la derivada de mayor orden que aparece en la ecuacién. Por ejemplo, Py dy Sat uy ee es una ecuacién de orden dos. A estas ecuaciones también se las conace como eeuaciones de segundo orden, También son de orden dos las ecuaciones (1.2),(1.3) y (1.5), en tanto que las ecuaciones (1.1) y (1.4) son de primer orden. +y=0 (1.6) 1.2 Generalidades sobre ecuaciones diferenciales ordinarias En general, Ia ecuacién F(x, y.y'e0s y™) =0 (1.7) es Hamada ecuacién diferencial ordinaria de enésimo orden. Bajo restricciones con= venientes sobre la funcién F’ la ecuacidn (1.7) puede resolverse explicitamente para y™ en término de las otras n + 1 variables 2, y,y/.+++ .y(""!), obteniéndose y= flee ysy'er sy). (18) Para los propésitos de este libro asumiremos que esto es siempre posible. Sin em- bargo, hay que tener en cuenta que una ecuacién de la forma (1.7) puede representar més de una ecuacién de la forma (1.8). Por ejemplo, la ecuacién ly)? + 4y! — 62? =0 representa dos ecuaciones diferentes, »_ 24+ V4+ 623 , y= t Una funcién @ definida sobre un intervalo 7, es Hamada una solucién de la ecuacién diferencial (1.8) si para todo x € 7. es derivable hasta el enésimo orden y se tiene G(x) = Fe. G(@). O(a) Por ejemplo, verifiquemos que es una solucién de la ecuacién da? Sustituyendo nuestro candidato a solucién en el miembro izquierdo de la ecuacién (1.9) obtenemos fu dg, = Ae? +2" — 62" =0, lo que completa la verificacién. Un concepto importante es el de linealidad 0 no-linealidad de una ecuacién dife- rencial. La ecuacién Flay yl yy) = es Hamada lineal si la funcién F es una funcién lineal de las variables y, y’,+++ . Asi, por ejemplo, una ecuacidn lineal de orden n puede ser escrita de la forma =R(x). (1.10) Con este concepto Ia ecuacién (1.6) anterior es no-lineal, y la ecuacién (1.9) es lineal. La ecuacién wy!" + xy! + (a? = )\y= 408 es también lineal. La forma en que la variable independiente entra en la ecuacién no tiene nada que ver con la propiedad de linealidad. 1.3 Motivacién Las ecuaciones diferenciales aparecen frecuentemente en modelos matemiticos que tratan de describir situaciones de la vida real. Muchas leyes naturales y hipdtesis pueden ser traducidas vfa el Ienguaje matemético en ecuaciones que envuelven derivadas. Por ejemplo, derivadas aparecen en fisica como velocidades y acelera- ciones, en geometria como pendientes, en biologfa como razén de crecimiento de poblaciones, en sicologfa como razén de aprendizaje. en quimica como rapide. de reaccidn. en economfa como razén de cambio del costo de vida. y en finanzas como razén de crecimiento de inversiones. En diversos modelos matematicos, para obtener una ecuacién diferencial que describa un problema real. se asume que la situacién es gobernada por leyes muy simples. Una vez que el modelo es construfdo en la forma de una ecuacién diferencial, la siguiente etapa es resolver la ecuacién diferencial y utilizar la solucién para hacer predicciones relativas al comportamiento del problema real. En el caso que estas predicciones no concuerden razonablemente con la realidad, se debe reconsiderar los supuestos iniciales para obtener un modelo més cercano con Ia realidad. En muchos casos la modelacién de un fenémeno conduce a ecuaciones diferenciales que no se resuelven con Ia teorfa conocida y constituyen motivacién para muchos desarrollos matematicos, En la practica y, a fin de atacar el problema propuesto. se hacen algunas simplificaciones y modificaciones al modelo para resolverlo de manera exacta 0 aproximada con la teorfa conocida. Si la funcién y = y(x) representa una cantidad desconocida que queremos es- tudiar. del célculo sabemos que la primera derivada y/ = “ representa la razén de cambio de y por unidad que cambia x. Si por ejemplo se conoce esta razén de cambio (digamos por experiencia o por alguna ley fisica) y se sabe que es igual a una funcién f(z, y). entonces la cantidad y satisface la ecuacién diferencial ordinaria de primer orden y! = f(z.y). De idéntica manera pueden plantearse ecuaciones de or- den superior. A continuacién daremos algunos ejemplos especificos donde aparecen ecuaciones de primer y segundo orden, Algunos de ellos seran analizados con més detalle en capitulos posteriores, Biologfa. Se ha observado que para un gran tipo de colonias de bacterias estas tienden a crecer en una raz6n proporcional al mimero de bacterias presentes. Para tales colonias, sea N = N(t) el ntimero de bacterias presentes en cualquier instante 1. Entonces, si k es la constante de proporcionalidad, la funcién N = N(L) satisface Ja ecuacién ordinaria de primer orden N=KN (1.11) Esta ecuacién es Hamada ley de Malthus para el crecimiento de poblaciones, T. R. Malthus observd, en 1798. que la poblacién de Europa parecfa duplicarse en intervalos regulares de tiempo, y asf concluyé que la raz6n en que la poblacién crece es proporcional a la poblacién presente. Notemos que la funcién V(t) toma solo valores enteros y luego no es continua y menos diferenciable, Sin embargo. si el ntimero de bacterias es muy grande, podemos asumir que puede ser aproximada por una funcién diferenciable N(4). ya que los cambios en el tamaiio de la poblacién ocurren sobre pequefios intervalos de tiempo. Més adelante estableceremos que su solucién es N(¢) = N(O)e, donde N(0) es el ntimero de bacterias presentes inicialmente, es decir, cuando ¢ = 0. La solucién N(t) puede ser representada gréficamente como en la Figura 1. ‘N(t) = N(O)et Habrfa que enfatizar que la ecuacién (1.11) es un modelo matematico que des- ctibe una colonia de bacterias que crece de acuerdo a una ley muy simple. quizAs demasiado simple, obteniéndose de esta manera una ecuacién diferencial muy simple. Claramente, un modelo més realista se obtiene tomando en cuenta algunos otros factores como crecimiento exponencial. limitaciones de alimento, etc, Por supuesto, la ecuacién diferencial que se obtiene en estos casos es mucho més compleja. Farmacologfa. Se sabe en farmacologfa que la penicilina y otras drogas admi tradas a pacientes desaparecen de sus cuerpos de acuerdo a la siguiente regla: Si y(t) es la cantidad de droga en un cuerpo humano en el instante ¢. entonces la raz6n. de cambio #(¢) de la droga es proporcional a la cantidad presente. Esto es, y(é) satisface la ecuacidn diferencial ya—ky (1.12) donde k > 0 es la constante de proporcionalidad. El signo negativo en (1.12) es de- bido al hecho que y(£) decrece cuando ¢ rece. y luego la derivada de y(£) con respecto at es negativa, Para cada droga, la constante / es conocida experimentalmente. La solucién de la ecuacién (1.12) es y(Q=we™, (1.13) donde yp = (0) es la cantidad inicial de droga (dosis inicial). y Figura 2 Como se ve de la ecuacién (1.13) (ver también Figura 2), la cantidad de droga en el cuerpo del paciente tiende a cero cuando ¢ + 90. Sin embargo, en muchos casos es necesario mantener (aproximadamente) una concentracién constante (y por 10 tanto aproximadamente una cantidad constante) de droga en el cuerpo del paciente por largo tiempo, Para lograr esto es necesario dar al paciente una dosis inicial yo luego a intervalos iguales de tiempo. digamos 7 horas, dar al paciente una dosis D de la droga. La ecuacién (1.13) indica la cantidad de droga en el cuerpo del paciente en cada tiempo ¢; luego. es simple determinar la cantidad de dosis D. En efecto. al instante 7, y antes de administrar la dosis D, la cantidad de droga presente en el cuerpo es (7) = ye". Si queremos mantener la cantidad inicial yo en los tiempos 7,27, 37,+++. la dosis D, que debemos suministrar al paciente cada 7 horas. debe satisfacer Ia ecuacién ye” +D=y Luego D=w(l-e™). Mecénica. El principio de determinacién de Newton establece que el movimiento de una partfcula o de un sistema de ellas depende de las posiciones y velocidades del sistema. Aplicando esto a la aceleracién é de una particula. se tiene la ecuacién # = F(t.x.4), (1.14) que se reconoce como la Segunda Ley de la Mecdnica Newtoniana. En ella, F es la suma de fuerzas aplicadas y en general depende del tiempo ¢. de la posicién x y de la velocidad é. Por ejemplo. si suponemos que un cuerpo de masa m cae bajo la sola influencia de a gravitacién, tenemos. considerando um sistema de referencia unidireccional orientado al centro de la tierra F = m-g. g:aceleracién de gravedad = constante , (1.15) donde mg: magnitud tinica debida a la gravedad, Hamada peso del cuerpo. Si y(¢) es la posicién en que se encuentravel cuerpo en el instante t medida desde una cierta posicidn fija. su velocidad v = “ es la raz6n de cambio de posicién, y su aceleracién a = & = £4 es la razdn de cambio de velocidad, ambas con respecto al tiempo. Luego la ecuacién (1.15) se convierte en @y _ Si alteramos la situacién admitiendo que el aire ejerce una fuerza de resistencia proporcional a la velocidad. tenemos dy Famg-k mg dt y la ecuaci6n diferencial asociada es Py _ dy mop =m: TT Trayectorias Ortogonales, Considere una familia a un pardmetro de curvas dada por la ecuacién F(x,y) =e. (1.16) Diferenciando obtenemos Frade + Fydy = 0. donde F, y Fy son las derivadas parciales de F’ con respecto a x e y, respectivamente, Asi, dy__ Fr aT, nos dé la pendiente de cada curva de la familia (1.16). Queremos encontrar otra familia de curvas tal que cada miembro de la nueva familia intercepte a cada miembro de Ja familia (1.16) en dngulo recto: es decir, queremos encontrar las trayectorias ortogonales de la familia (1.16). Debemos tener en cada punto, entonces,que el producto de la pendiente de la curva que pasa por ese punto de Ila familia (1.16) y la pendiente de la correspondiente trayectoria ortogonal debe ser igual a —1. Asf, teniendo en cuenta (1.17). Ja solucién general de la ecuacidn diferencial (1.17) dy _ Fy day Fy’ nos dé las trayectorias ortogonales de la familia (1.16). (1.18) Figura 3 Hay muchas interpretaciones fisicas y usos de trayectorias ortogonales: 1. En campos electrostéticos las Iineas de fuerza son ortogonales a las Iineas de potencial constante, Asi, partiendo de las Iineas de potencial se pueden encontrar las Iineas de fuerza tal como indican (1.16). (1.17) y (1.18). 2. En flujos de fluidos bi-dimensionales las Ineas de movimiento del flujo son ortogonales a las meas equipontenciales del flujo (ver Figura 3). 3. En metcorologfa las trayectorias ortogonales de las ésobaras (curvas conectando los puntos que reportan igual presién barométrica) indican la direccién del viento desde areas de alta a baja presién. Estas son algunas aplicaciones de ecuaciones diferenciales. Otras ser4n desarro- Iladas en detalle a lo largo del texto. Capitulo 2 Ecuaciones Diferenciales de Primer Orden 2.1 Preliminares Dado un subconjunto A C R? y una funcién f : A > R, consideramos el problema de encontrar un intervalo 7 Cc R y una-funcién diferenciable ¢ : 7 > R, tal que para todot € J, fa) Lae dy () #( = Ft. o(0)- Este problema es llamado: ecuacién diferencial ordinaria de orden 1, y es denotado por y' = f(t,y)- (2.1) Si tal fumcién existe y verifica (a) y (b) en Z, entonces ¢ es Tamada una solucién de (2.1) en I. 2.2 Ejemplos preliminares Consideremos primero el caso en que f es independiente de y. es decir, consideremos la ecuacién. y=f(). (2.2) donde f esta definida en algtin intervalo 7. El problema es encontrar una funcién diferenciable ¢ en 7, tal que ¢'(t) = f (1). Este problema ya fue estudiado en célculo, y sabemos que si f es continua sobre J, la funcién ¢ definida por t v= | flsas, donde fg es algiin punto fijo de I, es una solucién de (2.2), Ademés si ¢ es cualquier solucién de (2.2), entonces para todo ¢ € '(t) = f(t)- ft) =0 e-¥ Jo que implica que existe una constante ¢ tal que dH=W(t)t+e, WET. (2.3) Observe que cualquier funcién de la forma (2.3) es solucién. Asi todas las soluciones de (2.2) son conocidas en el caso en que f es continua sobre J, y tado se reduce a un problema de integracién. Consideremos ahora la ecuacién yl =ky. (2.4) que sirve de modelo para el crecimiento de poblaciones (ver (1.11)) si k > 0.0 de desintegracién radiactiva (ver (1.12)) sik < 0. Primero que nada tenemos la solucién obvia 4(t) = 0 para todo t*€ BR, Podemos también verificar facilmente que dada una constante ¢ arbitraria. funciones de Ja forma H(t) = cel (2.5) son soluciones, que estén definidas para todo ¢ € R y nunca se anulan sic #0. Por otra parte si (2) es una solucién cualquiera de (2.4), que es positiva en un intervalo I. tenemos para todo t € T v(t)? Jo que implica, vfa integracidn, que para to fijo en J (2.6) y haciendo la sustitucién u 8). obtenemos vO 4 k(t = lo) = [ =du = In| v(to) Ue Finalmente. exponenciando se obtiene para todo ¢ € I UE) = Yih“ (2.2) que es de la forma (2.5) y est en realidad definida para todo t € R. Si partimos ahora con una solucién q que es negativa en el intervalo 7, en (2.6) debemos hacer la sustitucién w= —7)(s) obteniéndose y se llega nuevamente a la funcidn (2.7). Estableceremos en Ia seccién subsiguiente, que para condiciones bien generales sobre la funcién f(é.y) (que son verificadas por f(t,y) = ky). si dos soluciones ¢ y u de y' = f(t,y) definidas sabre el mismo intervalo abierto J, coinciden en un punto fy € J, entonces P(t) = H(t) para todo t € J. Usando esto podemos entonces canchiir, que todas las soluciones de (2.4) son de Ia forma (2.5). 2.3. Campos de direcciones e isoclinas EI problema planteado por la ecuacién y! = f(t,y) tiene una interpretacién geomé- trica simple. Para facilitar, supongamos f esta definida para todas los valores de ¢ e y. Entonces para cada punto (f,y) en el plano tenemos el ntimero real f(t, y). { Qué significa este ntimero ? Nuestros ejemplos sugieren que es la razén de cambio de y con respecto de ¢ en ese punto. Para interpretar esto geométricamente pensemos en una Iinea recta, 0 un segmento de linea recta, de pendiente f(t, y) por el punto (t.y). La longitud de tal segmento no importa; solo su pendiente interesa, Asi a tado punto del plano es asignado un segmento de linea recta 0, lo que es lo mismo, una pendiente. Observe que no hay segmentos paralelos al eje y, tales segmentos tienen pendiente infinita y corresponderfa a puntos en donde f no esta definida. La correspondencia de Ineas rectas (0 pendientes) a puntos del plano (¢,y) seré llamado un campo de direcciones. La Figura 4 muestra una parte del campo de direcciones de la ecuacién dy/dt = ky con k = 2. El cardcter auténomo de la ecuacion (f es independiente de £) se refleja en el hecho que puntos sobre una linea paralela al eje 1 se les asigna la misma pendiente: la pendiente en el punto (t.y) es 2y, y por lo tanto es independiente de #. No S ~~ ~~ NN NS ‘1/1 / \SAYNQN SS 7/1 / \SNYANNSSTE SASLSSLSL \\\Q NSN ace < MN ADL EEESS 1 Sf St ASS A ANNNSNNNSNAN ND =ESEE57457 V\VNAVANANNANY ST 2557574 1, VVAVAVVVAN SBA GL IN Figura 4 Figura 5 La Figura 5 muestra el campo de direcciones de la ecuacién He poy, es) En ambos ejemplos. la funcidn f esta definida en todo el plano (¢,y). Ejemplos de ecuaciones diferenciales définidas solamente en una parte del plano son dy/dt = Vij, que esta definida solo para ty > 0, es decir, en el primer y cuarto cuadrante 1 incluyendo sus fronteras, y la ecuacién 4 = = que esta solamente definida ¥ para 2 + y? <1, es decir, en el interior del cfrculo de centro en el origen y radio 1. Las curvas f(¢,y) = constante son Hamaatlas isoclinas de la ecuacién diferencial y' = f(t.y). A menudo es més facil dibujar el. campo de direcciones si previamente se dibujan algunas isoclinas. Observe que las isoclinas de una ecuacién auténoma son rectas paralelas al eje ¢. Las isoclinas de la ecuacién (2.8) son las rectas t — 2y = c. Supongamos ahora que podemos encontrar una curva en el plano (t,y) que es tangente en cada uno de sus puntos a Ia recta asignada a ese punto por la ecuacién diferencial y! = f(t-y). Tales curvas tienen. por definicién, una tangente en cada uno de sus puntos, Como estas tangentes nunca son paralelas al eje y, Ja curva es el grafico y = ¢(¢) de una funcidn ¢ definida sobre algtin intervalo del eje ¢ y tiene derivada en cada punto de este intervalo, La pendiente de la recta tangente a la curva en cada punto (¢.4(2)) es, por definicién, la pendiente f(¢,4(E)) asignado a ese punto. Por otra parte, esta pendiente es igual a ¢'(t). el valor de la derivada de gent, Ast #(t) = f(t, @(0) para todo f en el intervalo de definicién de ¢. Luego @ es una solucidn de la ecuacién diferencial. Inversamente, si tenemos una funcién ¢ que es una solucién de y! = f(t.y). es obvio que su grafico y = ¢(£) es tangente en cada uno de sus puntos a la recta asignada a ese punto por la ecuacién diferencial. Llamaremos curva solucién al gréfico de una solucién, Las Figuras 6 y 7 muestran algunas curvas solucién de las ecuaciones (1.11) con k = 2 y de (2.8), respectivamente, Figura 6 Figura 7 2.4 Existencia y unicidad de soluciones Consideremos la ecuacién diferencial dy _ 42/8 a Es facil comprobar que las funciones1().= 0 y v(t) = #2 son soluciones. que ambas estén definidas para todo t € R, y que-u() = v(0) = 0. Es decir ambas curvas solucién pasan por el punto (0.0) y por supuesto u # v. Por otra parte si consideramos la ecuacién wey ly-ts que esta definida solo sobre la recta y = ¢. podemos comprobar ficilmente que no tiene soluciones, En efecto, nuestra funcién f(t, y) = 0 para todo (f,y) en su dominio de definicién, Si existiera solucién u. esta tendrfa que estar definida sobre algiin intervalo abierto a < t < by para cada ¢ en ese intervalo deberfamos tener: (t, u(t) pertenece al dominio de definicién de f, lo que implica u(¢) = t: y ademas w(t) = f(t, u(e)) = 0. lo cual no es posible. Estos ejemplos nos muestran que es posible que una ecuacién diferencial no tenga soluciones, y que también es posible que dos curvas soluciones diferentes de Ia misma ecuacién diferencial de primer orden se intercepten entre si. Queremos excluir tales casos. Queremos estar seguros que una curva solucién pasa por cada punto de la regién donde la ecuacién diferencial esta definida, y también deseamos estar seguros que solamente una curva solucién pasa por cada punto. La primera de estas demandas parece més razonable que la segunda. Hay muchas razones puramente matematicas para esta tiltima, pero las més simple es més cientffica que matematica, Cuando decimos que un sistema fisico es gobernado por una ecuacién de primer orden dy/dt = f(t.y), queremos decir que si en el instante fg el sistema est en el estado yp, sus estados futuros estin completamente determinados por la ecuacién diferencial - esto es. guiados por el campo de direcciones. De esta forma una ecuacién diferencial puede gobernar un sistema fisico solamente sia un punto dado (tc. Yo), le corresponde exactamente una solucidén u que satisface u(to) = yo. Cuando este es el caso, decimos que la ecuacién tiene la propiedad de la unicidad. Afortunadamente, es posible garantizar que una ecuacién tiene la propiedad de la unicidad si la funcién f tiene ciertas propiedades que pueden ser verificadas directamente. Para establecer este resultado necesitamos una definicién, Decimos que un sub- conjunto D del plano es abierto si tado punto de D es el centro de un rectangulo abierta que esté contenido en D. Mas precisamente. D es abierto si para todo punto (to. yo) en D. existen ntimeros positivos a y b tales que cualquier punto (¢, y) satisfa- ciendo | 1% |< ay | y—yp |< b también pertenece a D. El plano mismo es abierto; otros ejemplos son (1) el conjunto de los puntos (é, 4) satisfaciendo (? + y? < 1; (2) el conjunto de los puntos (t,y) satisfaciendo y < 0; (3) el plano menos una linea. Ejemplos de conjuntos que no son abiertos son (1) una linea; (2) el conjunto y > 0: (3) el conjunto 0 Runa funcién continua, Suponga ademds que f tiene derivada parcial con respecto a y en todo punto de D y que Of /By es continua sobre D. Sea (to, yo) un punto de D. Entonces la ecuacién diferencial dy/dt = f(t,y) tiene una solucion u definida en un intervalo alrededor de te que verifica U(ty) = yo. Mas atin, sé v es una solucion definida en el mismo intervalo que u, y u(ta) = yo, entonces v =u. El punto (to, yo) es frecuentemente Hamado una condicién inicial, y el problema de encontrar una solucién u tal que u(tc) = yo es Hamado un problema de valores iniciales, El Teorema (2.4.1) podrfamos interpretarlo diciendo que bajo ciertas cir- cunstancias un problema de valores iniciales tiene una tinica solucién. Si pensamos un poco nos daremos cuenta que esta no es una descripcién muy correcta del teo- rema, y més atin nos darfamos cuenta que no hemos sido muy precisos al introducir el concepto de unicidad. Como lo hemos planteado ninguna ecuacién de primer orden tiene la propiedad de la unicidad. En efecto, si tenemos una solucién cuyo grafico pasa por un punto dado, siempre podemos encontrar otra solucién diferente cuyo grafico pasa por el mismo punto: basta tomar como intervalo de definicidn de la segunda solucién un intervalo un poco mas pequefio que el intervalo de definicién de Ja solucin original, y definir la segunda solucién de la misma forma que la primera. Sea u una solucién de dy/dt = f(t,y) definida sobre un intervalo J, Sea v una solucién definida sobre un intervalo J' que contiene al intervalo J pero no coincide con él. Si u(t) = u(é) para todo ¢ en J, diremos que w es una continuacién de u. El grafico de u es entonces simplemente una parte del grafico de v. Si u no tiene continuacién, diremos que u es una solucién maxima. Asumiendo que f satisface el Teorema (2.4.1), uma solucién definida sobre un intervalo de la forma [a,b] no puede ser maxima, En efecto, si ponemos fy = by ys = u(t) y aplicamos el teorema al punto (¢1.y1), obtenemos una solucién v definida sobre un intervalo [ay, by), con ay < tr 00 0 u(t) > —00 cuando ¢ se aproxima a ese punto. El siguiente teorema nos dice que esta es la tinica posibilidad. Teorema 2.4.3 Suponga que f y Of /dy estén definidas y son continuas en todo el plano. Sea u solucién méxima de dy/dt = f(t.y). Si el intervalo de definicién de u tiene un punto extremos finito a, entonces | u(t) | oc cuando t > a. 2.5 Solucién general y problema de valores ini- ciales Cuando nos enfrentamos a una ecuacién diferencial nuestro primer impulso podria ser tratar de encontrar todas sus soluciones. También idealmente nos gustarfa es- cribir estas soluciones en término de funciones bien conocidas. Esta puede hacerse para muchas ecuaciones importantes. Por ejemplo en la subseccién (2.1) se deter- miné que toda solucién de y' =f. (f continua), (2.9) es de la forma : w= | f(s\ds+c, (2.10) Is donde fo es algtin punto del intervalo donde f esta definida, y c es una constante También mostramos que todas las soluciones de (2.11) son de la forma g(t) = ce, (2.12) donde ¢ es una constante, Diremos que una familia de funciones, que generalmente depende de un cierto niimero de constantes, es la solucién general de una ecuacidn diferencial, si toda solucién de dicha ecuacién pertenece a la familia. De esta forma (2.10) y (2.12) son, respectivamente, las soluciones generales de (2.9) y (2.11). Pero algunas veces, queremos encontrar soluciones particulares que cumplan cier= tas caracterfsticas especiales. De ellas conviene destacar a las que son solucién del problema de valores imiciales Y=f(ty). yllo) =u (2.13) Observe que si f(t-y) verifica las. condiciones del Teorema 2.4.1, entonces el problema de valores iniciales 2.13 tiene una nica solucién maxima. En general nos referimos a esta solucién. como la soluciét particular de a ecuacién y! = f(t.) que verifica la condicién y(to) = yo 0 como Ja curva solucién que pasa por el punto (to, yo). Ejemplo 2.5.1 Encuentre la solucién particular de la ecuacidn y! = 2y que verifica yl) =2 La solucién general de nuestra ecuacién es y(t) sce, LER. Para encontrar nuestra solucién particular resolvemos la ecuacién = c= 2%. Luego la solucién buscada es y(t) = 2e77e* LER. o bien y(t) = 22) LER. 2.6 Ecuaciones de variables separables Para ecuaciones de la forma | dy = OMY) (2.14) es posible, en ciertos casos, encontrar férmulas para las soluciones. La técnica que se es conocida como separacién de variables, y una ecuacidn de la forma (2.14) es Hamada una ecuacidn de variables separables. Observamos que los sistemas auténomos son ejemplos de estas ecuaciones. Daremos una simplificada descripcidn de esta técnica, que nos permitiré desa- rrollar rigurosamente algunos ejemplos. Supongamos que la ecuacién diferencial (2.14) est definida en un conjunto A = Ix J, donde I y J son intervalos abiertos de la recta real, y que las funciones g:1 Ry h: JR son continuas Primero observamos que si para yp € J tenemos h(yo) = 0, entonces la funcién constante y(t) = yy. t € I es solucién de nuestra ecuacién diferencia. Sea ahora Jp un intervalo abierto contenido en J, tal que h(y) 4 0 para todo y € Jy y que sea maximal con esta propiedad (es decir. si existe intervalo .J, que contiene a Jp tal que h(y) 40 para.totlo y € J,. entonces J, = Jp). Para (t.y) € I x Jo, podemos escribir la ecuacién de la forma dy iw (0) y ast las variables son separadas dejando solamente y's a la izquierda y ¢’s a la derecha, Tomamos ahora integrales indefinidas a ambos lados, obteniendo wy _ fay yn | aeitte. donde c es una constante arbitraria. No es necesario poner una constante arbitraria al lado izquierdo, ya que esta constante puede ser combinada con Ia del lado derecho. Esta tiltima ecuacién puede ser escrita Hy) =Gl) +e, donde Hy G son las integrales indefinidas de 1/h y g. respectivamente. Ahora resolvemos esta ecuacién para y. Para cada c (posiblemente restringido a cierto rango), la funcidn resultante nos dard una o mis soluciones de la ecuaci6n diferencial (2.14), Ejemplo 2.6.1 dy oye a (2.15) La ecuacién esta definida en A = R x R= R? y h(y) = y? se anula solo para y = 0. Luego ya tenemos la solucién constante y(¢)=0. £€ R. Para y # 0 separamos variables dy = 2ldt , - y tomamos integral indefinida a ambos lados: aete. y Resolvienda para y abtenemos 1 -— ) ’= a (2.16) ‘Verifiquemos que para cada ¢, la funcién ¢(t) = fe satisface (2.15). Diferenciando obtenemos ¢'(!) = gize. y como también 24(¢(4))? = Gear. tenemos ¢/(t) = 24((t))? y la ecuacién es satisfecha. Ahora separamos estas funciones en soluciones, es decir, funciones definidas sobre intervalos. Para toda c positiva, la formula (2.16) nos da una funcién definida para todo t. Todas estas soluciones son negativas; sus graficos son las curvas mostradas en la Figura 41. Para c = 0, obtenemos dos curvas, definidas respectivamente para 1 < Oy ¢> 0; ambas son negativas, Para cada valor negativo de ¢ obtenemos tres curvas, Una de estas es positiva y est4 definida para —/=¢ < 1 < =e las otras dos son negativas y estén definidas parat @-/—c y > \/=e, respectivamente. Note que todas las soluciones obtenidas son. maximales. y \ A Figura 8 i Como podemos asegurarnos que no hay otras soluciones maximales ? Esto se responde usando el teorema de existencia y unicidad (Teorema (2.4.1)). Como f(t-y). que es igual a 2ty?, y 5f(t,y)/Ay, que es igual a 4ty. son continuas en todo el plano, una y solo una curva solucién pasa a través de cualquier punto dado (fe. yo). Si nos damos cuenta que hay un punto que no esta en ninguna de las curvas solucién que hemos encontrado. entonces claramente nuestra familia de curvas solucién es incompleta. Por otra parte, si por cada punto (tp, yp) podemos encontrar en nuestra familia una solucién u que verifica u(to) = yo, entonces nuestra familia es completa. La primera afirmacién es consecuencia de la parte de la existencia del teorema: por todo punto (tc. yc), existe una solucién w tal que (tc) = yo. La segunda afirmacén es consecuencia de la parte de la unicidad: no puede haber més de una curva solucién a través de cualquier punto; si nuestras soluciones cubren todo el plano, no hay otras. El asunto se reduce a: dado un punto (to. yo), 3, hay un ntimero ¢ tal que yo = ee ? Resolviendo esta ecuacién obtenemos ¢ = —£3 — 1/yp, si yo # 0. { Que pasa si yo = 07 En este caso, la solucién es la funcién constante y(é) =0. £€R. Luego. mceR Ia familia { y(t) y(t) = 0 con £ en el intervalo que coresponde segtin c, es la solucién general de nuestra ecuacién, Ejemplo 2.6.2 Ay a (2.17) Esta ecuacién tiene la solucién constante y= 0. Para y #0, la ecuacién se escribe Si y > 0 integrando obtenemos Ing) =t+e. Para despejar y, tomamos exponencial a ambos lados: ya =e =kel, donde k es una constante positiva arbitraria. Si y < 0 la ecuacidn se puede escribir de la forma dy staat. e integrando se obtiene In(-y)=tt+e = -y=et sete =ke'~ y luego donde & es una constante positiva arbitraria, Asi y=tkel. dependiendo de si y > 0 0 y <0. Esta formula es equivalente a y=ce. (2.18) donde ¢ es una constante no nula arbitraria. Diferenciando (2.18) se obtiene dy/dt = cc! = y. luego (2.17) se verifica. También notamos que c = 0 produce la solucién constante, por lo que (2.18) es solucién para todos los valores de ¢, Todas estas soluciones estén definidas para todo ¢. Finalmente. observamos que yp = ce! tiene la solucién ¢ = yee", por lo que las curvas (2.18) cubren todo el plano, Como (2.17) tiene la propiedad de la unicidad, hemos encontrado todas las soluciones. Algunas cutvas solucién son mostradas en la Figura 42. | Figura 9 Ejemplo 2.6.3 dy = cos*(y). 19) aq ow) (2.19) Las soluciones de cos?(y) = 0 nos dan una cantidad infinita de soluciones constantes de Ia ecuacién diferencial. Estas son y = (n + 4), donde n es un ntimero entero arbitrario. Si cos*(y) # 0, podemos escribir dy ww Como 1/cos*(y) = sec?(y) y f sec2(y)dy = tan(y), tenemos. después de integrar tan(y) =t+e. (2.20) Para resolver esta ecuacién para y, podemos simplemente poner y = arctan(t + 0) Si hacemos esto, tendrfamos que asumir que y est entre —1/2 y 7/2; y todas las soluciones de (2.19) obtenidas de esta forma estarfan en esta franja. En lugar de esto, debemos asumir que y est en alguno de los infinitos intervalos abiertos cuyos extremos son ceros consecutivos de cos?(y). Supongamos, entonces, que 7/2 + nt < y < /2+nm7 para algtin entero n, y asi, -7/2 < y—na < 7/2, Como tan(y) = tan(y — nm), podemos escribir (2.20) de la forma tan(y— nm) =t+e y aplicando tangente inversa a ambos lados obtenemos y= nm + arctan(t +c). (2.21) En esta férmula c es arbitraria, y n es un entero arbitrario. Dejamos al lector verificar que las funciones definidas por esta formula son realmente soluciones, y que junto con las soluciones constantes cubren el plano, Como (2.19) tiene la propiedad de la unicidad. tenemos que todas las soluciones han sido encontradas. Note que cada una de ellas esta definida para todo t. Ejemplo 2.6.4 dy _ an Como la secante no tiene ceros, no hay soluciones constantes. Separando variables e integrando obtenemos 2tsec(y). —n/2 VIF = 5, y elevando al cuadrado 1-2-2 = Volviendo a las variables X,Y, obtenemos -2=—-=— = 2 -Y= 1-25-55 = yee KY =e, y en las coordenadas «x.y, la solucién general (en forma implicita) wv —2w(y—2)-(y-2 =e, cER. 2.7 Ecuaciones Diferenciales Exactas y Factor In- tegrante Consideremos ecuaciones diferenciales ordinarias de la forma M(x. y)dx + N(x-y)dy = 0. (2.25) Definicién 2.7.1 Diremos que la ecuacién (2.25) es una ecuacién diferencial exacta en D, si existe una funcién u(a,y) tal que Ou pe) = (x.y) V(ay)€D. En tal caso Ou Ou 7 du = 5g tt + ay! = M(x.y)dt+N(x.y)dy = 0, y (2.25) es equivalente a du = 0. Luego y=yle), ©e€T essolucién de (2.25) = $ u(z,y(r))=c, VaeT Proposicién 2.7.2 Sean M,N funciones con derivadas parciales continuas de pri- mer orden en una bola abierta B.C R?. Entonces M(x,y)de + N(a,y)dy = 0 es exacta en B= w= Demostracién. Suficiencia. Supongamos existe u tal que 2 = M y B, Entonces OM _ Ou _ du _ a (2) = aN dy ~ DyOe ~ Budy ~ Te \Dy) ~ De 2% en B. Debemos encontrar u tal que 24 = M y Necesidad. Sabemos que Bones. Fijemos un punto (20, yo) en B. Para tener 24 = M definimos para («.y) € B 2 = [ M(s,y)ds + g(y), 20 con g(y) a determinar, para que se cumpla también 24 = N, Entonces Qee.y) = [ s,s + oy) x Yy * ON = | o =_(s.y)ds + g'(y) N(a,y) — N(ao-y) + 9'(y)+ Por lo tanto debemos tener 0 = N(t,y) -[ N(2to. dt. y nuestra funcién buscada es u(x.y) = [ M(s,y)ds + f N(qp, t)dt. 0 0 Ejemplo 2.7.3 Resolvamos (32? + Gay? )dx + (62?y + 4y*)dy = 0. Tenemos M(sx, y) = 322 + Gry? y N(x, y) = 6x2y + 4y*. Por lo tanto OM ON a = = Diz.y), ay rey) = Bay = Fee) para todo (1,4) € R2. y la ecuacidn es exacta: Pongamos u(z,y) = [oe + Gry?)dx + g(y) = o + 32°y? + gy). Asi Fee) = Gry + gy). y como N(a.y) = Gx°y + 4y', debemos tener gy) = 4? = oy) = V- Por lo tanto u(z,y) = 23 + 327y? + y'. Asf, las curvas soluciones (x, y(x)) de la ecuacién satisfacen + 307 +yf=c, cER. Definicién 2.7.4 Si la ecuacién M(x.y)du + N(x-y)dy = 0 no es exacta en D, se llama factor integrante a toda funcién = p(x. y) definida en D tal que a(x, y)M(a,y) + u(r.y)N(x.y)dy = 0 es una ecuacion exacta en D. Observaciones 2.7.5 Es claro que toda solucién de y = y(x). 2 € 7, de wMdx + wNde = 0, que verifica: u(x. y(x)) esta definida y p(x. y(x)) 40 para todo x € 7, es también solucién de Mdx + Ndy =0 en I. Ejemplo 2.7.6 La ecuacién yl + xy)dx — xdy = 0 (2.26) no es exacta. Sin embargo para y # 0 la ecuacién multiplicada por pu(z..y) = 1 ul + 2) ay — ~ay = 0, P ye es decir 1 1 (+ ade ~ Say = 0, (2.27) es exacta. Ast u(2.4) = 2 es factor integrante de (2.26) en el conjunto D = {(x-4)/y #0}. Para resolver (2.27) ponemos Pero como debemos tener JY) =0 => gy)=e. Por lo tanto, toda solucién y = y(x) de (2.27) verifica la ecuacién implicita 2 cia i Ft He parwalaim ee R. Despejando y obtenemos yl) = ae, ce R. para x tal quo Xe - 2? 40 ue) = 5 :Pa q . Observacién 2.7.7 Para que 1 = 14(x,y) sea factor integrante de Mdr+Ndy =0 debemos tener Jum = 2 uN), oy Ox an" Jo que implica Op a + 1 (du =~ i (Ft a(n) = aN (2.28) La ecuacién (2.28) es una E.D.P. conocida como la ecuacién del factor inte- grante. Se puede demostrar que esta ecuacién, bajo condiciones bastante generales, siempre tiene solucién. El problema es encontrar sus soluciones. 2.7.1 Casos en que es facil encontrar el factor integrante Q Si 1 (dM _ aN) _ \ ¥ ( ayo @) = f(x) (depende sdlo de x), entonces nuestra ecuacién admite factor integrante que depende solo de x. En efecto si u=yi(x) la ecuacién (2.28) queda de Ja forma addn(u(e))) 1 (a an ~ dy Or wr) = It ) ea) = fe): Jo que implica r z In(u(e)) = [ f(sjds— obien p(x) = elo, De 2) Si at (2M _ aN M \Oy7 Oe entonces nuestra ecuacién admite factor integrante que depende solo de y. ) = gly) (depende s6lo de y). En efecto si «= p(y) la ecuacién (2.28) queda de la forma A(ln(u(y))) _ =1 (+ aN Oy Ox On 7] Jen = oy): Jo que implica ott)at nly) = eb Ejemplo 2.7.8 Resolvamos 2 (20 + ey + ) dx + (x? + y)dy = 0. Poniendo 3 M(e.y) = ary +ary tS, y Mey) = 22 +9, tenemos . OM = on pa? yy? ON = on Oy WY ap = aes y la ecuacién no es exacta. Pero 1 (9M _ aN lo, > W (Gp ~ Be) = rape = Por lo tanto. el factor integrante es p(x) = ef =e, Nuestra ecuacién multiplicada por el factor integrante es 240 2442 & (Quy taty + >) de + e(a" +y')dy = 0. Entonces a yt uty) = eT ( Qey ta yt dt + g(y) = wy [wear + y [ Pear) + x + gy) 3 = 2y(xet et) + v(ee 2 [ rear) + ge + gy) = ye? + De" + gly). Pero a TD — Nay) = Pete tqy) — a? +7) = gy). Por lo tanto Ast y la solucién general es ye ( Ejemplo 2.7.9 Resolvamos fae + (y?=In(2))dy = 0. Poniendo y M(a,y) = g, y Nay) = y®-Wn(2), tenemos aw 1, aN = oy or! Yor , y la ecuacidn no es exacta. Pero 1 (= =) _ yl 2 dy Ox implica que el factor integrante es 1 wy) = 2/4 = er2in) = Gh) = = Nuestra ecuacién multiplicada por el factor integrante es 1 In(zx) _ wt t (0-2) ay =0. Luego 1 1 = — = ) . u(x.y) = [se + g(y) yin) + gly) Pero du yy = bh). h@) _ aye) N(a,y)= ee +0'(y)-yt eT Jy): Jo que implica a ¥e Jy) =y = gy = 5. Por Io tanto u(t.y) = tina) 4 2 “y) = Ga) +5: y la solucién general satisface la ecuacién implicita. 1 =In(x) +2 =, . ne) +5 c. ceR 2.8 Ecuaciones Lineales Consideremos la ecuaci6n lineal de primer orden. dy = ') a way +b). (2.29) y supongamos que la funciones a(t) y B(f) estan definidas y son continuas so- bre un intervalo abierto 7. Consecuentemente la funcién f(t.y) = a(t)y + b(t) y (Af /3y)(t.y) = a(t) son continuas y luego nuestra ecuacién tiene la propiedad de la unicidad, Mostraremos en esta seccién que las soluciones de (2.29) pueden ser expresadas por medio de una férmula en la que aparecen integrales. Esta formula debida a Leibniz, puede ser derivada de varias formas. Presentaremos primero un método de resolucién que aunque no es el més corto, tiene la ventaja que la idea basica puede ser aplicada a ecuaciones lineales de orden superior y a sistemas lineales. La ecuacién (2.29) se dice homogénea si b(/) es idénticamente cero sobre I. y no-homogénea si existe ¢ € I tal que b(¢) #0. La ecuacién homogénea dy Tt = a(t)y (2.30) se resuelve usando separacién de variables. Las soluciones estan dadas por y(t) = cel 0% (2.31) y estan por lo tanto definidas sobre todo el intervalo J. Para discutir la ecuacién no-homogénea, consideramos simulténeamente con ella la ecuacién homogénea obtenida suprimiendo el término b, La ecuacién homogénea obtenida de esta forma es Hamada ecuacidn reducida; la ecuacién no-homogénea misma es referida como la ecuacién_completa. Sea u(1) cualquier solucién de la ecuaci6n reducida que no sea la solucién.cero. Intentaremos encontrar una funcién v(é) tal que u(t)u(t) sea solucién de la ecuacién completa. Asumiendo que tal v existe, sustituyendo y por vu en (2.29), se obtiene (v(t)u(t))! = a(t)o(tju(t) po(t) « vl (u(t) + vu (t) = a(t)u(t)u(t) +b). Como w'(t) = a(t)u(d). la Ultima ecuacidn se reduce a v'(Qu(t) + a(tju(éo(t) = a(tu(tju(t) + b() . ul(t)u(t) = 0). Como u(f) es no nula, UG) y por lo tanto ( b(t) u(t) = [are De esta forma obtenemos Ja formula ) yD) = wc] ww +0. (2.32) Es muy facil verificar que esta férmula nos dé una solucién de (2.29) para todo c: 4 worf Barxo wot = atu f APar+c) +60 = a(t)y(t) + b(t). Hemos obtenido asf una cantidad infinita de soluciones de (2.29). Mostraremos que en realidad las hemos encontrado todas. Para chequear esto denotemos la integral [24 por Q(), y ast Ja formula queda y(t) = u()(Q(t) + ©). Poniendo 1 = to,y = yp. obtenemos c= 74, —Q(to), y por lo tanto podemos realizar cualquier condicidn inicial. Note que todas las soluciones estan definidas sobre todo el intervalo 1. Para completar la derivacién de la formula prometida, reemplazamos la funcién u(t) en (2.32) por ef 84, Establecemos el resultado como un teorema Teorema 2.8.1 (Formula de Leibniz) Las soluciones de (2.29) estart dadas por y(t) = ef wat | eo LOMB()dt +0). (2.33) donde t esté en I yc €s una constante arbitraria. La integral indefinida f a(f)dt. que apatece dos veces en (2.33). puede ser elegida como queramos, pero debe ser la misma en ambos lugares. El lector puede facilmente verificar que la solucién de (2.29) que satisface u(fe) = yp esta dada por: u(t) = om pt eS (s)ds + yp] (2.34) Ejemplo 2.8.2 1 dy _, 1 pnt Aqui a(t) = 2 y b(t) =e. Sustituyendo en (2.33): y(t) = la fe Petat +0) = ef eeldt +0) = af eval +6) = cet e, El método con que fué derivado (2.32) es conocido como variacién de pardmetros. En efecto. una solucién de la ecuacién completa se obtiene permitiendo que el pa- rémetro ¢ en la formula y(t) = cu(t) para la solucién de la ecuacién reducida varte. es decir, sea una funcién v de ¢ Otro método de resolucién consiste en escribir (2.29) de la forma (a(t)y + b(t))dt — dy = 0 (2.35) y darnos cuenta que admite factor integrante que depende solo de la variable t. Luego aplicando los métoados de la seccién anterior encontramos las soluciones. En efecto, si M(t.y) = alty +b), y N(ty) = -1. tenemos + (= ON (== = -a(t). a (Fe SP) en = at y luego tenemos factor integrante HE) = Sot, La ecuacién (2.35) multiplicada por si(é) es eI (a(ty + b()\dt = eS dy = 0. (2.36) De esta forma u(t.y) = = [cleans A(t) = yew FX + D(A), Hey) = ya(tyen LO + n'(t). Comparando con (2.36). obtenemos W(t) = bien" => A(t) = / bier LO" at y por Io tanto day) = myer FeO Per Oat, De esta forma y = y(t) es solucién de (2.29) en el intervalo 7, si existe constante ¢ tal que perf [ayer Id = cy vel, Despejando y obtenemos nuevamente la férmula de Leibniz tat (« + [oye toot) . Ejemplo 2.8.3 Usando este tiltimo métado encontremos las soluciones de w = ytan(x) + cos(x) Escribiendo la ecuacién de la forma (ytan(x) + cos(x))dx — dy = 0. tenemos a(x) = tan(x), y el factor integrante es pla) = en ftenere = cf Pde ln(eos(e)) cos(z) . Multiplicando por el factor integrante nos queda la ecuacién (ysen(x) + cos*(x))da: — cos(x)dy = 0, y luego u(.y) = ~ festenty + h(x) = —ycos(x) + h(x). Como a ony) = q Foley) = genx) + h(x). debemos tener (a) = coP(2) => h(x) = [cece = 5 [ (+ es2n)ar 1 1 = 52 + pene). Luego u(z,y) = -yeos(x) + bn + Foen(2z), y las soluciones y = y(z) deben verificar -ycos(z) + =x + =sen(2r) = ¢ y 3 7 Jo que implica 11 y(x) = 5 |5t + zeenlr) + 2.9 Ecuaciones que se reducen al caso lineal 2.9.1 Ecuacién de Bernoulli Son ecuaciones de la forma = +p(x)y = f(xjy", con n#l. Multiplicando por y-” obtenemos 2 nay = fle). dx Haciendo el cambio de variables z = y'~". tenemos dz _ yon dy s , Go mya = A l= pee + F(a), es decir be @ a _n)pla)z = (1- ) at = npla)z = (=n) fl), que es lineal. Ejemplo 2.9.1 Resolvamos la ecuacién de Bernoulli dy 4 m7 ptt Tenemos n = } y multiplicando por y~? obtenemos y nos queda la ecuacién dz 21 de > La correspondiente ecuacién homogénea es dz _ 2 dz" 2 e integrando se obtiene In(z) = 2In(x) +In(c) => Haciendo variar la constante y reemplazando en la ecuacién completa se obtiene dL. 2 1 S = ¢(r)r? +e(2)2x = get ge = da = tr4t-4 2° ' a2 Or = ou) = Fin(a) +e. Luego. Jo que implica (a) = [en = Gin(a) +24 2.9.2 Ecuacién de Riccati Es una ecuacién de la forma dy TE wloyy + aay? = fla). Esta se resuelve si uno conoce una solucién particular, digamos y,(zc). Para ello ponemos y(x) = 2(x) +y(x) y reemplazando se obtiene fy = (x) =X 4 Ma) Fe = Pay = aay? ¥F(2) = Ta) + FA) es decir, d —p()y — ge)? + f(x) = = — pla)an(e) — aa)in(w)? + f(x) i = S = v(x} — y) + a(a)(ut - 7) es decir, de v2 ) Te 7 TP) — (OH + 221) dz F = (We) + alauute))= — a que es de Bernoulli con n = 2. Ejemplo 2.9.2 Resolvamos Claramente la funcién (2) = 4 es solucién particular, Poniendo y(x) = 2(z) + y reemplazando en Ja ecuacién abtenemos dz 1 21 2 dz 2 > 2 SS = P4224 5-5 SS Sera 2. dP roe dc Para resolver esta ecuacién de Bernoulli multiplicamos por 27? aodz 3 dx x y hacemos el cambio de variables w = 27', que implica 4 °2. obteniendo du 2 dr x Tenemos asf la ecuacién lineal de primer orden du 2 yy z dr cuya solucién general es w(x) = fe ([ -e!##ar +0) - 1(f 2 ce = a(/ dete) = _ =#2 + 3¢ ~ Be Luego yy _ 3 2a) = wet = a, y la solucidn general de Ia ecuacién inicial es 1 —2 Observacié6n 2.9.3 Conocida una solucién particular y,(x) de una ecuacién de Ricatti el cambio de coordenadas (x) = y1(x) + by nos lleva directamente a una ecuacién Tineal. 2.10 Ejercicios resueltos Ejercicio 2.10.1 Considere la ecuacién diferencial 2y) y — wy! = a(l+a7y),a>1. a) Encuentre la solucién general. b) Encuentre la solucién particular que verifica y(1) = 4+ c) Encuentre el intervalo m4ximo donde la solucién particular anterior esta defi- nida. Solucién. a) Reordenando tenemos: dy yr-a -a = (ar? / == ——., ¥ (a? a)y = dr (ax +1) y separando variables dy dx, dy dx ade y-a x(at+l) = acti” Luego integrando obtenemos In(y—a) = n@)=In(az +1) +n), y exponenciando y-a = 4 ~ ax+1" Por lo tanto Ia solucién general es cx u(r) =a + To. ceR. b) Imponiendo a la solucién general la condicién y(1) = 47. obtenemos a c a__ @+ate _ at "ta ~~ aio a OO Luego nuestra solucién particular es aa ya) ~ a@+t c) El dominio de la fimcién aa yx) - es la unidn de los intervalos abiertos ‘ell « Hol Como a es positivo, el 1 pertenece al segundo intervalo y Tnego bins | —-, +00] , a Ejercicio 2.10.2 Mostrar que la ecuacién diferencial es el intervalo maximo buscado. 2rtyy + yt = 42° se reduce a una ecuacién homogénea mediante la transformacién y = n. Determine el valor de n y resuelva la ecuacidn. ”. para cierto / Solucién. Si y = 2" entonces 1’ = n2"12/, y sustituyendo en la ecuacién obtenemos 2atz"(n2"*eby + 2" = 4a8 es decir Qnat2""Mde = (409 = 2dr. 3 Para que sea homogénea debemos tener 2n — 12 y dn = 6, Luegon = 5 y tenemos Ia ecuacién homogénea (40° — 2)de = 3x42?dz. Por lo tanto a“ dg’ — 28 1 t\2 2\4 & - Sar - GQ - (3). . dz du Poniendo z = 2u, tenemos = =u-+25>, y reemplazando en la ecuacién dx dx ot oy yg AA Aye 2 route dc 3 3 Be . Luego separando variables Bdu de ue +3ue—4 es decir > 3u? du dx Separando en fracciones a Bu?du_ udu) _ de 5lwee—F we sETE o bien > > Buedu — 3u2du _ Bde wal” w+ x Integrando cbtenemos 28 In (5) = -5In(x) + Ino) y exponenciando De esta forma a _ P44 Ejercicio 2.10.3 La ecuacién (2x°y — 2y°)dy = (32° + 327y?)de se reduce a una ecuacién homogénea haciendo un cambio de coordenadas de la forma x = w?. y = v4, con p.q constantes adecuadas. Encuentre dichas constantes y resuelva la ecuacién, Solucién. Tenemos xr => de = pu du e y 8 => dy = qed. Sustituyendo nos queda (2u5e4 — 2u%)qu!'dv = (3u + 3u??v?*)pul"du, es decir Qq(u8Po°A-! — v4 dy = 3p(ulP! + ui tv2)du, Al hacer 2q = 3p cada término queda de grado 6p — 1. Poniendo 6p — 1 = 1. obtenemos p = zYI=5Y reemplazando en la ecuacién (u — v)dv = (w+ v)du. Esta ecuacidn ya fue resuelta en el Ejemplo 2.6.6, obteniendose la solucién implicita v 1 2 2 -j)-- 2 7) = arctan (2) = 5In(u? +0?) =e. Luego volviendo a las variables originales, se obtiene que Ia solucién general cumple ¥y lity) = aretan (4) - zh t+y*)=c. we . . 1 1 Ejercicio 2.10.4 Haciendo los cambios de coordenadas u = 52%, v = 5°, resuelva la ecuacién (2a? + 3y? — T)adr = (32? + 2 — 8)ydy = 0. Solucién. Tenemos los = a1, us 5 => du = ade y v=5y Reemplazando en la ecuacién obtenemos = dv = ydy. (du + 6v — 7)du — (Gu + dv — 8)dv = 0. Como Ia solucién del sistema du + 6u -—7 Gu + 4u - 8 hacemos el cambio de coordenadas u Fl, v=t44 stl, valts. obteniendo Ia ecuacién homogénea (4s + 6t)ds — (6s + 4t)dt = 0. Asi ds 3s + 2t do Is +3 : d: lz . y poniendo s = tz, tenemos & = 2 +45. y la ecuacién d o bien 2 In(t) + In(c). y exponenciando Por Io tanto (2-1)? (2D? Volviendo a las variables iniciales tenemos (s—t)? =o (u=v-3)? (+08 (u+v-3) I ° y finalmente " ° a T (x? +4? - 3) Ejercicio 2.10.5 Encuentre la solucién general de la ecuacién (w@ — 2y + ddr + (Qe — y + 2)dy = 0. Solucién. Como la solucién del sistema r-%y+4=0 para obtener una ecuacién homogénea hacemos el cambio de coordenadas ceu, y= vt. Entonces da = du. dy = dv, y reemplazando, obtenemos (u — 2v)du + (Qu — v)dv = 0. Para resolver esta ecuacién ponemos 1 = fu. y como dv = tdu + udt,. reemplazando en la ecuacién obtenemos (u = 2tu)du + (2u = tu) (tdu + udt) = 0, es decir (1 — P)udu + (2 - thu'dt = 0. Separando variables se tiene e integranda Por lo tanto us -G=4 +07 P ero bye t=-= : ue implica Luego nuestra solucién esté dada por (w@+y-2) (w—y+2)2 o bien elevando al cuadrado (@w@t+y-28 _ (w-y+2) Ejercicio 2.10.6 Resolver la ecuacién dy _ , (uty+2) dx at+ytl Solucién. Como el sistema ut+y 0 0 ee +2 +1 no tiene solucién, hacemos la Sustitucién «+ y = u. Luego de + dy = du, reemplazando obtenemos dua dx =. (u+2 > dr uti) ~ Separando variables se tiene (u+1P = Q(u+ 2)? + trip = de a 3ue+10u+9 0 1 2 Gu+410 2 1 _ ( ~ he+idus9 * Soa) du = de e integrando 1 2 2 5 gu — 5 ln? + 10049) + F arctan (245) =ate Luego nuestra solucién general satisface 5 5 Flrty) - ; In(at+y)?+10(@+y) +9) + L arctan (Ate) =arte. Ejercicio 2.10.7 a) Muestre que la ecuacién diferencial dy _y y YY gingn (4) dz wIS se transforma en ecuacién de variables separadas usando el cambio de variables y=. b) Use a) para encontrar la solucién de dy y se dy oy, dx r Solucidn. a) El cambio de variables your = Hinge y= de ae Reemplazando en la ecuacién obtenemos dy m+nyn Fy) ute su tay" f(r), +07 + F(2) es decir a w_ ymtn=14, wT” v que es de variables separadas, b) La ecuacién se escribe de la forma By Dg Oy Poe 4) = 2 + ay? sec? (F) mot erie Luego m=0,n=—2 y f(v) =sec?(v) y la ecuacién queda de la forma dv 3 saa. de. u7? sec?(u) o bién wcos*(v}dv = a78dr. Como, 1 1 1 1 / ? cos*(ujdu = Fe + e*sen(2v) + queos(2v) — Zsen(2v) + ©, nuestra solucién w = v(x) verifica eid v e aut (5-2) sen(2v) + Zeos(2u) = —> + ©. Por lo tanto. la solucién y = y(2) de nuestra ecuacién verifica Ia ecuacién dy? + 3xr(2y? = 22)sen (#) + 622ycos (#) 4 er = cr, Ejercicio 2.10.8 Encontre la solucién particular de la ecuacién, )) 7 ) jaa + 2ay? + 6x) de + (ae Py 4. de”) dy = 7 3 7h) + ry? + 4e dy 0 Solucién. Sean Inlay) 2 In(x) - M(a.y) = — + ge + 6x, N(a,y) = an + x7yP + 4e77Y, Como _ 1 2 a ry = ou to = ew. nuestra ecuacién es exacta, Luego para resolverla integramos M(e.y) con respecto ax obteniendo u(r.) = In(hi(y)) In) + bay? + ax? + gly). Entonces du _ nz) oo Wye” = ying TY + gy): e igualando con N(z.y) se obtiene la rela¢ién gy) = 4c". Luego 1 aly) = -2e-. yulrsy) = In(in(y)) In(x) + ya0°y* + 32? — 2°, De esta forma la solucién general satisface In(In(y)) In(x) + sey + 322 = 2° =, 73 2 Ewaluando en (1,3) obtenemos ¢ = F¥— 2. Por lo tanto In solucién buscada y = y(2) verifica la écuacién In(In(y)) In(w) + hat + 3x? — 2674 = aT EFjercicio 2.10.9 Demuestre que ji(z, y) = xy? es factor integrante de la ecuacién (2y — Gr)dx + (3x —4x?y")\dy = 0. Use este factor integrante para resolver Ja ecuacién. Solucién. La ecuacién multiplicada por pu(..y) = xy? queda (ry? — 6x71? )da + (327y? — 42°y)dy = 0. (2.37) Si . . M(a.y) = 2ey?-627y? y N(x,y) = 327 - 4a5y, tenemos oM - 2 992 oN - 2 1992 Fy ew = Gry? - 120*y y Bp ty) = Gry? - 122°. Asf la ecuacién (2.37) es exacta y por lo tanto u(x, y) = 7? es factor integrante de la ecuacién inicial. Para resolver la ecuacién integramos M(x. y) con respecto a a obteniendo ulx.y) = [meena = Py—wiy+h(y). Para calcular h(y) imponemos la condicién du — = Na. uy (x.9) obteniéndose la ecuacién 3a2y? —4e3y + hl(y) = Bay? — Ardy. Luego y)=0 => — hy) =0. Por lo tanto u(x,y) = 22y2 — 222 y la solucién general de nuestra ecuacién esta dada implicitamente por la ecuacién Pp—wy = c. cEeR. Ejercicio 2.10.10 Resuelva el problema de valores iniciales dy rt+y \ ae : =1 or Wy y(0) y determine el intervalo maximo donde esta definida la solucién. Solucidén, Tenemos a = a <=> Dydy = (a+ dv => (x + de = 2ydy = 0. Pongamos Mwy) = t+ => N(wy) = -2y => Entonces aM aN 5) 4g ay Or y la ecuacién no es exacta. Pero como IM, By depende sélo de xr, nuestra ecuacién admite como factor integrante a la funcién ila) = ef“ = Multiplicando nuestra ecuacién por este factor integrante obtenemos la ecuacién diferencial exacta ea + pdx + s2ye"*dy = 0. Luego existe funcién F tal que OF OF _oe 2) — Loy et Te THY ¥ Gps 2ye™ Por Io tanto F(x,y) = / rare | ede + p(y) => F(x,y) = —xe7* Pero —2ye" = = =0-0-2ye* + 4'(y) Jo que implica HY)=0 = dy)=e, y entonces F(a,y) =—me™* — €" —ye*te. Finalmente como para 2 = 0 debemos tener y = 1, el valor de nuestra constante ¢ es 2. Luego la solucién buscada y = y(w) verifica Jo que implica yay =2%t- 2-1. Como y(0) = 1, nuestra solucidn es y(a) = Vee aT. Para determinar el intervalo maximo donde la solucidn esté definida, llamemos f(x) = 2e* - 2-1, Luego f'(x) = 2c" -1y f(x) = 0 = « = —In(2). Como #(—In(2)) = In(2) > Oy f"(z) = 2€? > 0, tenemos que f(x) > 0 para todo x € R. Esto implica que el intervalo maximo donde la solucién esta definida es todo R. Ejercicio 2.10.11 Encuentre la solucién general de Ia ecuacién diferencial (yln(y) - 2ry)dx + (vt+y'e")dy = 0. Solucién. Poniendo M(a,y) =yln(y)-2ry yy Nay) =a +y8e, tenemos ee) Bee) = ny) 20 Luego tenemos el factor integrante que depende.solo de y = ftw = p-iny) — 1 wey) = lt e ; Nuestra ecuacién multiplicada por el factor integrante es (In(y) = 2x)de + G +yYe)dy = 0. Entonces u(z,y) = [ong -2n,¢r + gy) = In(y)x—2? + gly). Ou _ 2 Wy BOD =a + Jy). Comparando esto con ¢ + yPe! obtenemos g'(y) = yet. Asi oly) = [rea = ye - 2 [ veray = Pe’ — Ayer — 2) (y= 29+ Qe". y tenemos u(x.y) = In(y)a — 2? + (y? — 2y + 2c". Luego nuestra solucién general esta dada por la ecuacién implicita In(y)e = 2? + (y? = 2y + QeY = c, CER. Ejercicio 2.10.12 Determine las condiciones bajo las cuales la ecuacién M(x.y)dx + N(x.y)dy = 0 tiene factor integrante de la forma p(x, y) = h(x + y). Solucién. Si p(x.) = h(x + y). tenemos An(u(r.y)) _ W(e+y) Aln(u(a.y)) _ '(a+y) Ox hc+y) * ay hey)? y la ecuacién del factor integrante (2.28) queda de la forma Fete vey) — Men] = Ft Jo que implica _ W(ety) _ , (zy) = Tat = f(a+y). Mu N-M Luego nuestra condicién es = f(«+y) (dependa sélo de x + y). En este caso el factor integrante es w(t,y) = h(a+y) donde h(u) = ef Mau, Ejercicio 2.10.13 Use lo anterior para encontrar la solucién general de: (7a® + 3x°y + 4y) dx + (42° + 2 + 5y)dy = 0. Solucién. Poniendo M(x.y) = 72° + 30°y+4y yy — N(a.y) = 402 + @ + By. tenemos 3(1 = 32?) 38 \ —3r = 3rytyte” cty fe+y). Luego Fu) = 2 = hua om = 4B, y nuestro factor integrante es ue.y) = (ety). Sea entonces M(a.y) = (vtyPM(a,y) = 72° + 240°y + 3004y? + 16x2y8 + Ba7y! + 4aty + 1207? + 12048 +4y4 oy N(x.y) = (w+y)*N(w.y) = 42° + 1205y + 120ty? + day? + x! + 8a3y + 18277? + 16xy? + 5y!. De esta forma ule.y) = / M(x, y)dx = a8 Fda y + Gay? + dey? + 28y! + wly + day? + 6a7y8 + dry" + 9(y) , ia y) = 405+ 1225y + 12044? + 4r2y3 + a4 + Se8y + 1807y? + 16ry + g'(y), y comparando con N(x, y) obtenemos: A gy) = by Por lo tanto u(x.y) = x7 + 40°y + 625y? + 4aty? + x8y! + aty + 4a5y? + 6x?y? + dry ty, y entonces nuestra solucién general viene dada en forma implicita por Ia ecuacién al + da%y + Gay? + daty? + ahy! + 2ty + 4a9y? + 627y? + 4ry'+y? = c, CER. Ejercicio 2.10.14 Determine las condiciones bajo las cuales la ecuacién M(a.y)de + N(a.y)dy = 0 tiene factor integrante de la forma ja(e, y) = h(ay). Solucién. Si p(x, y) = h(y). tenemos Aln(u(a.y)) _ hi(xy) Aln(u(x.y)) _ hi(ay) = ot dx Tay)” Oy Try) y la ecuacién del factor integrante (2.28) queda de la forma MOY) Np. y) — 9M = My — Ney Tc) yN(e.y) -— aM(z.y)] = Ty (uy) — Z@y, Jo que implica H(ay) h{ay) yN-aM Luego nuestra condicién es (t,y) = = f(zy). we) = fry) ~ , yNoaM (x,y) = f(xy) (dependa sélo de xy). En este caso el factor integrante es w(z,y) =Rfry) donde h(u) = ef fe, Ejercicio 2.10.15 Use lo anterior para resolver la ecuacién y(a?y? + 2)dx + 2a(P—2?y?)dy = 0. Solucién. Poniendo M(a.y) = (vy? +2) yy — N(w.y) = 2x(1 274°), tenemos a, 7 fle). y Luego fu) == = hu =e IM™ =, y nuestro factor integrante es H(z,y) = (ay). Sea entonces xy) = (ry) *M(a,y) = at +2a8y? oy x.y) = (xy) SN (ay) = 20? y9 — 297". De esta forma u(x.y) = fit, y)de = In(x)— xy +. gly), au no Bye) = 2 2y Sh +a'y). y comparando con N(x, y) obtenemos Jy) = -2yt => oly) = -2Inly). Por lo tanto u(x.y) = In(x) — ay? — 2n(y), y entonces nuestra solucién general viene dada en forma implicita por la ecuacién In(x) — ay —2In(y) = c. CER. Ejercicio 2.10.16 a) Demuestre qué la Bcuncién M(x. 1)dr-+ N(x. y)dy = 0 bajo Ta condicién a - = f(v-+y2). tiene factor integrante de la forma ju(x, y) = h(a-+y?). b) Aplique para encontrar la solucién general de (3x + 2y + y)dx + (x + dary +,59?)dy = 0 Solucién, a) La ecuacién del factor integrante es yOln) _ y,2tn(u) _ aM _ aN on ey Oy Ox’ Sea 2 =x+4? y pongamos pu = pu(z). Entonces Aln(u) _ Aln(w) Az _ Alm) Or Oz Ow Aln(u) _ AIn(y) Dz _ Alun), Oy Oz Oy Oz ue Reemplazando en la ecuacidn del factor integrante obtenemos ain _ 0M _ ON dy Ox (N —2yM) o bien aln(u) Oz Luego la condicién para tener un factor integrante de la forma p(x. y) = h(x + y) es que b) Tenemos M(t.y) = 3c+2yt+y?. N(a.y) = v+4eyt Sy. Por Io tanto es decir din) _ 1 Oz z Jo que implica wz) = 2 => wey) =a2+y. Nuestra ecuacién multiplicada por el factor integrante es (3a? + Qary + dary? + 2y? eye + (x? + 407y + Gry? + dey? + 5y")dy Entonces u(x.y) = [022m -4a7+29-+ ae = 8 407y+207y? +20y? +2y!+9(y) « Jo que implica tu dy Comparanda con la ecuacién diferencial abtenemos (x.y) = 2? +40?y + Gry? + dey? + 9/(y). A 5 gy) = Sy => gy) =. Por lo tanto u(e.y) = vt aPyt 2074? + 420y? +2yt+y° y nuestra solucién general esta dada implicitamente por Ja ecuacidn wp aPy +207? + 420y3 + ay +y? = ceR. Ejercicio 2.10.17 Resolver la ecuacidn diferencial (7ety — 3y8} dx + (2x5 -9xy"}dy = 0, sabiendo que existe un factor integrante de la forma 2"y", Solucién. Nuestra ecuacién multiplicada por ay" es (Za™4yn + 1 = Bay) dx + (xy? — 9a™yP¥7) dy = 0 . Poniendo M(2,y) = 72™4yn+1—3xy"*8 y N(x.) = 20 5y — gamHtyr?, tenemos a ay = Tee Dey" — Bln Sry y an Fe = lm + Baty" —9(m + Vary" Por lo tanto la ecuacién sera exacta si n+l) = Am+5) 3(n+8) = m+), lo que implica m =2 yn=1. Luego M(x,y) = Tay? — 3274? y N(e.y) = 2x7y—92°y8. Integrando M(x, y) con respecta a x obtenemos u(r,y) = ay? —2%y? + h(y) ay (x.y) = 2x7y—923y® + h'(y) . y comparando con N(x, y). conclutmos que ny) =0 == hy) =0. Por lo tanto la solucién general en forma implicita es aypo-axy=c. ceR. Ejercicio 2.10.18 Encuentre la solucién general de la ecuacién de Bernoulli a + Fean(ayy = —asen(x)y. Solucién. Dividiendo por y7® obtenemos dy 1 ody 1 a PF + 5 tanlayy asen(x) y nuestro cambio de coordenadas es dz -ady = at -2y' or y por lo tanto dy __1dz 3% YY ae > Fade Reemplazando en nuestra ecuacién y multiplicando por —2 tenemos la ecuacién lineal dz z tan(x)z = 2rsen(r) . Primero resolvemos la ecuacién homogénea dz = tan(2)2. dx Separando variables obtenemos & = tan(a)dr, ¢ integrando In(z) =In(cos(z)) + In(c) =n c ;) : cos(x) y exponenciando zat ~ cos(x) Buscamos entonces una solucién de la ecuacién no homogénea de la forma ) ele) 20) = SG) Reemplazando en ella obtenemos la relacién “ =2asen(z) =+ (2) = 2xsen(2x) cos(x) = 2sen(22) . ¢ integrando por partes (a) = [senenae = oh con(2e)+g f cost2njtr = ~4oeos(2r) + 4sen(22) 40. Por Io tanto y luego Tarcos(2) + Isen(2x) +c Ejercicio 2.10.19 Resolver el problema de valor inicial y gl = 2y4 - y]. 40) = 1. Solucidén. Esta ecuacién es del tipo Bernoulli y la podemos escribir de la forma , 1 y+ gy x = 2ny!, y multiplicando por y~ nos queda 1 1 ry’ 4 ay? = =(1 = 2). yy + By 3! ) Ly! = 73 = 3,4 ty) = Sea wu = y7%. Por lo tanto wl! = —3y74y! y ym4y! = —dv. Reemplazando obtenemos 1, 1 1 , gu + gu = 30-2), 0 bién w—-u=2-1, u(0)=1 La solucién de la correspondiente ecuacién homogénea es un(t) = cet. Usando el método de variacién de pardémetros, buscamos solucién de la ecuacién no-homogénea de la forma Luego debemos tener de =U-1 = et) = (4 Yew. Asi c(t) = fee — Detdt = -(1 + Met +a, y por Io tanto u(t) = -(1 + 2) + ce. icial u(0) = 1 implica cy = 2, Obtenemos asf La condicién i u(t) = —(1 + 20) + 2e, y entonces y(t) = [-(1 + 20) + 2e]°. Ejercicio 2.10.20 Considere la ecuacidn diferencial y+ Ul=-2)y - YP = x(x-2). a) Encuentre solucién particular de la forma y = Ax + B. b) Encuentre la solucién general. c) Encuentre la solucién particular queypasa por el punto (2.2) y el intervalo maximo donde esta definida. Solucién. a) Poniendo y:(x) = Ax + B tenemos x(x) = A y reemplazando en Ja ecuacién A + (1-2) (Av +B) — (Av+ By? = x(x-2), es decir A+2B-B? + (24-2B-2AB)x + (-2A- A?) a? = -2n + 2°, que implica A=-l y B=1 = (vt) =l1-c. b) Como nuestra ecuacién es una ecuacién de Ricatti, hacemos la sustitucién 1 ysl-r+s ¥ z Tenemos dy 1 1 dz da. 2dr’ y reemplazando en Ia ecuacién cbtenemos Ldz 1 1) 1-1 - -r44)- fi-c44) = 2-2) 1-35 + 20-2) (1 r+t) ( at 5) u(x - 2), que se reduce a dz cs Por Io tanto 2a) = -% + Luego la solucién general de nuestra ecuacién es 1 ya) =l-a2+ = c) Para encontrar la solucién que pasa por el punto (2,2) debemos tener Luego la solucién particular buscada ‘es. y(n) = 1+ y como 2< i el intervalo maximo donde est4 defitidla es J-= Ejercicio 2.10.21 Para x > 0 considere la ecuacién de Riccati 2. 1 yo + typ - (lt da + 207)y = (lt 0420? +2°) a) Encuentre solucién particular de la forma y:(2) = e?* (Az + B), b) Encuentre su solucién general. Solucién. a) Si y(x) = &" (Ar +B). tenemos y(t) = @(A+2B + 2Az]. ey? = F*(B? + 2ABr + Aa’). La tart 20°) (2) = (1+ 4x + 2x?) (Ax + B) [B + (A4+4B)x + (4A42B)x? + 2Ax']. Luego reemplazando obtenemos er er —— (1l+04207403) = - 7 r [B + 2Bax + (244+2B-2AB) 2? + (2A-A?) 2%, y por Io tanto B=1, A=1 De esta forma la solucién particular es w(x) = e (a+1). b) Como es una ecuacién de Riccati hacemos el cambio de coordenadas y=nte y=" zt Entonces J a) = yf v'(a) y'@) = (2) - mee” ) 2 yg)? = e2Iy(ay? 4 2 AO 1 eT y(x) ey (a)? + ola) + ee ~ 2a tar 2) (0) = HE at de + 22°) fle) + wor y reemplazando en la ecuacién obtenemos Nr) tip) — W@ ope 2 n(x) w(x) mee +e lay + 2a) + 1 1 1 — = (144i + 22?) —] = f(x), vay = (+ 4a + 20°) n(x) + aa! = 10 donde f(z) = —— (1+a+422?+2°), Rie (a) + Pyle)? — = (1+ de + 22?) a(x) — m2r po yl) + erg v(a}? o(a) - Fear par) = f(x). Como y:(z) es solucidn, la relacién anterior se reduce a m2n ty Wil2) 1 wee + Tay t TR 1 2 1 = - z(t ae + 20") =0. Multiplicando por v(x)? se obtiene oa) =e Qy(a)o(e) + 1 = 2a bate + 20%) o(z) 2e7* n(x) — da pan+ 20%) u(x) + = hi +20 u(x) + e7?* Luego tenemos la ecuacién lineal (0) = ~E [1+ 2rhu(a) +e, cuya solucién es v(x) = ef aadae [/ ef aayir 2 dy 4 el , donde a(x) = -4[1 +22}. Como [aac = -/ 2 [i +2r\dr = =(In(x) + 20), = lie fe = c6 zy tes Tenemos Luego la solucién general de la ecuacién original es 2ne* we) = e+) + A, CeR Capitulo 3 Aplicaciones de Ecuaciones Diferenciales de Primer Orden 3.1 Familias de Curvas y Trayectorias Ortogona- les Hemos visto que normalmente la solucién general de una ecuacién diferencial de primer orden es una familia de funciones. que contiene una constante arbitraria, lamada pardmetro. Si la ecuacién diferencial verifica nuestro Teorema de Existen- cia y Unicidad (Teorema 2.4.1), por cada punta del dominio de definicién A de la ecuacién diferencial pasa una tinica curva solucién (maxima). Por lo tanto, las curvas de nuestra familia cubren nuestro dominio A y son disjuntas entre sf. Llamaremos en general, familia a 1-pardmetro de curvas sobre un conjunto A del plano, a cualquier familia, que dependa de un pardmetro, de curvas que son disjuntas entre ef y que cubren A, Si la familia a 1-parametro de curvas viene dada implicitamente por la ecuacién f(v.y.c) = (3.1) en la mayorfa de los casos podemos encontrar una ecuacién diferencial cuya solucién general esté dada por (3.1). Para lograr esto, primero derivamos implicitamente 3.1 respecto de x, obteniendo una relacién del tipo ay ee IO Luego, eliminamos el parametro c (si es posible) usando las ecuaciones 3.1 y 3.2. Iegando a una ecuacién diferencial de primer orden: (anit) <0, (3.2) da Ejemplo 3.1.1 Considere la familia de cfrculos tangentes al eje OY en el origen P+y =r, cER, y encontremos la ecuacién diferencial asociada. Derivando con respecto a a obtenemos Pero y reemplazando en la ecuacién diferencial tenemos dy _ v+y 2a + 2y— = =. i ae x o bién dy dx Como aplicacién considé¥emos el problema de hallar trayectorias ortogonales. Diremos que una familia de curvas es una familia de trayectorias ortogonales de otra familia de curvas. si toda curva de una de las familias es ortogonal (es decir, perpendicular) a todas las de la otra familia. Por ejemplo, la familia de trayectorias ortogonales de la familia de cfrculos centrados en el origen x? + y? =. ¢ € R, es Ia familias de rectas por el origen y = crc € R. y Figura 11 Las correspondientes ecuaciones diferenciales son y yey ae diy dv a" respectivamente, y la ortogonalidad de sus curvas solucién se refleja en que x =-l. yr De hecho, la familia de trayectorias ortogonales de una familia de curvas que es la solucién general de la ecuacién y/ = f(x-y), esté dada por la solucién general de la ecuacién dy 1 a ~ Fey Ejemplo 3.1.2 Encontremos la familia de trayectorias ortogonales de la familia de cfrculos tangentes al eje OY en el origen x? + y? = 2ex. c € R, del ejemplo anterior. La ecuacién diferencial asociada es dy dx Day (33) y luego debemos encontrar la solucién general de la ecuacién dy __2ay eee (3.4) La forma més sencilla de resolver esta ecuacién es intercambiando los roles de las variables x e y poniendo > y qy tay Observe que esta ecuacién es la misma ecuacién 3.3 con xe y intercambiados. Luego la solucién general es la solucién general de 3.3 con x ¢ y intercambiados, es decir P+y=2y.ceER, que es la familia de circulos tangentes al gie OX en el origen. @ © Figura 12 Ctra forma de resolver 3.4 es escribiéndola de la forma 2Qayde + (y? —2")dy =0. y observando que tiene factor integrante que depende sdlo de variable y. En efecto, si M(r.y)=2ty yy) May) =y-2?. at (OM _ ON) ay = -2 M\dy Oe) > Wy 2d _ 1, P Multiplicando por el factor integrante obtenemos la ecuacién exacta entonces y el factor integrante es (x.y) = elt r a aide + (1-5) ay=0: Entonces © 2 \ u(x,y) = ote + (0) = 7 ty) y we. Ou — ey. Baye) = Btw: Por lo tanto gy) =1 = gy) = 2 ul@.y) = —+y.- (x,y) vty Luego la solucién general es , x _— = 2c. 7 +y 1c 0 lo que es Jo mismo. multiplicando por y x+y = Www.cER. 3.2 Reacciones qufmicas de primer orden y desin- tegracién En general no es dificil observar en la naturaleza diversas reacciones quimicas entre elementos. Por ejemplo, si tna moléculas de cierto tipo, por la accién del medio, tienen tendencia a desintegrarse en moléculas més pequefias a un ritmo que no se ve afectado por la presencia de otras sustancias, es natural pensar que el ntimero de moléculas que se descomponen en una unidad de tiempo sea proporcional al ntimero total presente (reaccién quimica de primer orden). Supongamos que en = 0 se tienen sr gramos, Si denotamos por (cl mfimero de gramos presentes en el instante £ (luego x(0) = xo). tendremos que { es el ritmo de crecimiento de a: y —4 es el ritmo de decrecimiento, De esta forma si k > 0 es Ja constante de proporcicnalidad, tenemos la ecuacién dx —G ake Integrando se obtiene In(a) = Ina) = -kt = a(t) = agen, Se Hama semi vida T al tiempo requerido para que la sustancia se reduzca a la mitad. De esta forma InQ) = -hQQ)=-kT = T=: 2 1 ! Fue = sae? Por Io tanto, si se conocen k 0 T experimentalmente, por esta relacién se conoce la otra cantidad. Ejemplo 3.2.1 Desintegracién radioactiva. EI radio carbono tiene semi vida de més 0 menos 5.600 afios. Este se produce en la alta atmésfera por la accién de rayos césmicos sobre el nitrégeno. El radio carbono por oxidacién pasa a diéxido de carbono y este se’mezcla (por el viento) con el diéxido de carbono no radiactivo ya presente, La proporcién en el carbono ordinario ha alcanzado hace tiempo un estado de equi- librio. Todas las plantas y los animales que comen plantas, incorporan esta proporcidn de radio carbono en sus tejidos, Mientras el animal o la planta viven. esta proporcién permanece constante, Pero al morir deja de absorber radio carbono y el que habia en el momento de morir sigue desintegrandose. Asf si un fragmento de madera antigua tiene la mitad de radioactividad que un Arbol vivo, éste vivid hace unos 5.600 afios (= T). Si solo tiene la cuarta parte, determinemos el tiempo 7 en que vivid. Tenemos entonces 2 = net => ; =e =s ~In(4) = -hi. y luego In(2), > - . 2In(2) = tf == 7? = YF = 11.200 (afios aproximadamente) . Esto proporciona un método para poner fecha a cualquier objeto antiguo de origen orgénico: madera. carbén, fibra vegetal, huesos, cuernos o piel. Ejemplo 3.2.2 Si la vida media de una sustancia reactiva es de 32 dias, Deter- minemos el tiempo é en que 24 Kilos se convierten en 3 Kilos. Tenemos nQ) in(2) = 4 =p a BQ = BY) a0) = 4 y BaT= T= ka Ts Entonces debemos tener \ 1 > B= af) = Mew => g=eN => Kt = In), Jo que implica 1 7 = SRO ~ 5.32 = 06 das. Ejercicios 3.2.3 Resuelva L.- Siel 25 % de una sustancia radiactiva se desintegra en 100 afios. ;, Cual es la vida media ? 2. En un proceso con unaysustancia radiactiva se hacen dos mediciones. La primera, dos horas después de iniciado el proceso arroja la cantidad de 100 mer.; la segunda. una hora después, indica la presencia de 8 mgr. ;, Cual es la cantidad original de sustancia radiactiva ? Ss Usando carbono 14 (C4) cuya vida media ¢ un fésil humano que contiene 25,2 mer. de C’ ser humano vivo es 53.8 mer. 168 aftos, determine la edad de . sila cantidad presente en un Ejemplo 3.2.4 Crecimiento de bacterias. Sea N(f) la cantidad de bacterias presentes en el instante ¢. Entances dN we T= Nacimientos - Muertes = a(i)N — b(0).N z donde a(1) (respectivamente, b(1)) es la proporcién de nacimientos (resp.. muertes) con respecto a la cantidad de bacterias presentes en el tiempo ¢. Entonces, tenemos Ja ecuacién XO 7 1(0)efias) ts) WO 7 WO ~ HO, = NO = N(O)efe(a(s)-8))as | ne Por ejemplo si a(t) =a y b(t) =b. tenemos N(t) = N(0)e¢~?! Ejemplo 3.2.5 Suponga que conoce N(0) y M(t). {, Cuanto tiempo 7 debe tran- scurtir para tener NV bacterias ? Llamemos Nj = N(0) y Ni = N (ty). Entonces N() = Nee, y M, = Nt) = Noe@O8 => Por lo tanto N = N(@) = Nee as Ejemplo 3.2.6 Una superficie electrizada se descarga con una velocidad propor- cional a la carga. Hallar la carga en funcidn del tiempo. Si designamos por C(t) la carga presente en el instante ¢, nuestra ecuacién es nue- vamente dC a -kC. Por lo tanto C()= Coe. Ejemplo 3.2.7 Ley de enfriamiento de Newton. La velocidad con que se enfrfa una sustancia en el aire es proporcional a la diferencia de la temperatura de la sustancia y la del aire. Si designamos por T,(t) ¥ Tn respectivamente, la temperatura de la sustancia en el instante ¢ y la temperatura (que suponemos constante) del medio (aire) en que se encuentra la sustancia, nuestra ecuacién diferencial es dT. a = -h(Ts(Q - Tm) + Separando variables r dT oe = The, TO-T, e integrando entre 0 y ¢ a Ti) =Tm) _ (rapt) =~ obtenemos la solucién T(t) = Tn + (Ts(0) — Tne. Por ejemplo, si la temperatura del aire es de 20° y la sustancia se enfrfa de 100? a 60° en 30 minutos, calculemos en que instante la temperatura de la sustancia serd de 40°, Tenemos T.(0) = 100, Tm = 20, 7,(30) = 60. Luego T.(t) = 20 + 80e™. Evaluando en ¢ = 30 obtenemos 60 = 20 + 80c~™, Jo que implica Reeth =» -mQ) =e =» e = WO. Sea f el tiempo buscado. Entonces 40 = 20 + 80c~"? = t= s Jo que implica 60 minutos . 3.3. Procesos quimicos simples Suponemos que A y B son compuest0s quimicos que reaccionan entre ellos de acuerdo a las ecuaciones A= B Asumimos A(0) = Ap > 0, B(O) = Oy A(t) + B(t) = Ap para todo t. Entonces B(é) = Ap — A(E) y nuestra primera ecuacién diferencial se transforma en la ecuacién lineal de primer orden A+ (ky + ky)A = beg. De esta forma t A(t) = €~ filha thadas [4: + fetes Aad 0 J = evthita)t [4 4 has enti tta)e - Othe hth \ ~ = 1 mI kag hy +h? = a 1 = eth Observe que si ponemos f= man = fe 9 m= pon = obtenemos AQ) = Bee“™r¥¥2# 4 Ae y BW) = Bell -— HH) , Observe finalmente que los valores de ky y kz. que en general se obtienen experi- mentalmente, verifican 3.4 Circuitos eléctricos simples Sabemos que las leyes de Newton nos permiten establecer relaciones entre las fuerzas que afectan a un sistema mecénico. De manera similar , las leyes de Kirchhoff (1824-1887) nos permiten establecer relaciones entre los elemnetos que praveen y usan energia en un circuito eléctrico. Un circuito eléctrico es un dispositivo que permite la circulacién de un campo eléctrico. Un campo eléctrico es el campo de fuerza asociado a un carga eléctrica. Finalmente, la carga eléctrica es uno de los constituyentes basicos de la materia y se reconocen dos tipos que son arbitrariamente designados como positiva y negativa. EI principio general es que cargan iguales se atraen y cargas opuestas se repelen, La velocidad de la carga eléctrica se denomina corriente. Esto es: si q es la carga, entonces la corriente es J = 4 En un circuita eléctrico se reconocen elementos activos y pasivos. Elementos activos son baterfas. pilas, motores eléctricos. Elementos pasivos son resistores, inductores y capacitores, Por ejemplo, consideremos un circuito R L C en serie como en la Figura 13. Figura 13 Consta de tres ramas: una resistencia R, un autoinductor Ly un capacitor C. Una rama se puede considerar como tm mecanismo eléctrico con dos terminales. Por ejemplo la rama C tiene terminales f y 7. Estos terminales se conectan entre si para formar los nodos @. 3.7. Por cada rama circula una corriente cuya intensidad se mide por un niimero real, digamos, ip. iz,ic. donde por ejemplo, iz es la intensidad a través del resistor. Las flechas del diagrama que-Orientan las ramas indican el sentido en que fluye la cortiente. Si ip > 0, entonces fluye de faa. Ley de Kirchhoff para las intensidades: La suma de las intensidades de corri- ente que van hacia un nodo es igual a la suma de las que se alejan. Esto también puede enunciarse como Ta cantidad neta de corriente a través de cada nodo es cero. En nuestro caso in = in, in = -ic. E] estado del circuito esta caracterizado por la intensidad i = (iz. iz,éc) junto con Ja tensin (voltaje), 0 bien, las caidas de tensién (cafdas de voltaje) en cada rama. Sean estas vp, tz, te» Para medir la tensién se coloca un voltimetro en cada uno de los nodos a, (3,77 que marca u(a), 0(3),0(y). Entonces tr = v(8) — v(a), vr = v(a) — v7). te = v(8) — vy). Ley de Kirchhoff para las tensiones: vz + vz — vc = 0 (en nuestro circuito, Figura 13). Este es un caso particular de la Ley de Kirchhoff para tensiones que dice que la catda neta de voltaje en un circuito cerrado es cero. Si se considera el circuita Figura 14 se tiene ip =in=ic y ptt ttc =0. Ley de Ohm: La cajda de voltaje vp a través de un resistor es proporcional a la corriente J que pasa por el resistor. tn = RI. Leyes de Faraday y Lenz: La cafda de voltaje a través de un inductor es propor cional a Ja raz6n de cambio instantdned de la corriente, dl w= La : Si consideramos el circuito B R Cc a L 7 Figura 15 donde E = E(t) es una fuerza electromotriz que proporciona un voltaje (0 energia potencial) al circuito en el instante t. Ley de conservacién de voltajes de Kirchhoff: v7 + wp = E(t). Obtenemos asf la ecuacién lineal de primer orden RI + iw = E(t) En el caso en que E(t) = Ey es constante. dl dl _ Lat Rl =o = ee 7 cuya solucién es = eft Fo (Bt I(t) = e# [100 + 3 (f= )j . Observaciones 3.4.1 Imponer la condicién J(0) = 0 quiere decir que en el ins« tante inicial no circula corriente por el circuito. En este caso antes de iniciar el circuito nuestra situacién puede ser como la mostrada en la Figura 16. “tl O E Figura 16 Una condicidn del tipo 1(0) > 0 se obtiene cuando existe otra fuerza clectromotriz, constante Eq que se aplica por un tiempo apropiado para obtener una corriente estacionaria (0) (Figura 17). En el instante ¢ = 0 el interruptor pasa del punto A al punto B cerrando un circuito propulsado por una fuerza electromotriz E (Figura 18). “hi “thi be OF kk S| Ex Or A Figura 17 Figura 18 En este caso la ecuacién es Ri _E 1 Rh, _E y= V+ Fl=F. 10) = b>. Otra situacién se presenta cuando no hay fuerza electromotriz en un circuito R- L, Antes de activarse el circuito podemos tener la situacién de la Figura 19. En el instante ¢ = 0 el interruptor pasa del punto A al punto B, cerréndose un circuito que no tiene fuerza electromotriz (Figura 20). a it &L &L O L 3 A Figura 19 Figura 20 La ecuacién es 1 8k ) M+ FT=0. 10) = h>0, cuya solucién es I(t Observe que el citcuito se descarga una-ver. que la fuente de voltaje a sido suprimida. = I(0)e"#. Frecuentemente E(t) es de la forma E(t) = Bycos(wt) (fuerza electromotriz alterna) Se tiene la ecuacién R t+ zi = —cos(wt) . y suponiendo (0) = 0, se tiene la solucién I(t) = Bem fe cos(wu)du , 0 bien ~ BEF Te © 1) = (Festa + sxen(ot)) FoR tr 3.5 Problemas de mezclas Para la obtencién de un remedio. de wna pintura. de un trago preparado, en escala we, 8 Hecesario mezclar diversos ingredientes los cuales hacen parte de una solucién perfectamemnte homogeneizada (esto es: no importa la muestra, la distribucién de ingredientes es siempre la misma). Por ejemplo. consideremos un recipiente de V litros de capacidad que contiene una solucién perfectamente homogeneizada (por ejemplo: agua y sal) como en la Figura 21. 9° A Figura 21 Se accionan simulténeamente las laves A y B. haciendo ingresar por A agua pura a razon de a lts/min y se extrae solucién por B en la misma proporcién. Sea (1) la cantidad de sal presente en un instante 1 posterior, Entonces “0 es la-cantidad de sal por litro en el recipiente, y la variacidn de la cantidad de sal es 2 x(t) # = -a>- De esta forma x(t) = x(0) eF, Si en lugar de agua pura entra una solucién que contiene c gramos de sal por litro. entonces es decir de. a ——— = -sdi — at) = Vic $+ (2(0) — Vecjen*, poe cP! 2) + (#(0) ) Las miiltiples posibilidades que se presentan en los problemas de mezclas se reducen ala ecuacién é =e(t) — st). donde e(¢) y s(t). son respectivamente. la cantidad de sal que se aiiade y se retira en el instante de ¢. Ejemplo 3.5.1 Considere el mismo recipiente de la Figura 21 y suponga que de nuevo por la Have A entra agua pura a razén de a Its/min; pero que por la Ilave B sale solucién a razén de 6 Its/min, con b > a. Tenemos entonces « e(t) = 0 (no hay entrada de sal). V — (b-a)t: es la cantidad de liquido presente en el instante ¢. Luego y nuestra ecuacién es Separando variables obtenemos dee b T7 Ve boa Ejemplo 3.5.2 Suponga la misma sittacién anterior, pero entrando por A en lugar de agua pura, solucién con concentracién de ¢ gramos por litro Tenemos i) = BS g ts e) = open tas, ts = popsoi lm’ y muestra ecuacidn es de ad °""V—@-ai Esta es la ecuacién lineal de primer orden de b a+ V>@-ai" = ca, cuya solucién es a(t) = V ( - Ejemplo 3.5.3 Consideremos ahora dos tanques como en la Figura 22 A Yu Vo B ° 6 C Figura 22 Al primero de Vj litros de capacidad entra agua pura a través de la Have A a razon de b Its/min. Por la Mave B, también a razén de 6 Its/min sale solucién del primer tanque y entra en el segundo. Finalmente del segundo tanque, por la Have C sale solucién a razén de b Its/min. Sea x1 (¢),:r2(t) la cantidad de sal en el primer y segunda tanque, respectivamente, en el instante ¢. Tenemos entonces, en el primer tanque, raz6n de entrada e1(t) = 0 y raz6n de salida wo 1 , _ mild) grs , Its si(t) = VT Winn’ Por otra parte, en el segundo tanque. la razon de entrada e2(é) es igual a la razén de salida del primer tanque s1(é). Luego z(t) grs , Its el) = ae nim’ y la razén de salida es ok. ap(t) gts, Its y= BOs, Is, 80 = eon Tenemos asf el sistema de ecuaciones diferenciales a = by Consideremos ahora b = 2 It/min. Vj = 1 It, Vo = 2 It y las condiciones iniciales (0) = 5 gr y #2(0) = 6 gr y tratemos de determinar cuanto debe funcionar el sistema para que del segundo tanque empiece a salir solucién con concentracién por debajo de 1 gr/It. Con estos valores tenemos el sistema a = -2a ty = 2a, — 2% Resolviendo la primera ecuacién obtenemos ry(t) = 5e7*, y reemplazando en Ia segunda ecuacidn tenemos la ecuacién lineal tp + a = Ie, Por lo tanto, usando la Férmula de Leibniz (2.34). t a(t) = (/ e066) = 2(t) = Wert — 10c, 0 Debemos encontrar F tal que ne =1 esdecir 16e% — 10e7% = 2. V2 Poniendo u = 7 y dividiendo por 2 tenemos la ecuacién 5u?-Su+1 = 0, cuyas soluciones son eV 4 1368. 5 4+Vil 5 uw = ~ 146 yw = Como para t positivo u * <1, tenemos que weet => TX1,989. Bjemplo 3.5.4 Queremos inyectar un medicamento en un érgano humano. Supon- gamos que el volumen de circulacién sanguinea del drgano es 150 cm? y que se inyectan 1 cm3/min de agua destilada con 0.3 mgt/cm? de concentracién de medica- mento. La sangre entra al 6rgano a la misma razén que sale. Si en el instante inicial no hay presencia del medicamento j, En qué momento la concentracién del medicamento en el érgano sera de 0.05 mgr/cm®? Si designamos por 2x(¢) la cantidad de medicamento presente en el érgano en el instante ¢, tenemos x(0) = 0 y nuestra ecuacidn es x b=031- 2.1, P= 03 150 Tenemos entonces la ecuacién lineal cr += 03 cuya solucién es Queremos encontrar 7 tal que Entonces a(f) = z = 75 =} 45 - ase7thl = 75 37.5 5 = eth = Ejemplo 3.5.5 Una solucién de acido nitrico fluye a razdn constante de 6 Its/min. hacia el interior de un gran tanque que inicialmente contiene 200 litros de una solucién del mismo Acido al 0.5%. La solucién contenida en el tanque se mantiene uniformemente distribuida y sale del tanque a razén de 8 It/min. Si la solucién que entra al tanque es del*20-% de Acido nftrico, determine la cantidad de este Acido presente en el tanque al cabo de ¢ minutos. ;, En qué momento el % de Acido contenido en el tanque sera de 10 % ? Sea r(2) la cantidad de Acido nftrico presente en el instante 1. Ingresan: 6 It/min. al 20% , lo cual’significa: 1.2 It/min. Salen: 2 It/min. Inego de ¢ minutos. Luego la ecuacién diferencial es dx dr a ~ 1? > on7 8: m bi o bien ts 4 at im-i" = La ecuacién ¢s lineal y su solucién general es a(t) = et fe + [vet tar = Ainaoo-g [e+ 1a [ereaoeoa 12. (100 44 [- 412 [rao - ovat (100 - #)! [- + ooo - 0] (100 — #)4 + 0.4(100 — 1). En ¢=0 hay 200 It. al 0.5% . Por lo tanto (0) = 1lt. Ast 1=c-100'+04-100 => c¢ = —39-10074. Entonces 4 t \ — paoo—1)—39(1-L) . a(t) = 0.4(100 = #) — 39 ( im) Ahora, si Z es el instante en que en el estanque hay 10% de Acido, debemos tener x(t) 100-9 = 1° Jo que implica es decir lo que implica 1004 195 (100-7)? = = 19.9573 min. 3.6 Problemas resueltos Ejercicio 3.6.1 Dada una curva y gente entre el punto P = (x.y(z)) y = y(x). sea L(x) la longitud de la recta tan- su punto de inteseccién T con el eje OX. y P T 8 Figura 23 a) Demuestre que — ya) Ee Lx) = TVD b) Sia es una constante no nula, encuentre la ecuacién diferencial de la familia de curvas que verifican Ly(0) = a(y(a))?. c) Demuestre que la familia de trayectorias ortogonales a Ia familia de curvas del item b) esta dada por y(a) = cosh(ar +0). beER. Solucién, a) Tenemos en el tridngulo TSP PS Lr = PT y son) = BRE TE Ademas , y! = tan(9) = sen(9)-sec(9) = 4. VIF UP Lr Jo que implica Lr = Gv b) Tenemos ay? = 3 THY = “ayy! = VIFUF = YY? = 14+. Luego (y-)yP =1 = VeP=Ty =1, y obtenemos la ecuacién diferencial 1 y= 7 eye = c) La ecuacién diferencial para las correspondientes trayectoria ortogonales es y = -V@Pat- Separando variables obtenemos dy Ver—1 = -dx, ¢ integrando 41m (y+ V@=T) = wont. Por Io tanto In (ay + PHT) = -(av+b) => ayt M@PHT = oe, Esto implica ath 4 g=(arst) y ast ylx) = cosh(an +b). DER. Fjercicio 3.6.2 Un profesor redacta las notas del curso con una rapidez propor cional al ntimero de hojas ya escritas. Por otra parte sus alumnos son capaces de leer los apuntes con una velocidad constante, Al comenzar el curso, el profesor entrega 10 hojas a sus alumnos y postericrmente se las va proporcionando a medida que las escribe. Determine el atraso de uno de sus alumnos en la lectura de las notas al finalizar el 3° trimestre si al cabo del primero Hevaba un atraso de 20 paginas y al término de 6 meses un atraso de 70 paginas. Considere cada trimestre de tres meses sin receso entre cada uno de ellos. Solucién. Usemos las siguientes variables: t : tiempo medido en meses. M(t) : ntimero de hojas escrita al cabo de ¢ meses. L(t) ; ntimero de hojas lefdas al cabo de ¢ meses. Tenemos entonces los siguientes datos H(0)=10, L(0)=0. HQ)=L0)+20. H(2)=L(Q)+70. y las ecuaciones diferenciales dH dL a TNH ga? Sea H el ntimero de hojas (notas) ya escritas, Tenemos entonces dH _ _ oe Oe La condicién inicial H(0) = 10 implica c = 10, y por lo tanto H(t) = 10e". Por otra parte. si L es la variable que indica la lectura de los apuntes, entonces dL Wu? = Marvite La correspondiente condicién inicial L0)=0 = @=0 = L(t) = pt. Ademis las relaciones H(3) = L + 20, H(6) = L + 70, implican el sistema 10% = 3p + 20 le = Gp + 70. Restando Ia segunda ecuacién con dos veces la primera y poniendo x =e, se obtione la ecuacién cuadratica 102? — 20% = 30 cuya solucién positiva es z = @ = 3, De esta forma In 10 gS no) yoay Ast 10 H(t) = 0c?) La) = zh y el ntimero de pdginas de atraso al cabo de 9 meses es (9) — L(9) = 102) — 30 = 270 — 30 = 240. Ejercicio 3.6.3 Un modelo matemético para describir la poblacién humana es a'(t) = ax(t) — bx?(t) donde a = 0,029 y b = 2.695- 107". ;Cudntos habitantes Hegara a tener la Tierra segtin este modelo? Justifique sus afirmaciones. Solucién. La ecuacién es de variables separables y se puede escribir de la forma dx ax = ba? o bien Luego integrando obtenemos x at+c=In : a b de donde bx. t = cet d= $e, aoe OO SO = ipa La cantidad de habitantes que Iegard a tener la Tierra se obtiene calculando lim 2(t) = © = 1076-10". fim, 7 Ejercicio 3.6.4 Un esquiador acuético ubicado en el punto (a, 0) es tirado por un bote localizado en el origen y que viaja hacia arriba a lo largo del eje OY. Hallar la trayectoria del esquiador si éste se ditige en todo momento al bote, Solucién. Supongamos que en el instante t > 0 el bote esta en el punto (0.5) y que el esquiador esta en el punto (xr. y). Figura 24 Debemos tener entonces P+(y-bP =@ = |y-bl= VP-P = b-y=Ve-#, ya que b> y. Como la curva y = y(z) es decreciente tenemos también dy _ bry | Ve@=e dx x r Luego como y(a) = 0, integrando abtenemos: [ Vee VES E h Para calcular esta integral ponemos u = acos(t), que implica du = —asen(t) y Vat = = asen(t). Asi ) ) ylo) = [ nem) CeO =a [ met) ct) — cos(t) a arccos(#) = alln(sec(t) + tan(d)) - sen(0)] /, - «fo (EF) 3] (HE) aln —)- @— a. Ejercicio 3.6.5 Considere un tanque que contiene 1.000 litros de agua, dentro del cual una solucién salada de salmuera empieza a fluir a una velocidad constante de 6 litros por minuto. La solucién dentro del tanque se mantiene bien agitada y fluye hacia el exterior deftanque a uma velocidad de 5 litros por minuto, Si la concentracién de sal en Ja salmueta que entra al tanque es de 1 kildgramo por litro. determine cuando sera de 63/64 kilégramo por litro la concentracién de sal en el tanque. Solucién. Sea x(t) la cantidad de sal que hay en el tanque en el instante f, Entonces Ia velocidad de entrada de sal al tanque en el instante ¢ es I Kg — 1. min’ Tt También en el instante Z. la cantidad de quido en el tanque es de e(t) = 6 V(t) = 1.000 + (6—5)E It. Ja concentracién es a(t) Ke 1.000% It” y la velocidad de salida de sal es It a) Ke y= 5 AB 9 = 5 am Tooort Tt” Luego nuestra ecuaci6n diferencial es a a ad 1000+ 1° Para resolverla, consideramos primero la ecuacién homogénea de _ be a ~ ~ T000+t" que se puede escribir de 5 wi _5_ iy, @ To00+¢ “ La solucién de la homogénea es Cc oo = App HE Haciendo variar la constante c = c(t) y reemplazando en la no homogénea obte- nemos e@) _ 5 = 6 ami 8 = e(x) = 6(1000+4)5 = elt) = (1000+48)% +e. Por lo tanto c y= ———_.. a) = 1000-44 aE Como (0) = 0, tenemos ¢ = —1000°, y entonces nuestra solucién es 10008 y= _ -. a) = 1000-+t— Tang HE Asi, la concentracién del sal en el estanque en el instante ¢ es 1000 + 1 = Ati 10008 1000-41 ~ (000+ HE” Tenemos que encontrar ¢ tal que _ 100% _ 63 (1000+ H)6 ~ G4 Entonces 1 1000¢ ui = Toots = > (1000+¢° = 64-1000° =+ 1000+¢ = 2000. y por lo tanto ¢ = 1000 min. Ejercicio 3.6.6 El eje OY y la recta x = c son las orillas de un rfo cuya corriente fluye a velocidad uniforme a en Ia direccién de y negativa, Una barca entra al rfo por el punto (c. 0) y se ditige hacia el origen con velocidad 6 relativa al agua. {Qué trayectoria seguiré la barca? Determine condiciones para a y b que permitan a la barca alcanzar la otra orilla, ;En qué punto tocara tierra? Figura 25 Solucién. Las componentes de la velocidad de la barca son de, dy TF = ~beos() FA = -a+bsen(0). Jo que implica dy _ -a+lsen(o) _ ~* +E de ~~ bea) = ay/x? + y? + by ba: , que se puede escribir de la forma dy _ 4 y dx b x Haciendo el cambio de variables z = 4, obtenemos: dz _ lady LW Yoh ype sy.) - 1s, dx x dt x es decir Ja ecuacién _al di ~ ba Separando variables tenemos integrando In(z + viF#) = a) + n(C) y exponenciando 2+ VIF2 = C23. Despejando 2 obtenemos 1 1 ] =tlo gt - by, v=} fost - Lea] Jo que implica 1 1 == #- sat), we) =} [oat - Let] Imponiendo la condicién inicial y(c) = 0. obtenemos 1 Ch =act = C=ct, ule) = § (ea _ ey" Observemos que la barca Iegaré a la otra orilla del rio solo si y(x) esta definido en x = 0. Para que esto ocurra debemos tener 1— ¢ > 0, es decir b > a. Para b > a tenemos y(0) = 0; y luego la barca Hega a la otra orilla en el punto (0.0). Pero si b =a, tenemos = 5-1 y por lo tanto la barea llega al otro lado en el punto (0. — al y por Io tanto Ejercicio 3.6.7 Una fabrica de papel esta situada cerca de un rfo con un fluido constante de 1000 m°/seg, el cual va a dar a la tinica entrada de un lago de volumen 10° m®, Suponga que en el instante ¢ = 0, la fabrica de papel comienza a bombear contaminantes en el rfo a razén de 1 m®/seg y que la entrada y salida de agua del lago son constantes ¢ iguales. {Cual sera la concentracién de contaminantes en el Jago en cualquier tiempo £? Solucién. Tenemos V = 10°, velocidad de entrada y salida de agua del lago es a= b= 1001, y la concentracién de contaminantes en el agua que entra al lago es = 1/1001. Luego nuestra ecuacién diferencial es 1001 = toor ~ ig 10!: o bién 4p, WOLe 10° La solucién de la ecuacién homogénea es 1 mt) = cet Usandb el método de variacién de parémetros. buscamos una solucién de Ia ecuacién no-homogénea de Ia forma x(t) = c(t) er" Entonces debemos tener 0° dies. = dace S&S w= McBee. 1001 y luego 9 10 ay - — ie 0 = iq tee™ Pero 9 9 10 10 0=20) =a +e = c= -TpD y por lo tanto 108 _ OT (1 - ee De esta forma la concentracién de contaminantes en el lago en el tiempo t es We a eH), Ejercicio 3.6.8 Se ha determinado experimentalmente que la variacién de peso de un tipo de pez varia segtin la ley dp 2/3 = ap’? - Bp, a = oP P donde p = p(t) representa el peso del pez y a, 6 son constantes positivas que caracterizan la especie, {Para qué valor del tiempo ¢ le parece razonable autorizar Ja captura de peces de esta especie? Solucién. Como la ecuacién diferencial dp 2/3 ® gn = ap qt oP = oP es del tipo Bernoulli, hacemos el cambio de variables = 1+ queimplcn = dpa. u = ph que implica T= 37747 Sustituyendo obtenemos la ecuacién lineal du 3a + Bu =a. La solucion es . ut) = le [c + gf ea : es decir Por lo tanto of = [8 (Peet), Como esta funcién es creciente, el mayor peso es a . a Poo = jim ptt) = (3) : y un tiempo razonable para autorizar la captura ser, por ejemplo. aquél para el cual p(t) > J Pox: esto es Ejercicio 3.6.9 En el interior de una casa, y en un cierto instante, el termémetro marca 70° F. El termémetro se traslada al exterior de la casa, donde la temperatura del aire es de 10° F. Tres minutos después el termémetro marca 25° F. Determine la ecuacién que permite conocer la temperatura del termémetro en el exterior de la casa en cualquier instante t. Solucién. Segtin la Ley de enfriamiento de Newton, la ecuacién diferencial es Wa ay T= WAT - 10), y tenemos los datos T(0) = 70. T(3) = 25. Separando variables obtenemos aT T-10 La condicién = -kdt = WT -10)=-kt+e = T(t) = 10+ ce™. T(0) =70 = c=60 = T(t) = 10 + 606. La otra condicién 73) = 25 =} 2 = 10+60e°% = e*= a T(t) = 10 + 60 [vi . Por lo tanto Ejercicio 3.6.10 Una barra de largo L, seccién transversal A y densidad (masa por unidad de volumen) ¢¢ sesumerge en un Ifquido de densidad p. Recuerde que segiin el principio de Arquimides: ef Uquido ejerce sobre el cuerpo que se sumerje una fuerza opuesta que es igual al peso del fluido desplazado por el objeto. Si x denota la parte de la barra sumergida, considerando una velocidad inicial wo. a) Obtenga la ecuacidn diferencial que describe al movimiento. b) Hasta que profundidad desciende la barra? c) Site = 0 2 cual es la condicién para que la barra descienda completamente? i, Cual es la velocidad maxima de descenso? Solucién. a) Sea m = L+ A+ py la masa de la barra y sea ¢ = 0 el instante en que se comienza a sumergir la barra, Las fuerzas que actuan sobre el objeto son la fuerrza gravitacional mg y la fuerza Fy dada por el principio de Arquimides. Como x(t) denota la longitud de la parte sumergida en el instante ¢ y el peso del fluido por unidad de volumen es pg, el peso del fluido desplazado en el instante ¢ es A x(t) pg y por lo tanto Fe = —Apg-x(t). Entonces aplicando la Segunda Ley de Newton ma" = suma de las fuerzas que actuan sobre el objeto se tiene la ecuacién m-2" = mg — Apgr(t). b) Haciendo » = 2’, en las variables v y « la ecuacién anterior queda de _ Apge “a m Separando variables obtenemos dy = G - aeu) dr, m e integrando lo 1Apg > 3 = gt Dom 2 tk Como en f = 0 tenemos x =O y t = tp, se tiene y asi 4 2_ _ Aed 2, 2 v= 2gx — SE8a? + af. Poniendo el valor de m, se reduce a = 29% - ae + vg. La barra desciende hasta v = 0. Luego, para determinar hasta donde desciende la barra debemos resolver la ecuacién cuadratica cuyas soluciones son Luego la barra desciende hasta eo | 2, LF 2 =f (e+e. pa OF c) Sito =0, la barra desciende hasta fol p y luego la barra desciende completamente en este caso si r= po L p oes ee >e. oF 2=L p25 La velocidad maxima de descenso se obtiene cuando #4 = 0. Como » dw de a a" tenemos wy = =0 = 2c =0 = mg-Apgr=0. De esta forma la velocidad maxima de descenso se obtiene cuando p= ™ _ Lh, Ap p Capitulo 4 Ecuaciones Diferenciales de Segundo Orden 4.1 Teorema de Existencia y Unicidad Como hemos visto una ecuacién de segundo orden es de la forma F(a. yyy") = (4.1) Bajo condiciones bastante generales sobre Ja funcién F’, la ecuacién (4.1) se puede escribir de la forma p 4 = f(x,y, (4.2) Como en el caso de las ecuaciones de primer orden, para éste tipo de ecuaciones también tenemos un teorema de existencia y unicidad de soluciones. Antes de enun- ciarlo. y a manera de ejemplo de Io que sucede en situaciones muy generales, anal- icemos la ecuacién yl! = 2? + sen(a). Integrando sucesivamente obtenemos , 7 3 a y(x) [ J ~ cosls) + er)ds =-p- sen(t) + (4% + C2. Como ahora la solucién general depende de dos constante arbitrarias, al imponer la condicién inicial y(0) = 1. por ejemplo, obtenemos como tinica condicién ¢ = 1. De esta forma, la familia de funciones que depende de la constante ¢1 ot ule) = 5 — sen(e) + ae + 1. es solucién del problema { y" = @ + sen(r) y0) = 1. Para fijar la constante c; necesitamos una condicién adicional, que puede ser el valor de la solucién en otro punto (problema de frontera) 0 bién, el valor de la primera derivada en el mismo punto (problema de valores iniciales). Observe que en el caso de problemas de frontera, estamos pidiendo que la sclucién pase por dos puntos distintos prefijados. Veremos mds adelante, que en muchos caso no existe tal solucién. Para el problema de valores iniciales se pide que la solucién pase por un punta dado y que la pendiente de la solucién en dicho punto asuma también un valor dado, A este tiltimo tipo de problemas se refiere el siguiente teorema, Recordemas primero que un subconjunto D del espacio es abierto si todo punto de D es el centro de un rectangulo que esta contenido en D. Mas precisamente, D es abierto si para todo punto (29, yo. 20) en D. existen ntimeros positivos a,b y c tales que cualquier punto (x,y, 2) que satisface | x — a |< a, | y— yp |< b. | 2-2 | 0 (resp. cos(r) < 0) para todo x € Ty. Una primera integracién nos da dy y una segunda Luego la solucidn general es rn(a) =In(; 1 t+art+e, rel, néeZ. Trost) Las soluciones que verifican y() = 1 estan definidas en Ty = Debemos resolver la ecuacién l=y(m) =am+e. Jo que nos da cy =1—¢7. Luego las soluciones que verifican y(m) = 1 son 1 \— -) . - y(t) = In Co :) +e(x-—m)+1 ve} : Si queremos ademéas que verifiquen y!(7) = 0. como y (x) = tan(z) +c debemos resolver la ecuacién O=y(m)=a. Luego la solucién es tinica y esta dada por con f continua sobre un conjunto abierto A del plano. En este caso introducimos la variable p= 4, de donde se obtiene @ = ar Entonces stituyendo tenemos dp ap LP) que es una ecuacién de primer orden. Ejemplo 4.2.2 Resolvamos la ecuacién diferencial ty . dy de? * “de Sea p = #4, Entonces tenemos 2 = £4 y la ecuacién de primer orden dp = oy + pam. Esta es equivalente a la ecuacién lineal de primer orden 2 a. cuya solucién es pix) = e See [a+ fel Mar] ple) = etme [at omer] 1 p(z) = = [evs f omar] ) 1 a p(x) = zp aty . Pero p = # implica dy _ x a TR Finalmente integrando obtenemos 2148 y(a) = 5, tote que es la solucién general. 3. Ecuaciones del tipo &y _ dy. Ste tu.D con f continua sobre un sobre un subconjunto abierto A del plano. También en este caso introducimos la variable p = a obteniéndose &y _ dp _dpdy _ dp a ~ de dye ay” y la ecuacién se reduce a dp _, dp p=fy.p). obien 7 dy que es de primer orden. Ejemplo 4.2.3 Encontremos la solucién general de la ecuacién _ (Wye (= fy da? . 2, Poniendo p= #. tenemos £4 = & Ja ecuacién queda = Te = ay — ? Dividiendo por p, la ecuacién se puede escribir de la forma dp _ dy D cuya solucién es Ply) = cry. Volviendo a las variables y y tenemos la ecuacién dy =cry quoes equivalentea Y= oar de HY due es ea ye Tntegrando obtenemos In(y) = 2 + In(c2) y exponenciando y(x) =c26". 4, Ecuaciones del tipo F(x, yyy") =0 donde F(x, y.y/',y") es la diferencial totalde una funcién y(x, y, y/)- En este caso nuestra ecuacion es dy =0 y por Io tanto sus soluciones son las soluciones de la ecuacién de primer orden w(a,yy) =e donde ¢ es una constante arbitraria, Ejemplo 4.2.4 Encontremos la solucién general de yy" + (y'P = La ecuacién se puede escribir como d(yy’) =0 Io que implica yy! = cr, 6 lo que es Jo mismo ydy = cidx cuyasoluciines y? =ct+e. 5, Beuaciones del tipo F(a.y.y,y") =0 tales que existe funcién p(x, y, y') de modo que p(x, y.y')F (a, y.1/,y") es la diferen- cial total de una funcién (x.y, y"). Como en el caso anterior, resolvemos »(ar.y,y/) = c. Entonces cada solucién de esta ecuacién es solucién de F(r.y,y',y") =0 o/y de pla.y,y/) = 0. Luego. debe mos eliminar las soluciones superfluas. es decir aquellas que verifican p(c.y.y/) = 0 © aquellas que indefinen 1 y no verifican F(x, y,y/.4") = 0. Ejemplo 4.2.5 Encontremos usando este método nuevamente la solucién general de yy" —(y'P =0. Multiplicando por u(y) = jf se obtiene w= _ gt y Luego tenemos = cute = Inyyscr + Ine) = y(t) = ac. La tinica funcién candidata a ser solucién superflua es y = 0 (ya que 1 no esté definida para y = 0), pero no lo es ya que claramente es solucién de Ta ecuacién original. 6. Beuaciones del tipo F(a.y.y,y") =0 donde F es homogénea respecto a la segunda, tercera y cuarta variable: es decir. existe n < N tal que para toda (2, y, 2, w) se tiene F(a, ky, kz, kw) =k" F(x, y,2,w) Introducimos una nueva variable 2 a través de la expresién gael, Derivando ambos lados con respecto a x dos veces. se obtiene y zl My ya (24 ach ete, y al reemplazar en nuestra ecuacién O=Fla.yyyy") = Fae! 24, ze! #42, (22 + 2!)ef 242) elena, 12,242) = F(r.1,2,2 +2!) =0, que es de la forma f(x,2,2") =0 (ecuacién de primer orden) . Ejemplo 4.2.6 Resolvamos la ecuacién uy!" — (y'P = bry? . Aqui F(e,y.y'.y") = yy" — (y)? — Gry? . que es homogénea con n = 2. Poniendo y = ¢! *", Ja ecuacién se transforma en Alt (2 42! — 2 — Gr) =0 que es equivalente a 2’ =6r. La solucién de esta tiltima ecuacidn es 2(t) = 30? +e, Jo que implica yx) = (fr ted — et terte | y por lo tanto y(x) = Qe, 4.3 Ecuaciones Lineales de Segundo Orden Son ecuaciones de la forma ao(x)y" + ar(x)y! + a2(2)y = A(z). (4.3) donde en general ap, a1. a2 y ¢ son funciones continuas definidas en un intervalo J. Un ejemplo importante de este tipo de ecuaciones es la que modela el movimiento de una masa acoplada a un resorte: mat dx kx = F(t), mp tg the = FO donde m representa la masa del objeto, ¢ y k son constantes y F es una funcién dada. Volviendo a Ia ecuacisn (4.3), si ao(x) # 0 para todo x € I. dividiendo por ap(x) reducimos (4.3) a su forma normal y" + pr(a)y! + pa(a)y = g(x). (4.4) Por lo tanto (4.4) es de Ia forma y” = f (2, yy") con f(,y,2) = g(x) — palx)y - pi(x)z. Ast tanto f. como Of /dy = —po(x) y Sf /z = —pi(x) son continuas en 1 x B®, y entonces nuestra ecuacién verifica el Teorema de Existencia y Unicidad (4.1.1). Mas que esto. se puede probar que. dado un punto (2t0. Yo. 20) € I x R?. existe una tinica solucién u de (4.4) definida en todo el intervalo J, tal que u(9) = yo y w(x) = 20 Para deducir con mayor facilidad importantes propiedades de este tipo de ecua- ciones diferenciales, asociado a las funciones p; y pz de antes, consideremos el ope- rador ZL que toma cualquier funcidn u, dos veces diferenciable sobre el intervalo I, y le asocia la funcién L[u] definida por Llu) = w(x) + r(x)! (x) + pola)ule) (4.5) Usando este operador Ia ecuacién (4.4) se escribe de la forma Ly =49(2). (4.6) Tal operador se Hama operador diferencial lineal pues verifica: 1) Leu! = cul para todo c € R. 2) Liuy + us] = Ly] + Liu). Combinando ambas propiedades se obtiene 3) DISTR cet] = EP L[up], donde cy... GR La demostracién de 1), 2) y 3) es muy sencilla y se deja de ejercicia para el lector. 4.3.1 Ecuacién Lineal Homogénea de Segundo Orden Son ecuaciones de la forma. y" + play + po(a)y = 0. (4.7) con P,P funciones continuas definidas en un intervalo 7. Usando el operador diferencial L esta ecuacidn se reduce a Ly =0. (4.8) Como consecuencia de la linealidad de L, se tiene el siguiente resultado: Teorema 4.3.1 1) Si m es solucidn de la ecuacién (4.8). entonces para todo ce R, cy es solucion. 2) Sin. son soluciones de (4.8), entonces 1 + yp es solucién. 3) Luego si tny+++.Ym son soluciones de (4.8), entonces cualquier combinacién lineal de ellas. digamos S, crye. €8 soluc 4) Si (4.8) (con coeficientes p;(x) reales) tien€ una solucién compleja y(x) = u(x) + iv(x), entonces la parte real u(x) y la parte imaginaria v(x) son solu- ciones (reales) de (4.8). 5) Siy es solucién de (4.8) y existe xo € I tal que y(xo) = y'(wo) = 0, entonces y(x) = 0 para todo a € I. Demostracién: 1) y 2) se dejan como ejercicios. 3) es consecuencia directa de 1) y 2), Para 4), notemos que si y(x) = u(2) + v(x) es solucién de (4.8). entonces Liyl(x) = L[w(x) + iL{v|(r) = 0, para todo €7 Pero como un niimero complejo es cero sélo si su parte real y parte imaginaria son cero, tenemos que Lw(z) = 0. Lele) = 0. para todox eI. y por Io tanto wy v son soluciones de (4.8) en I. Finalmente 5) sigue directamente del teorema de existencia y unicidad, ya que Ia funcidn idénticamente cero’es también solucién., Definicién 4.3.2 Las funciones u;(x),.... n(x) se dicen linealmente dependi- entes (L.D.) en el intervalo I, si existen constantes c1,....Cn. no todas nulas. tales que cru (2) $e +Gntn(t) =O para todo x ET. (4.9) Las funciones wi(),....tn(«) se dicen linealmente independientes (1.1) en I si (4.9) se verifica solo cuando cy = +++ = Cn = 0. Ejemplo 4.3.3 Las funciones 1,2,22,,..,2” son LiL. en cualquier intervalo 7. En efecto si Qtarter te +eq2"=0 Vr el entonces. todo € I es rafz de este polinomio que es de grado 0. Entonces ky. kz son raices reales y distintas de (4.11). y la solucién general es or y(a) = ae + ee! a eR, 2) d=0, Entonces hy = ky = -2L € Ry yi(x) =e es soluci6n, Afirmacién yo(x) = xe" es también solucién. En efecto, = kay (a) + (4.12) (4.13) yl) = hye? 4 eh => b(a) = kiyp(a) + re’ De la ecuacién (4.12) obtenemos e"!* = y$(x) — kiyo(x). Reemplazando esto en (4.13) obtenemos 2 a ai Ya) = Payal) — Ruel) = Th) - ee) = &, tenemos y como. 2 ay a2 yb(x) = Ge — PO: lo que implica acy3 (x) + arys(x) + aryp(x) = 0. Esto prueba la afirmacidn y por lo tanto Ja solucién general en este caso es ya) =e" (cy + a), C0 ER. 3) d <0. En este caso ky, kz son ntimeros complejosieonjugados, . a =a-if, k= : =-—, ky=a-if, kk=a+i8. con a as De esta forma 019 = 9 (cos(Gx) — isen(Gx)) y 2" = €* (cos(Gx) + isen(Gx)) son rafces complejas de (4.10). Luego la parte real y1(x) = e** cos(Ar) y la parte imaginaria y2(x) = e#sen(x) son soluciones reales. Ademés como ellas son L.L., la solucién general es ylx) = €*(cy cos(Bx) + cxsen(2)). e102 ER. 0. Ejemplo 4.3.14 y/’ — 3y' + 2y La ecuacién caracteristica es -3k+2=0, cuyas rafces son ky = 1 y kp = 2. Por Io tanto la solucién general es y(a) =e +e. 4,0. ER. Ejemplo 4.3.15 y/’ + 4y/ +5y =0. La ecuacién caracteristica es 44k +5=0 cuyas rafces son ky = —2 —i y ko = —2 +7. Por lo tanto la solucién general es 2" (c cos(x) + esen(xr)), c1.@ €R. Ejemplo 4.3.16 y+2y/+y=0. La ecuacién caracteristica es +2k+1=0, cuyas rafces son ky = ky =—1, Por lo tanto Ja solucién general es y(t) =e" (4 +x). C1. ER. 4.3.3 Ecuacién de Euler Son ecuaciones de Ja forma aen?y" + any! + ay = 0. (4.14) CON dp, a1. 42 constantes reales, ac # 0. Si hacemos la sustitucién x = e! (para x > 0), obtenemos 4 = e! y por lo tanto =e, De esta forma &y _ dd d dy dt wn EY aay ty et = Y= ae > a at wat as = by a _ dy a = -a ty _ ty) de dt de at’ Reemplazando en (4.14) obtenemos om en Ey _ ayy ace + ay =0, dP dt que es equivalente a la ecuacién lineal homogénea con coeficientes constantes: ood + (a = 05) + any =0. (4.15) La ecuacién caracteristica de (4.15) es agk? + (a1 = ag}k + a =0. De este modo si ky es rafz de esta ecuacién, y(t) = ef es solucién de (4.15), lo que implica que y(x) =e es solucién de nuestra ecuacién inicial (4.14). Nota. En la practica a veces es conveniente buscar directamente soluciones de (4.14) de la forma y(x) = x. Ejemplo 4.3.17 «y+ 32y'-y=0. La correspondiente ecuacién caracteristica es 2 3 = + Sk-1=0, cuyas races son ky = 5 y ko = —2. Por lo tanto la solucién general es y(t) =? ter, 4m ER. Ejemplo 4.3.18 22y"— ry! +y=0. La correspondiente ecuacién caracteristica es RP -2k+1=0, cuyas rafces son ky = k2 = 1. Por lo tanto son soluciones para la ecuacién transfor- mada t nae. y wll) ste. Ast w(t} =x, y w(x) = (In(2))a, son soluciones de nuestra ecuacién y lasolucidn general es y(t) =2(4 +eIn(e)). ce ER. Ejemplo 4.3.19 22y'+2y'+y=0. La correspondiente ecuacién caracteristica es R+1=0, cuyas rafees son ky = -i y ky = i. Por lo tanto son soluciones para Ja ecuacién transformada y(t) = cost). y yo(t) =sen(t). Asi w(x) =cos(In(x)). y ye(x) = sen(In(x)), son soluciones de nuestra ecuacidn y la solucién general es y(a) = cy cos(In(z)) + esen(In(x)), 1.02 ER. Ejercicio 4.3.20 Considere la ecuacién ay(ax + b)?y" + ar(ax +b)y! + aay =0 Por medio de una sustitucién de variables transformela en una ecuacién de Euler y resuelvala, 4.3.4 Ecuaciones Lineales de Segundo Orden no Homogé- neas Consideremos la ecuacién y! + pr(a)yl + palaly = f(a) (4.16) donde p;.p2 y f son funciones continuas definidas sobre un intervalo J. Usando el operador diferencial lineal L definido en (4.5). esta ecuacién toma la forma Ly |= f@). (4.17) Las siguientes propiedades son consecuencia inmediata de la linealidad del operador L. 1) Sim es solucion de L{y ] = 0 y @ es solucién de L{y |] = f(x). entonces y+ 9 el solucién de Liy | = f(x) 2) Si y; es solucién de Lly ] = fi(z). para i = es solucin de Ly |= 0", a: fila). donde a; n, entonces y(x) = 72 .,m, son constantes. ayia) 3) Suponga que las funciones p1,p2,U y V son real valoradas. Entonces, si la ecuacién Ly] =U(a) + iV(x) tiene solucién yla) = u(x) + ée(2) con wy 0 real valoradas. entonces u(¢) es solucién de Ely] = U(x) y v(x) es solucién de Lly) = U(c). Teorema 4.3.21 Considere la ecuacién Lily ] = f(x). con coeficientes pi,p2 y f continuos en un intervalo I. Si cyyy(x) + Coys(x), con ¢4.C. € R, es la solucién general de L[y ]=0. y § es una solucién particular de L[y ]= f(a), entonces Y(x) = Crt(2) + w(t) +92). a.@ER, es la solucién general de L/y ]= f(x). Demostracién Sea , una solucién cualquiera de L[y] = f(x). ‘Tenemos que demostrar que existen constantes c,.cz € R tales que a(x) = cm (e) + coyel(x) + Ha), Va el. Pero como § — J es solucién de la ecuacién L[y ] = 0, existen constante c.c € R tales que n(x) — (2) = crm (x) +ceom(x), Vr el, lo que termina la demostracién. Ejemplo 4.3.22) y"+y=2. Claramente (x) = x es solucién particular. Consideremos ahora la ecuacién homogénea y” + y = 0. Su ecuacién caractel es k2 +1 =0y por lo tanto su solucién general es a cos(t) + @sen(r), G.@eR. Por lo tanto la solucién general de nuestra ecuacién inicial es y(x) = c.cos(z) +esen(r) +2. C1. €R. 4.3.5 Método de variacién de constantes A continuacién introduciremos un procedimiento para encontrar una solucién par- ticular de una ecuacién lineal no homogénea bajorél supuesto que conocemos la solucién general de la correspondiente ecuacién homogénea. Como siempre L denota el operador Lul(x) = u"(x) + pi(a)u'() + p22)u(e). donde py,p2 son funciones continuas sobre un intervalo I Suponga que cyys(x) + coyo(x), C1-¢ € Res la solucién general de Ely ] = 0. Dado una funcion f continua sobre 7, buscaremos una solucién particular de L[y ] = F(a) de la forma: y(x) = ea(x)yr(%) + c2(x)9n(a) » Tenemos entonces dos funciones incdgnitas ¢4() y ¢2(2). Estas deben ser tales que e(x)y1(x) + c2(2)yp() satisfagan la ecuacién y" + palx)y! + pala)y = f(x). Es decir tenemos dos funciones incégnitas y una tinica ecuacién. Podemos entonces pedir que ¢(z) y c2(x) verifiquen una ecuacién adicional que facilite su célculo. Observe que si y(x) = ex(x)yr(x) + c2(x)y2(2). entonces (x) = ca(x)yy (x) + c2(x) yg (x) + 4 (2)rn (2) + (x) y2(z) - Para que por lo menos al hacer Ia primera derivada de y(:r), las funciones c1() y (x) se comporten como constante. imponemos la condicién adicional Anne) +(x) =0. Veer. Con esta condicién tenemos y(x) = e(x)yn(a) + co(e)yo(x). ¥(@) = al(z)y (2) +e(@)y(e) y w(x) = aay (2) + ea@ys(2) + Ayla) + A(x). Reemplazando en nuestra ecuacién y ordenando obtenemos F(x) = (x)y (a) + Bla)yb(a) + c1()(Y(@) + prlw)ah (a) + pola)a(a)) +2(2) (yf (x) ee x) + po(x)yo(2)) e4(x)4 (2) + B(x) vb(x) Por Io tanto nuestras funciones ¢1(x) y c2(ar) deben satisfacer el sistema. { GQ (x)yn(@) + G(x)ya(x) ean (@) + B(e)ya(a) con funciones incégnitas ¢(x) y (2). Observe que para todo x € I. el determinanite del sistema nla) wala nla) abla coincide con el wronskiano W(2) de la ecuacién homogénea. Como my y y2 son L.L. W(x) #0, y por lo tanto el sistema siempre tiene solucidn. Estas soluciones son — _f£@) (2) 1) — Lima) qa) = Way ¥ @9= Way Asf encontramos ¢,(2) = $1(2), Alar) = $2(z). Finalmente integrando obtenemos ate)= filer ba y ele) = f datnide+ a Ejemplo 4.3.23 y+ y= se Como la solucién general de y+ y=0 es cy cos(x) + c2sen(x), ponemos: y(x) = c1(x) cos(x) + c2(x)sen(x) « y tratamos de resolver el sistema (x) cos(x) + (x)sen(z) = 0 = (r)sen(x) +(x) cos(x) = Resolviendo obtenemos (2) =~ ey(a) =n os(a) Hy A(z) =1=> o(2)=2+G. Luego la solucién general es y(2) = G cos(x) + &sen(x) + In(| cos(x) |) cos(z) + asen(x), G,@ ER. Ejemplo 4.3.24 Hallar la solucién general de la ecuacién 2 1 ny 2d yi tc ty=s, (v0) sen(r) sabiendo que g(x) = “% es solucién particular de la correspondiente ecuacién homogénea. Para encontrar una segunda solucién yo(s) de la ecuacién homogénea. lineal- mente independiente con (2), usamos la férmularde Abel: p(t) = Co) [ecimtetey (oy ae, con p(t) = 2. Como . sen(r) f 1 a? wir) = - / pew z sen2(a) sen(:r) 1 sen(x) —cos(z) =
1, para cada i= 1,+++.m, usando este método podemos encontrar una solucién particular yj(z) de la ecuacién doy" + ary’ + aay = €"*(Pi(x) cos(qix) + Qi(x)sen(qix)) « Luego ype) = Soule) iat es solucién particular de (4.18). Consideremos entonces la ecuacién acy" + ayy! + ay =e" (P(x) cos(qx) + Q(x)sen(gr)) , (4.20) donde ag, a,.a2.r y q son constantes reales. dy #0. y P(«),Q(x) son polinomios. Teorema 4.3.25 Sean = max{gradoP. gradoQ}. a) Sir + ig no es raiz de la ecuacién caracteristica (4.19), entonces la ecuacién (4.20) liene solucién particular de lar forma up(#) = €'*(Rn(x) cos(gr) + Sn(x)sen(qr)) « donde Ry(x),Sp(x) son polinomios de grado n. b) Sir + ig es ratz de multiplicidad a de (4.19), entonces la ecuacién (4.20) tiene solucién particular de la forma pla) = xe" (Rn(x) cos(qr) + Sn(x)sen(qr)) . donde Ry(x),Sp(x) son polinomios de grado n. En cada caso los coeficientes de los polinomios Ry(x),Sq(x) se calculan reem- plazando yp(x) en la ecuacién. Ejemplo 4.3.26 Encontremos una solucién particular de Ia ecuacién y" + 4y! +5y = 2c, Como r + ig = 3 no es raiz de la ecuacién caracteristica R+4k+5=0, y el maximo entre los grados de P(x) = 2 y Q(x) = 0 es cero, debemos buscar una solucién particular de la forma ypla) = Ae, Para encontrar el valor de A calculamos las dos primeras derivadas de 1, yp(a) =3de* — yy y(x) = 9Ae**, y teemplazamos en la ecuacidn diferencial obteniendo 9Ac** + 12Ac** + 5Ac*™ = 2%", Por Io tanto 1 26Ac* = 2 => A=K, BAe’ fe B y nuestra solucién particular es 1 Yp(2) = BO Ejemplo 4.3.27 Encontremos una solucidn particular de Ja ecuacién y" +5y' +4y = 3+ 82? + 2cos(2x). La ecuacién caracterfstica es 4+ 5k+4=0. Escribamos la ecuacién de la forma Ly] = fi@)t fala), con fi(x) =3+ 822 y folx) = 2cos(2r). Para L[y] = fi(x). como r + ig = 0 no es rafz de la ecuacién caracteristica y grado de P(x) = 3+ 82? es dos, tenemos solucidn particular de la forma y(t) = Ap + Are + Ana? Con respecto a Liy) = fo(x). r-big = 2é tampoco es rafz de la ecuacidn caracteristica, Ademas como el maximo entre los grados de P(x) = 2 y Q(x) = 0 es cero, tenemos solucién particular de la forma ye(x) = Ag cos(2x) + Agsen(2x). De esta forma la ecuacién inicial Liy) = f(x) + a(x), tiene solucién particular de Ta forma . Yplt) = Ag + Art + Apt? + Ag cos(2r) + Aysen(2r) . Tenemos yf(a) = Ay + 2Ayx — 2Agsen (2x) + 2A, cos(22) , oy f(a) = 2g — 4Ag cos(2z) — 4Agsen(2z) Ast Liy|(x) = 249 — 4Ag cos(2x) — 4Agsen(2x) + 5(A + 2Aox — 2Agsen(2xr) +2A4cos(2x)) + 4(Ay + Ait + Aor? + Ag cos(2x) + Assen(2:r)) = (2A, + 5A; + 4Ay) + (10Ap + 4A})2 + 4An2? + 1044 co8(22) —10Agsen(2r) , y comparando con fila) + fol) =3 + 82? + 2cos(2x) , obtenemos las ecuaciones 2A2+5A1+44 = 104, + 4A, = 4A, = WA, = -10A; = oN wow cuyas soluciones son As =0- Ars By Ay = 2, A= —9. Ay =6. Luego Yp(t) = 6 — Sw + 2x? + Ben(2n) : es la solucién particular buscada. Ejemplo 4.3.28 Busquemos una solucién particular de y! — yf — by =e 4 267, La ecuacién caracteristica es RP -k-6=(k+2(k-3)=0. Como —2 es raiz de multiplicidad uno de ella, la ecuacién L[y] = e72 tiene solucién de la forma n(x) = Apte™**. Por otra parte —3 no es raiz de la ecuacidn caracterfstica y luego L[y] = e~** tiene solucién de la forma. y(t) = Are. Sea entonces pla) = Apne? + Arem**, Derivando se tiene Ufa) = Aye" =2Agte* —3Aye“™ yy yll(x) = —4 Ace“? 4 4Age?* + 9A Luego Liypl(a) = —4Age7®* + 4Agne~" + 9Ayem™ — (Ace? = 2Apae~* — 34,7”) —6( Are?" + Aye") = -5Age** + GAre™* y comparando con ¢~2" + 2e~8*, obtenemos Por Io tanto 1 1 ) a al ent 4g Lona yp(a) = —zae"™ + 30 es la solucién particular buscada. 4.4 Ejercicios resueltos Ejercicio 4.4.1 .Encuentre la solucién general de la ecuacién @y wx dy 1 de? * T-ade Tae! 1 sabiendo que una solucién de Ja ecuacién homogénea asociada es yy (x) Solucién. Usando férmula de Abel tenemos una segunda solucién de la ecuacién homogénea de la forma y2 = vy. con [aelroras -[z Sdn gy a er [a Arteyy = [ovemriar [oe —l)dz= [roa - [ae = re". Por lo tanto y2(x) = —xe~"e* = —x. De esta forma la solucién general de la ecuacién homogénea es a(x) ila) = ce = Or. Buscamos ahora una solucién particular de la ecuacién no homogénea de la forma p(x) = cr(a)e” = cola)x. Luego las funciones ¢ (x), &(2) deben satisfacer el sistema A(xjet—A(a)x = 0 Awe (a) = 1-2. Resolviendo el sistema obtenemos A(a)=-ce® = A)=-1 = Por Io tanto la sclucién general de nuestra ecuacién es y(x) = (vem + 7? + ye” — (-a + c9)2, o bien ya) =a? +r4+14+q a. Ejercicio 4.4.2 Hallar la solucién general de la ecuacién vt ty= » (v#0) sabiendo que g(x) = es solucién particular de Ja correspondiente ecuacién homogénea. Solucién, Para encontrar una segunda solucién ys(x) de la ecuacién homogénea. linealmente independiente con y4(r), usamos la formula de Abel: yo(r) = Co) [cimestey oy ae, con p(t) = 2. Como = [ m(ojte = -2In(r) = In(x™?), sen(z) fi 22 w(t) = OMY fo _ ae snl) v | Pera) _ sen(z) 1 ne a may sen?(r) _ _c05(e) - rc Por lo tanto la solucién general de la homogénea es sen(x) ._cos( wa) = PA pO oc m a Para encontrar la solucién general de la ecuacidn no homogénea usamos el método de variacién de constante. Sea sen() cos(x) u(r) = az) + cfc) Debemos entonces resolver el sistema Ge) 22 + 4(2)22 = 0 ) eosndesenta) of (gy xusenla)meos(n) a Sus soluciones son e integrando obtenemos a(x) = sen(t)+c1. ca(r) = cos(z) +e. Por Io tanto Ia solucién general de la ecuacién dada es yx) = con(2) ey) 22 + (cos(a) +2) s(t) es decir & Go) 1 ule) = a(0 A Foy + -. Ejercicio 4.4.3 Encuentre la solucién general de la ecuacién a £Y a B+ y= an(s), Ie haciendo la sustitucién 2 = 4 Solucién.Sea x = 4. Por lo tanto dx = ~ pul = Entonces dy = dy dt ly dx ~ dt dx ~ a &y d de da = 2p 4 wt dt e Sustituyendo obtenemos gh (oad . wy) — lov (0 43 (209 + on shed + y= tan(4), yy ty = Tan (5) . (421) Como la ecuacidn caracteristica es decir tiene rafces m = +42, la solucién general de la ecuacién homogénea &y dt? t t y(t) = c.cos (5) _c2sen (5) > @@QeR. Usando el método de variacién de parametros, buscamos una solucién particular de (4,21) de la forma w(t) = catyeos (5 ) + althen (5) . Luego debemos resolver el sistema es 0 qtan (5) « + edison (8) + c(t) cos (5) ; 1 t) ft 1 l 1 l a(t) = 5 tan 5) sla) = ~x8ee(5 + pla) = . t . t a(t) UI I —~ 8 8 ae ay + a 5 — oc YS N= + & 5 mae ey + 2 Por lo tanto tnfsce(£) + tn (£)] + sn ($)] eos (4) - cos ($) se (4 sec ( 5 an (5) + sens }] (5 cos {5} sen { 5 t t t = =nfue() + tan(!))e(). y la solucién general de (4.21) es Ll Ly) t l Ll y(t) = -In [x (5) + tan (). cos (5) +c, cos (5) + csen (5) 14, ER. De esta forma la solucidn general de nuestra ecuacién es wo = ohn (2) + (2 oe(B) + 1 1 crcos(—]} + esen(—}. c.e2ER. Qn Qa p(t) Ejercicio 4.4.4 Usando el método de los coeficientes indeterminados encuentre la solucién general de la ecuacién y! —y = 26>" — dae + 10 cos(2x) . Solucién. El polinomio caracteristico de Ja ecuacién homogénea yf’ — y = 0 es k2 —1=0. Luego la solucién general de la’ homogénea es yn(x) = cre~* + ene". Para encontrar una solucién particular de Ja ecuacién no homogénea usando coefi- cientes indeterminados, separamos en dos ecuaciones y’-y = e%(2-40) y (4.22) y"—y = 10cos(2) (4.23) Como -1 es salucién de la ecuacién caracterfstica debemos buscar una solucién par- ticular de (4.22) de la forma ys(o) = xe“*(Ar + B). Tenemos yi(v) = e7"(—Aa? +(2A-B)x+B) y yff(x) =e7*(Av? +(B—4A)x + 2(A-B)) y teemplazando en la ecuacién obtenemos e?(-4Ar + (4 = B)) = 2(2— 42). Luego A =1 y A— B=1, lo que implica B = 0. Por lo tanto 2, y(t) = 2° Consideremos ahora la ecuacién (4.23). Como 2 no es solucién de Ja ecuacién carace teristica, buscamos una solucién particular de la forma tp(2) = Ccos(22) + Dsen(2z) . Tenemos g(a) = -2Csen(2x) + 2Dcos(22) y yf) = —4C cos(2x) — ADsen(22) , y reemplazando en (4.23) obtenemos =5C' cos(22) — 5Dsen(2x) = 10 cos(22) Por lo tanto C = -2, D=0y yo(w) =)—2cos(2r) Luego p(x) = yi(x) + wo(x) = 276 — 2cos(x) es solucidn particular de nuestra ecuacidn original y su solucién general es y(x) = cye~* + ene + 27e-* — 2eos(x}. . 4,02 ER. Ejercicio 4.4.5 Para x > 0 encuentre la solucién general de la ecuacién Ary" + (2 - 8¥n)yl — Sy = (BVE + 2Qerv". usando el cambio 2 = 2. Solucién, Ponemos x = (2 lo que implica # = 2t, Ademés y = Bea Ley dr dt dx 2t dt y= 22m) a (1m). da? dx \ 2t dt dt \ 2t dt dx _ lL id 1 ey) _ alm iH “2 Reemplazando obtenemos wi [ie & t dt aE o bién éy dy , Ge AG By = ett + D. (4.24) La correspondiente ecuacién caracterfstica es 4k 5 =0=(k + 1k -5), y por lo tanto Ia solucién general de la ecuacién homogénea es nlf) = ce! + eet, Usando el método de los coeficientes indeterminados. buscamos solucién particular de (4.24) de la forma yp(t) = te“(A + Bt) Entonces y(t) = e(-BP + (-A + 2B)t + A) y(t) = e'Be + (A - 4B)t + 2(-A + B)). Reemplazando obtenemos e'(-12Bt = 6A + 2] = e'Bt + YQ]. Jo que implica Luego la solucién general de (4.24 y(t) = hes + 3t) + cet + coe. y por Io tanto la solucién general de nuestra ecuacidn inicial es ula) = ~PVFMG + BVE) + eM + exe, Ejercicio 4.4.6 Encuentre la solucién general alrededor de x =0 de la ecuacién y" + y = tan(x) + 3x - 1. Determine ademés el intervalo maximo donde esta definida. Solucién. Nuestra ecuacién es y! + y = tan(x) + 3% - 1, (4.25) Observemos primero que la ecuacién no esta definida para los x de la forma x = E +m donde n es un entero, ya que en estos x la funcién coseno se anula y por lo tanto no estan en el dominio de Ia funcién tangente. Como queremos una solucién alrededor de x = 0, debemos partir imponiendo la condicién —3 < x < %, Resolvamos entonces la ecuacién para x €] — al. La solucién general de la ecuacién homogénea es mlx) = creos(x) + cysen(2). Resolvamos primero usando variacién de pardmetros Ia ecuacién y" + y = tan(x) (4.26) Buscamos entonces solucién de (4.26) de la forma wl) = (x) cos(x) + ¢2(2)sen(x) , y por lo tanto debemos resolver el sistema cos(x}c4(x) + sen(x)&(x) = 0 —sen(a}c4(x) + cos(x)eh(x) = tan(x). Se obtiene Zz A(x) = —tan(x)-sen(x) Gla) = sen(x). e integrando = sen(r) — In(sec(x) + tan(x)) + cr. —cos(%) + ¢. a(z) elu) Observe que todo esto tiene sentido ya que para x €] — 4, 3[ tenemos que sec(x) + tan(x) > 0. Luego la soluci6n general de (4.26) es ys(z) = ex(x) cos(x) + c2(x)sen(x) — In(sec(x) + tan(z)) cos(x) , [ "y= 30-1 (4.27) usando el métado de los coeficientes indeterminados. buscamos solucidn de la forma y ésta esta definida para tado x €] — 3, Para resolver vr w(t) = Av + B. Reemplazando en (4.27) y comparando coeficientes se obtiene Jo que implica w(x) = 30-1, Luego la solucién general de la ecuacién (4.25) es y(x) = er(x) cos(x) + c2(x)sen(x) — In(sec(x) + tan(z)) cos(z) + 3x — 1, y el intervalo maximo donde esta solucidn esta definida es | — 3,3). Ejercicio 4.4.7 Encuentre la solucién general alrededor de x = de la ecuacién y! + y = tan(x) + 3% - 1, y determine ademés el intervalo maximo donde esta definida. Capitulo 5 Aplicaciones de Ecuaciones Ordinarias de Segundo Orden Asf como hemos hecho con las ecuaciones diferenciales de primer orden, presentamos ahora algunas aplicaciones de las ecuaciones diferenciales de segundo orden. 5.1 Curvas de Persecucién La primera aplicacién que veremos se refiere a la determinacién de la trayectoria que sigue un cazador al perseguir su presa. Ejemplo 5.1.1 Supongamos que un barco A, que viaja a velocidad constante a, esta persiguiendo a un barco B que viaja a velocidad constante 8, En ¢ = 0, suponemos que A se encuentra en el origen (0,0) y que B esté en el punto (6,0), 6 > 0; y que para t > 0, B se desplaza por la recta « = 6. Al cabo se ¢ horas, A se encuentra en P = (x.y) y Ben Q = (b. t). Determine la trayectoria de A como funcién de x para el caso a > B. Figura 26 Como el barco A persigue al barco B. la recta tangente al gréfico de la curva y = y(x) en el punto P debe pasar por el punto Q. Esto implica dy _ y=pt dt Como A avanza con velocidad constante a, en el tiempo ¢ recorre aé kilometros. Luego Ia longitud de la curva que recorre A en el tiempo ¢ es at; es decir, a= [' VIF TTF = Su- wl a0 y= [ view an. Poniendo y/ = w, tenemos Sy-we-b) = [ ViF wr du. y derivando con respecto ax yuo) -u)=View => Gb-aju! = view. Separando variables obtenemos la ecuacién a Vivw > ax—5 Las condiciones del problema-indican que w(0) = 0. Integrando tenemos B (a= In(w+vI +a) = —2In (5) a "Vo = w+vit = Vin = -w + (4) Cuando el barco A cruza la recta x = b captura al barco B. Por Io tanto el punto en que el barco A intercepta al barco B es 6.90) = ( bab ) / a? = y el tiempo 7 que demora la captura es ba ‘= 2-R Ejercicio 5.1.2 Desarrolle el ejemplo anterior para el caso a = 3. Ejercicio 5.1.3 Un conejo parte del punto (2,0) y corre por x = 2a una velocidad de 10 Km/H. Al mismo tiempo un perro sale de (0,0) con velocidad 15 Km/H persiguiendo al conejo. j, Cuanto tiempo demora el perro en pillar al conejo? 5.2 Movimiento de una Particula La ecuacidn del movimiento de una particula, segtin la Segunda Ley de Newton, es mi =F, (5.1) donde F es la suma de todas las fuerzas que acttian sobre la partfcula. m es la masa de la partfcula. & es la aceleracién de la particula relativa a algtin sistema de referencia. A.- Movimiento rectilfneo. 1.- Partfcula proyectada verticalmente hacia arriba, Supondremos que las tinicas fuerzas que acttian son: a) la fuerza gravitacional mg. donde m es Ja masa de la particula. y b) una fuerza de resistencia proporcional al producto del cuadrado de su velocidad por su masa. Designemos por x(t) la altura en que se encuentra la partfcula medida desde el punto de propulsién en un instante ¢ posterior, y por v = ¢ = & su velocidad. Reemplazando en (5.1) obtenemos la ecuacién de segundo orden mi = —mg — mk(é), que se reduce, usando la relacién v = 4 y simplificando por m, a —g- ke. ( a & Esta ecuacién se puede escribir de Ia forma pees = gt kv 1+ ( Integrando el lado izquierdo entre yy = v(0) (velocidad inicial) y v(¢) y el lado derecho entre 0 y £ se obtiene arctan (\s ) =-t4+ oem (\ w) . P= Fpgavctan (VE) tenemos u(Z) = 0, Luego este Fes el tiempo que debe trascurrir para que la partfcula aleance su altura maxima. Para poder calcular T = a(é), es decir, Ja altura maxima que alcanza la particula. vamos a escribir nuestra ecuacién (5.2) en términos de v y de x. Tenemos 1 > Observe que para du _ duchy _ dv “Gad ke” y reemplazando en (5.2) obtenemos de > vdv Ug => gem 2kudv Sot = The. Observemos que como la altura inicial 2(0) = 0, tenemos que v = v(2)/r=0. En- tonces integrando el lado izquierdo entre vy = v(0) y v(x) y el lado derecho entre 0 y , se obtiene In(g + ke(x)) — In(g + kez) = —2k2. Para calcular resolvemos v() = 0; Inego pet _ ay) 1) (gtk) 1 ko T= ~sr(In(a) Info + be) = 3m ( Ht) = Sin(i +b). 2. Particula proyectada verticalmente hacia abajo. Supondremos que las tinicas fuerzas que acttian son: a) la fuerza gravitacional mg, donde m es la masa de la particula. y b) una fuerza de resistencia proporcional al producto del cuadrado de su velocidad por su masa. Designemos nuevamente por 2(é) la distancia recorrida por la particula en un ins- tante ¢ posterior, y por v = 4 = # su velocidad. Nuestra ecuacién es ahora mi = mg — mk(é)?, ya que ahora la fuerza gravitacional esta a favor del movimiento de la particula. Usando la relacién v = & y simplificando por m, abtenemos g— ke. (5.3) b Sea ti) = v(0) la velocidad inicial. Observe primero que si vo = \/f . tenemos la solucién constante u(t) = \/¥ y por lo tanto x(é) = \/$t. Para tc # \/¥ la ecuacién se puede escribir de la forma de dv ore Sot E 1a(\/E up Como la ecuacién (5.3) cumple las condiciones del Teorema de Existencia y Unicidad de soluciones (Teorema 4.1.1) y a(t) = /Z es solucién, la condicién w< 2 = wt) Vt = o> /2 vero. Luego en cualquiera de los dos casos se tiene — E(t) tevin >0 vt>0. Por lo tanto, integrando el lado izquierdo de (5.4) entre vp y v(t) y el lado derecho entre 0 y ¢ se obtiene teu) | 1+ Fu 7 Vil-m ie (Se al Agrupando y exponenciando se tiene vo+ ve ut) vG= Vk ve _ ove, Va- VEU) 9+ VR tw . ; vw Despejando la variable v y denotando K = 44 a e tiene KVg+KVkou = (vg—- Veo) ev". ovk (K +e") =V (ev5"- K) . elt Ke its K’ que implica pvt KEL EK Sea w = K +eV"5" : entonées dw = 2/kgeV"9' dt, y como w-K = eV'5t, dx = dt. (5.5) obtenemos dw = 24/Fg-(w— K) dt. Luego 1 ail IVegu-Kk =" De esta forma, reemplazando en (5.5) se obtiene de = i . (w-K) - du w ” 3k (w — K wa Ky E ) > w(0) = 1+ K > 0. integrando obtenemos ya t nn (_)- bn (te*® aw) =e me): Luego 1 K+ erst 1 ar )o = mf) - — n(evt = 5 »( 1+K a n(O*) 1 = (GE (66) Observe que cuando v = /F, tenemos K = 0, y reemplazando este valor en (5.6), se recupera la solucién x(t) = \/¥ t, obtenida anteriormente. 3.- Partfcula proyectada hacia arriba desde la superficie de la Luna. Supondremos que la mica fuerza que domina es la fuerza gravitacional y que esta varia con la altura. Se sabe que la fuerza gravitacional de cualquier planeta o luna varia inversamente proporcional al cuadrado de su distancia al centro. Usando esto tenemos la ecuacién mi = kr, donde r(t) es la distancia a que se.encuentra la partfcula en el instante ¢, medida desde el centro de la luna. y= ar de dr y ei Como v = 4, tenemos # = a a = y He, dt Reemplazando en la ecuacién obtenemos mudo =f r mop meak_k TY = FSF > 2(kK_k my, = ur) =2(¢-44 3). Si aes el radio medio de la luna, tenemos k = mg a? donde gp es la gravedad sobre la superficie de la lina. Reemplazando en la ultima ecuacién se obtiene Si ocurre que 25 > 1, es decir si 12 > 2goa, entonces u(r) > 0 para todo #. y la particula escapa ‘del campo gravitacional de la luna, Por el contrario, si 5 < 1, entonces existe cierta altura 7 en la cual v(7) = 0, es decir, la particula alcanza una altura maxima 7 y luego regresa a la superficie de la luna. Es facil ver que Para calcular el tiempo que tarda en alcanzar su altura maxima 7. consideramos dr 6 a = fema(2-14 338) (con +,/>7 ya que t > 0), 0 bien tr Baal, Vita 8 con oy = gi; = 1. Resuelta esta ecuacién debemos encontrar 7 tal que r(Z) = 4. Un paracaidista cuyo peso (es decir, masa) es de 80 Kg, se deja caer de un helicdéptero que se mantiene a 6,000 mts. de altura. Suponemos que cae bajo la influencia de una fuerza gravitacional constante y que la resistencia del aire es pro- porcional a la velocidad del paracaidista. La constante de proporcionalidad es 10 Kg/seg cuando el paracafdas esta cerrado y 100 kg/seg cuando el paracafdas est abierto. Si el paracafdas se abre 1 minuto-después que el paracaidista abandona el helicdptero, ial cabo de cuanto tiempo llegar a la superficie? Solucién: Sea x(Z) Ja distancia relativa al helicéptero en que se encuentra el para- caidista en el instante £, Luego la ecuacidn del movimiento es mit(t) = mg — ki), & es decir. =mg—-kv => mi (2) =o E g+ Bu e integrando k = 9+ 0 = (-9 + oD) ent = = Zot (w- Po). Tenemos m = 80K y consideremos g = 9.81mt/seg?. Cuando el paracafdas esta cerrado tenemos ty = 0 y k= 10K g/seg. Esto nos dé v() = Fo =F) = 78.48 (1 - o¥) = rfl) =78,48 G +8 (o# - 1)| = 78,481 + 627.84 (oF - 1) . Como e7#! = 1 evaluado en 1 = 60 es -0.99944692.., aproximando esta cantidad por —1 obtenemos u(60) = 78,48 y x(60) = 4,080,96. Cuando el paracafdas se abre en la ecuacidn Hi) = Bo (w= Bale tenemos & = 100K'g/seg y las condiciones iniciales xr(0) = 4.080,96 y to = 78,48. Luego U(l) = 7,848 + (78, 48 — 7, 848)e“4# u(t) = 7,848 + 70, 63267 = a 7, 8481 — 56, 5056 (e7# ~ 1) + 4,080.96. Luego debemos resolver 7, 8481 — 56,5056 (e¥* - 1) + 4,080, 96 = 6.000. © Io que es lo mismo 7.8481 — 56, 5056e# = 1,975, 5456. La solucién de esta ecuacién es aproximadamente ¢ = 251,725 segundos. Luego se demora aproximadamente 311,725 segundos en llegar a la superficie. B.- Proyectiles (sin resistencia del aire). Suponemos que hay una velocidad inicial y que luego est sometido solo al campo gravitacional. Sean z, z las coordenados del plano del movimiento. Suponemos (x,2) =27+2k —mgk : fuerza gravitacional actuante. Vp = Vo(cos(a)7+ sen(a)k) : vector velocidad inicial. r(t) = (2(t),2(W) : posicién del proyectil en el instante ¢. La ecuacién del movimiento es entonces mi! = —mgk ¢ integrando obtenemos * = -gkt+i(0)=—gkt+Ve. lo que implica r(t) = ~aks +Vat Es decir, r(t) = (2(2). 2() = (Vocos(a)t. Vosen(a)t — 05). x(t) @ cos(a) g <=} 2(t) = tan(a)r- TWreosaye? )} Observaciones 5.2.1 1) Note que 2!(x) = 0 ==> tan(a) = are’. lo que implica x = “sen(a) cos(a) = valor donde el proyectil alcanza su altura maxima. 2) v2 2 ot) =0 43 2=0 0 a= elo tana) <= t=0 0 n= Micostadeen(e) — Henan). Este valor 1 = “tsen(2a) es el blanco del preyectil. 3) El proyectil recorre una distancia maxima cuando a = 3 y esta distancia es 42. Observe que esta distancia es proporcional al cuadrado de la velocidad inicial. 5.3. Vibraciones en Sistemas Mecanicos Aparecen cuando se perturba un sistema fisico en equilibrio que luego queda sujeto a fuerzas que tienden a restaurar el equilibrio. 1.- Vibraciones arménicas simples no amortiguadas. Consideremos un carro. de masa m sujeta por un muelle a un muro. Figura 27 El muelle no ejerce fuerza cuando el carro esti en su posicién de equilibrio, x = 0. Pero si se desplaza una distancia x. entonces el muelle ejerce una fuerza restauradora opuesta a la direccién del alargamiento y con una magnitud directamente propor cional al valor del alargamiento (Ley de Hooke): Fe=-kr,k>0. La constante & se llama constante de rigidez del muelle. Si aplicamos la Segunda Ley de Newton (Fuerza total = masa . aceleracién) obtenemos la ecuacidn diferencial nfiate a de La ecuacidn caracteristica es k 2 rte y luego la solucién general es x(t) = c, cos (\3 ‘ + esen (v=) > @QeER. Si en el instante inicial ¢ = 0, el carro se Ileva a la posicién x = Xp y desde alli se suelta sin velocidad inicial, tenemos las condiciones iniciales _ it ~~ dt y obtenemos ¢; = 2p y C2 = 0. Luego tenemos la solucién x(t) = 2 cos (VE. : 2O=m y¥ 0) (0) =0, cuyo grafico es Figura 28 Entonces la amplitud de la vibracién es x9. el perfodo (tiempo requerido para completar un ciclo) es T = 27 \/% y la frecuencia de la vibracién (ntimero de ciclos por unidad de tiempo) es f Observe que f crece cuando crece la rigidez & del muelle y cuando decrece la masa m del carro. En el caso que v(0) = vo > 0. tenemos c1 = aie ¥ a) = ye [-nn (ve) + 2 cos (vz ‘) Jo que implica \z Tr: ik im ik ~ no (Es) «wen (VE), que se puede escribir de la forma 2) = sn (yEras), donde A= \/23-+ Bo} es la amplitud, # = arctan (2/2) es el angulo de fase, fk vw = 2'(0) = ae = a= m Entonces x(t T= 2m os el periodo y f = ¥ os la frocuencia natural. 2.- Vibraciones amortiguadas. En este caso se agrega el efecto de una fuerza de amortiguamiento Fz, debida a la viscosidad del medio en que el carro se mueve (aire, agua, aceite, etc.), también opuesta a la direccién del alargamiento y con una magnitud directamente proporcional al valor del alargamiento: Fu=-c 2, c>0 c+ resistencia del medio. Tenemos entonces la ecuacién 2, mot Fo+ Fy. obién x, cde, k at ima tm = La ecuaci6n caracterfstica es ck = —=0, Pe op+ que tiene rafces A) Vibraciones Sobreamortiguadas. Corresponden al caso (55)? — 4 > 0 (es im decir, c > 2/km). Entonces p1,p2 son ntimeros negativos distintos y la solucién general es a(t) = ce! + ce, Bajo las condiciones iniciales (0) = a y u(0) = $7(0) = 0, se obtiene xo rat it i) = cP — poet"), x(t) — (pu poe’) cuyo gréfico es Figura 29 Observe que no hay vibracién y el carro tiende a restaurar su posicién de equi- librio, B) Vibraciones crfticamente amortiguadas.Bn este caso (6)? = & decir, c = 2Vkm). Aqui p. = p2 = —y/ y la solucién general es (es a(t) = (a + eateVit Al imponer las condiciones iniciales 2(0) = x9 y v(0) = 44(0) = 0. se obtiene a(t) = ae(l + de-vEt cuyo gréfico es del mismo tipo que el de la Figura 29. Luego no hay vibracién y el carro tiende a ir a su posicidn de equilibrio, C) Vibraciones Subamortiguadas. Ahora (35) — £ < 0 (es decir, c < 2Vkm). ‘Tenemos c fk cy? mer = am * Vin > Ga) * y la solucién general es L-(0)) = 0, se obtiene xo a(t) = “lacos(a t) + bsen(a t)] . Si ponemos ademas 6 = arctan (4) tenemos al) = VOTE Heos(at -6). cuyo grafico es r t Figura 30 Observe que la amplitud decrece exponencialmente. No es periddica, pero cruza Ja posicién de equilibrio x = 0 en intervalos regulares. Asf podemos considerar T= x, demas el mimero f = + = $. es llamado la frecuencia natural del 7 on sistema. 2.- Vibraciones forzadas. A las fuerzas anteriores agregamos una fuerza externa F. = f(t) que acttia sobre el carro, Esta se puede producir por vibraciones del muro © por un campo magnético externa. Tenemos x "ae Jo que implica x cde k de’ madt * im Un caso importante es cuando la fuerza externa es periddica f(t) = Fycos(wt). En ese caso la ecuacién diferencial es Ge cde k @tinatm Fy COsleut) - Como ya conocemos la solucién de la ecuacién homogénea correspondiente, pode- mos usar el método de los coeficientes indeterminados para encontrar una solucién particular. y con ello Ja solucién general de nuestra ecuacién, Entonces si iw no es rafz de la ecuacién caracteristica, buscamos una solucién del tipo tp(t) = Asen(wit) + Bcos(wi(t). Tenemos las derivadas a(t) = w(Acos(wt) — Bsen(w(t)), a(t) = —w?(Asen(wt) + B cos(w(t)) . y reemplazando en nuestra ecuacién multiplicada por m, obtenemos Fycos(wt) = k(Asen(wt) + Bcos(w(e)) + ew(A cos(wt) — Bsen(w(t)) —mu?(Asen(ut) + Bcos(w(t)) « Luego las constantes A y B deben verificar weAt(k-mu2)B = Fe (k-mu2)A—weB = 0. Por lo tanto Ac _ Falk = mu?) (k= mu?)? + ure? y p(t) = fy [wicsen(wit) + (k — mw?) cos(we)) « Ee muy ere ta): podemos escribir Fi p(t) = —=— —— 05(ut - 9). > Tamar re Asi por ejemplo, en el caso subamortiguado, la solucién general es Si ponemos $ = arctan (. ot [ cicos(a t) + cseu(a t) + EI primer sumando de esta expresin se Hama término transitorio ( tiende a cero cuando f se va para infinito) y el segundo sumando parte estacionaria (prevalece cuando el tiempo se hace grande), Por ello. se dice que la frecuencia de esta vibracién ; i es a y ane su amplitnd es pti Caso importante. Consideremos el caso anterior cuando la constante de amor- tiguacién c es nula y iw es rafz de la ecuacién caracterfstica (es decir w = \/E ). Tenemos entonces la ecuacisn diferencial fe ky Ro (Ey d2 "mm m )° La solucién general de la ecuacién homogénea es ap(t) = c.cos (z ‘ + cgsen (vE ‘ ; que se puede escribir de la forma ay(t) = Asen (VE Le) . — con A= q+ y ¢=arctan (2). Debemos buscar solucién particular de ecuacién no-homogénea de Ja forma p(t) = Cl eos (\2 ‘ + Disen (VE ‘ : Calculando la primera y segunda derivada de 2,(/) y reemplazando en la ecuacién obtenemos y por lo tanto Figura 31 De esta forma la solucién general es Observe que la curva 2 = 2;(¢). cuyo grafico es similar al de la Figura 28. presenta oscilaciones uniformes. Pero Ja solucién zp(), como lo muestra la Figura 31, oscila entre los valores + Se. y por lo tanto, su magnitud maxima tiende a infinito cuando ¢ tiende a infinito. Luego si en el sistema &x aE da. +cat ka = Fycos(ut). la constante de amortiguacién c es muy pequeiia. el sistema esta sujeto a grandes oscilaciones cuando la funcién de forzamiento tiene frecuencia (w) cereana a la fre- cuencia de resonancia del sistema (\/4) Estas grande vibraciones en resonancia son las que preocupan a los ingenieros. Se sabe que las vibraciones en resonancia ocasionan que las alas de los aviones se rompan, que los puentes se desplomen. etc. Fjemplo 5.3.1 Una masa que pesa 4 Ib. estira un resorte 3 pulgadas al Iegar all reposo en equilibrio. Se tira nego de la masa 6 pulgadas debajo del punto de equi« librio y se le aplica una velocidad de v2 pie/seg dirigida hacia abajo. Despreciando todas las fuerzas de amortiguacidn y externas que puedan estar presentes. determine la ecuacién del movimiento de la masa junto con su amplitud, periodo y frecuencia natural ; Cudnto tiempo transcurre desde que se suelta la masa hasta que pasa por la posicién de equilibrio? Como estamos en el caso de una vibracién simple no amortiguada, tenemos la ecuacin Fe Ar=o dP om cuya solucién general es o() 0. Tenemos L = 0.001, R = 0,02. C = 2, y E(t) = sen(1004). Reemplazando en (5.9) obtenemos 1 di se 0.00155 + 0.025 + 0,57 = 100¢0s(1004) . o bien 1 al = — + 5007 = 100, r Ns TE + 0 + 5007 100.000 cos(100 2) Como la ecuacién caracteristica J + 20k + 500 = 0 tiene rafces k = —10 + 202, Ja solucién general de Ja ecuacién homogénea es Ty(t) = e7 1c, cos(204) + cosen(208)] Usando el método de los coeficientes indeterminados, buscamos una solucién parti- cular de la forma I,(t) = Acos(1004) + Bsen(1001) . Reemplazando en Ia ecuacién no homogénea obtenemos 95, 20 “yap Boas Jo que implica I(t) = —95 cos(1004) + 20sen(1004)]. a 9.425 Entonces T(t) = 7 {eq cos (208) + casen(20t)] + ——<=[-95 cos(100£) + 20sen(1002)]. Du = Nuestras condiciones iniciales son I(0) = q(0) = 0. Para encontrar J'(0), se sustituyen los valores de L, R y C en (5.7) y se igualan ambos miembros para t = 0: (0,001)7'(0) +0.021(0) +0.59(0) =sen(0) => 1(0)=0. Obtenemos asi 95 0= 10) =a-s55 2.000 = 10) =H ‘ = PO) ==Wer +200 + SF Jo que implica ae 95 105 “om Y >“ TRR50 Por Io tanto 1) = eb ‘peso penn) -; = cos(1000) ++ Soe (1000)+ ra 5.5 Problemas resueltos Ejercicio 5.5.1 Un hombre se desplaza en direccién norte con velocidad constante de v metros por segundo. En el instante inicial lama a su perro que se encuentra a 100 metros al este de él. Si el perro corre con velocidad v2 = 2v1 ditigido a cada instante a su duefio, determine la trayectoria descrita por el perro y el tiempo en que tarda en alcanzar a su amo. Solucién. Supongamos que inicialmente el hombre se encuentra en el origen y que el perro se encuentra en el punto (100, 0). Sea P = (x.y) la posicién del perro en el instante ¢, En ese mismo instante el hombre se encuentra en H = (0. v;t). A P 100 Figura 33 Tenemos a sl Boy 4 a 7 v1 La longitud de la curva y = y(x) que describe la trayectoria del perro entre e] punto (100,0) y Pes 100 wy 100 _ tot = [ Vit yu? du => 5 ty) = | V1 + y'(u)? du ir in z Derivando con respecto a x y usando que vz = 2v4, nos queda 2A-ry") = -V1 + (y'P. Poniendo p = y/, que implica p/ = y/". y reemplazando obtenemos 2dp_—_ dev vitP 2 Al integrar se tiene 2m (p + VT + P) = n(x) + Inf) => pt VTP Hever. Como en x = 100, tenemos y = 0 y y' = 0, debemos tener c = 34, y Inega p+ VitP Por lo tanto e integrando * ; 3 . y(t) = 7h SS - 22) aw = A - mvt /, 10” Vu. 15 20, = a2 -1 200) ae OVE + De esta fi le esta forma 0) 200 w= 3 y el tiempo en que tarda en alcanzar a su amo es 2 200 segundos Bo, SSMS. Fjercicio 5.5.2 Un cuerpo de 8 libras de peso cae desde el reposo hacia Ia tierra desde gran altura. A medida que cae, la resistencia del aire acttia sobre él. Suponga que esta resistencia en libras, equivale numéricamente al doble de la velocidad, en pies por segundos. Determine la velocidad y la distancia de cafda en el tiempo 1. Considere g = 32(pies/seg2). Analice las funciones resultantes en el caso que Lo. Solucién. Por la Segunda Ley de Newton se tiene me +h, dt donde ma bo 8 “gg 327 4 F, = fuerza de empuje = mg = 8, y F; fuerza de resistencia = -2v. Por Io tanto tenemos la ecuacién 1 dv =8- 2 va 787 ee Separando variables y multiplicando por 4 obtenemos dv T= Sdt, © integrando —Inj4—e| = 8 — Inc) = 4-0 = ce. Como v(0) = 0, debemos tener ¢=4 y luego v(t) = 4(1—e7*), . : . * da Para conocer la distancia recorrida en el tiempo t, pongamos v = con (0) = a 0. Obtenemos asf la ecuacidn dx =4 (1-6) dt cuya solucin es St 1 xo) =4 [+S +a yeomo 2(0)=0 = a=-5 x(t) = Conclusiones, a) Como u(t) = 4 (1 - e~®). se tiene que Jim 0) =4. 1 . . b) Como a(t) = 4 |é+ x — 3) - se tiene que w > 00 si b> 90. Esto no implica que el movimiento seré.eterno, Al llegar a la superficie esta solucién no funciona. Ejercicio 5.5.3 Una barcaza esti siendo remolcada a 16 pies/seg . Cuando se revientia la cuerda que tira de ella, a partir de ese momento, contintia su movimiento en Iinea recta, pero frendndose con una velocidad proporcional a la raiz cuadrada de su velocidad instantanea. Si después de dos minutos de que se revienta la cuerda se observa que la velocidad de la barcaza es de 9 pies/seg , ;, qué distancia recorrerA antes de quedar en reposo? Solucidn. Se tiene ad a0) = 0. FZ = WAVE con las condiciones) u(0) = 16 pies/seg . “ v(120) = 9 pies/seg. Tntegrando la ecuacién se obtiene ae = -kt ta. Como 2(0) = 16, debemos tener cy = 8, y por lo tanto Pero 0120) =9 = 3=-00k+4 = bed. y luego t - TH dx t\? Bay (1 - in) . ¢ integrando 120 ty t\* rit) = "2 (: - ih) +a = -W0(4- 3) +e. La condicién x(0) = 0 implica cy = 40-4° = 2.560, y tenemos De esta forma t\* - a(t) = -40 (1 - in) + 2.560. La bareaza estara en reposo-cuando 4 = 0, Iuego esto sucederé alos ¢ = 480 segundos, Ademés como 3 x(480) = —40 (1 - =) + 2.560 = 2.560 pies, Ia distancia que recorre antes de quedar en reposo es de 2.560 pies. Ejercicio 5.5.4 Una particula de masa m se desliza hacia abajo sobre un plano inclinado bajo la influencia de la gravedad. Si al movimiento se opone una fuerza f = kmv? y es el Angulo de inclinacién del plano, encontrar: a) La velocidad de la partfcula en funcidn del tiempo, suponiendo que parte del Teposo. b) El desplazamiento en funcién del tiempo si (0) = 0. c) El tiempo necesario para que se desplace una distancia d desde el reposo. Solucién. La ecuacién del movimiento es dv 2 mo = mgsen() — kme?. ae = geen) a) Separando variables obtenemos dy q=e8 kdt. donde A= Vi Jeon(9) 6 Jo que es Io mismo dy dy vA Uta Como u(0) = 0, integrando obtenemos —2Akdt. In(v(t) = A) = In(=A) = In(v + A) + In(A) = - 2k AL, y exponenciando w+ © = Despejando y reemplazando el valor de A se tiene gy vO 1 + e2Vv basen} A axa oA = VE tanh(\/kgsen(®) ) . b) Integrando la expresién anterior y usando que «(0) = 0 obtenemos (Gisem(o) s ‘ 1H a(t) = Veo i | eee ‘ 89 eV * - /9 2 gksen(e ) = feuon | + Sag, Pees i+ 1 1+ ey skeen} = [geen —— ni *+— _)) * Taree 2 p~2y/gksen(6) t = Po ae t+ =) 2 (Teen Lan(eVIRmO") + In l+e . )) =2,/ghoen(Ght arene Lt € In(e —— ) n( Ht * eV IRA mire mre le In(cosh| _— Q). c) Sia=d Jintcosn( Yan 1) Jo que implica arecos h(e!4) ‘gksen(0) © Ejercicio 5.5.5 Una pelota de 6 onzas de peso se lanza hacia arriba desde una altura de 7 pies con uma velocidad inicial de 84 pies por segundo, Si la pelota est sujeta a una resistencia del aire igual a 3/128 (libras por segundo) de la velocidad de ella (pies por segundo) ; Que altura maxima alcanzaré antes de regresar a la tierra? Datos: 1 libra = 16 onzas, g = 32 pies por segundo al cuadrado, Solucién. Tenemos la ecuacién du 3 Oe mg — = 7. 0(0) = 84. ma mg — Tages #0) = 7. (0) = 84 donde 6 3 301 3 mg = Gomas = =Ib = Ib = m= 5-55 = se Reemplazando obtenemos 3 dv 3.3 dug mea 78 7 pst yt = 782, cuya solucién es u(t) = eS? | [ 22a + q = ot) = [16 + => u(t) = -16 + ce. Como v(0) = 84 tenemos = -16+ce¢ = c= 100. y entonces v(t) = -16 + 100e~*". Para calcular el tiempo fy que se demora en alcanzar la altura maxima debemos resolver ultc) = —16 + 100e" = 0. Por lo tanto 16 to = 4 4 2 joo = oR Ao = (3) = 2In (2) = h= -n(2 = (3). x(t) = 2(0) + [ u(s)ds = 7- 16t—50(e-* — 1) = 57-16t — 50e~**, 0 También tenemos y luego la altura maxima es o(tn (3)) = o7= 1610 ( 3 3 = 49-16-n( Ejercicio 5.5.6 Una masa de 4 libras se suspende de un resorte ocasionando un estiramiento de 2 pie. En el instante ¢ = 0. sin velocidad inicial la masa se desplaza 0.5 pie sobre su posicién de equilibrio y se suelta, En el mismo instante se aplica una fuerza externa equivalente a f(t) = Ssen(¢) libras. Suponiendo que no hay resistencia del aire, encontrar la ecuacién del movimiento resultante y la posicién del objeto al eabo de ¥ segundos. Considere g = 32,255. 3) = 34,3393 pies. Solucidn. La constante de rigidez k del resorte verifica 2k = mg =4 es decir a" +160 = G4sen(), (0) Aplicando transformada de Laplace y poniendo £(x(2))(s) = X(s) obtenemos 1 SX(s)+ 38+ 1GX(s) = Esto implica 64 1 _ *8) = Gypsy? 16 4 15 s? + 4? — eye Por lo tanto G4 on(t) — *Bsenat) — 4 cos(at) 5 5 3 x(t) Luego » : a 5,1 _ ae 0(G) = FRV2+ 5 = 3.510999. y el objeto esta abajo de la posicién de equilibrio, Ejercicio 5.5.7 Una masa de 2 kg, se sujeta a un resorte suspendido del techo. Esto ocasiona que el resorte se estire 4m, al Hegar al reposo en equilibrio. En el instante ¢ = 0, Ia masa se desplaza 1m, hacia abajo, y se suelta, En el mismo instante se aplica una fuerza externa f(t) = 4cos(é)newton al sistema. Si la constante de amortiguacién es 6 newton seg/m , determine el desplazamiento 2(¢) de la masa en un instante ¢ > 0 cualquiera, Considere g = 9.8m/seg”, Solucién. Tenemos m= 2Kg y estiramiento $m. Luego la constante f del resorte es 125 _ 125 = 2-98-22 = =. ka 29 196 10 De esta forma nuestra ecuacidn diferencial es 125 195 a" + 3a! + So0 = Fe col con las condiciones iniciales La ecuacién caracteristica es cuyas rafees son ‘ A= -- +21. 2 Luego Ia solucién general de Ja ecuacién homogénea es ant) = e~¥ [ey cos(2d) + egsen(2t)). Para encontrar una solucién particular de la no homogénea, usando el método de Jos coeficientes indeterminados, buscamos una solucién de Ja forma a(t) = Acos(t) + Bsen(t). Entonces a, (t) a(t) y reemplazando en la ecuacién obtenemos —Asen(t) + Beos(t), =Acos(t) — Been(t). 195 =e 5 cost) + B34 4 23 , ql) = [A+ 3B +S PAleos(t) + [-B-3A + FP Blsen(l). ‘Tenemos entonces el sistema 4A+3B = 18 { -3444B = 6 cuya solucién es 1 A=1, B= z Por lo tanto la solucién general de nuestra ecuacién es a(t) = e7# [ey cos(2t) + cysen(2t)] + cos(t) + Foen(t). Las condiciones x(0) = 1 y «!(0) = 0 implican respectivamente 3004 aqtl=l1 y tear sats = 0. Luego y nuestra solucién es en(2t) + cos(t) + feentt Capitulo 6 Soluciones en Serie de Potencias 6.1 Recuerdos de Series de Potencias Una serie de potencias en un punto ro es una expresidn de la forma SPan(e— a6)" = 9 4ax( — 20) + a2(a— m0)? +0 G4) n=O donde a es Ia variables y los coeficientes aj; son constantes, Se dice que (6.1) converge (esp. converge absolutamente) en x = r si la serie infinita (de ntimeros reales) ante =a)" (resp. SO ante = 20) |") n=0 converge: es decir, si el Iimite de la sumas parciales N Veanlr = 20)" (resp. 7 | an(r = 20) |") n=0 cuando N tiende a infinito existe ( pertenece a R). Si este limite no existe, se dice que la serie de potencias (6.1) diverge en x =r. El siguiente resultado, cuya demostracién fue hecha en los cursos de Célculo, determina para que valores de x la serie de potencia (6.1) converge. Teorema 6.1.1 1. Para cada serie de potencias de la forma (6.1), existe un niimero R. 0 < R R- 2. Si . dnt lim |" |= 7 nsx | Gn entonces ) O 0, podemos definir f(@) =YCan(a— 20)" Vay -R x 1 y= ‘seri ica) . =) > a7 (erie arminica) que es divergente, Para a = nos we Ia serie yr (serie arménica alternada) , n mt G ant que es convergente. Luego el conjunto de convergencia de esta serie de potencias es el intervalo 8, $ Ejemplo 6.1.3 Consideremos las serie de potencias ne Tenemos L = 1 y por lo tanto su radio de convergencia es R = 1. Como claramente esta serie no converge para x = +1, su conjunto de convergencia es el intervalo J-1.1[, Ademés VY -1 0. 1 Ejemplo 6.2.1 La funcién f(x) = — es analitica en ] - 1.1. Por (6.1.3) sabemos que es analitica en 2 = 0. Si xo € J-L.lf, 1 1 1 T-* (-m-@-m ~ T-a1- 1S = om Sea" n=0 y el radio de convergencia de esta serie de potencias es R =| 1— 2x0 |> 0. Ejemplo 6.2.2 Mas en general si una serie de potencias S7%p an(a — ao)" tiene radio de convergencia R-> 0, entonces la funcién f(a) = Ssy2¢an(w = 29)" es analftica en toda x €]ae — Ryo + Ri. En efecto si a1 € }t) — Ro + Ri, f esinfinitamente diferenciable en x = x1 y entonces so F(x) = 2 bn(a = 21)" n=0 con by, = Aen, Ejemplo 6.2.3 Todo polinomio f(x) = Ap + Aiw+++++ Anz”, es analitico en todo punto zp de R. Ejemplo 6.2.4 Toda funcidn racional f(x) = £2 (donde P(x). Q(x) son poli- nomios), es analitica en todo zp tal que Q(r9) #0. Ejemplo 6.2.5 Las siguientes funciones son analiticas en todo R: = tat ete = Dye sen() = = Do ee Sf _ _ oc (=1)" on cos(r) = = Dr Gore y las respectivas seri nen radio de convergencia R = oc. Ejemplo 6.2.6 La funcién ee ee = €s analitica para todo ag > 0 y el radio de convergencia de Ia serie de potencias es R=1, 6.3 Solucién en torno a puntos ordinarios Consideremos la ecuacién homogénea ag(a)y" + ay(x)y! + ag(a)y = 0, (6.2) en su forma normal y" + pla)y! + a(a)y = 0. (6.3) Definicién 6.3.1 Un punto xe se dice un punto ordinario de (6.2) 0 de (6.3) si las funciones p= % y q = % son analiticas en 9. Si xo no es ordinario para (6.2) se dice que xo ¢s un punto singular de-(6.2). Ejemplo 6.3.2 Considere la ecuacién ay! +— I+ se = zou + senla)y =0. Entonces p(x) = zig es analitica en todo zo # 1. Por otra parte as @) - M@lleceeeos Ly eye nr z = 3 Hl y como esta serie tiene radio de convergencia R = oc. q(x) es analitica en todo meR Asf el tinico punto singular de nuestra ecuacién es 9 = 1. El siguiente teorema nos dice como resolver (6.2) alrededor de un punto ordinario. Teorema 6.3.3 Si 1 cs punto ordinario de (6.2), entonces (6.2) tiene dos solu- ciones analiticas linealmente independientes de la forma y(z) = Sane = 0)" nat con radio de convergencia mayor o igual que la distancia de ae al punto singular (real 0 complejo) de (6.2) mds cercano a xo. Ejemplo 6.3.4 Encuentre la solucién general de yy =0 alrededor de aq = 0. Primero notemos que como nuestra ecuacién no tiene puntos singulares, las solu- ciones que encontremes nsando el teorema (6.8.3) estarsn definidas para todo « € R, Sea (x) = S3%panx". Debemos determinar los valores de los coeficientes a, para que Op sea solucién de nuestra ecuacién. Sus derivadas son: $2) = Soraya”! yee) = onl Vag”. nat n=2 Reemplazando en la ecuacién obtenemos oc x SE n(n = Vana”? = SP aya! = 0. ne n= Dejando ambas sumatorias con-término general .c” tenemos iMe (n +:2)(n + Vany22" ~ Yes" = =0. n y juntando las sumatorias 2az + $7 |(n + 2)(n+ Vany2 = Ana)" = 0. nai Por lo tanto debemos tener @m=0 y (n+2)(n+ Lany2-dp-1=0 Vn>1 Jo que implica Ant (n+2)(n+1) Para darnos cuenta como es la expresién del término general an, Jo calculamos para los primeros valores de n, Tenemos para n = 1.2 y 3 respectivamente a@=0 y dhiz= vn >1. ag = Esto nos permite darnos cuenta que para todo n > 1 an = BGO Asn = BAG Gants = 0, con ag ¥ ay arbitrarios. Luego si definimos pila) = pln) = Bnyan+ 1)’ como $i(2) y $o(x) son LL. ( W(¢1,42)(0) = 1) la solucién general de nuestra ecuacidn es (2) = appre) + ardr(x) , a.m €R y estd definida para todo x € R. 6.4 Ecuacién de Legendre y Polinomios de Legen- dre En ésta seccién estudiaremos las soluciones de la ecuacién de Legendre L(y) = (1 = 2?)y" = 2ay' + a(a+ Dy = donde @ es una constante real, alrededor del punto « = 0. Como las funciones 2a . _ a(a+1) Toe Y W= 7 [3 P(t) = son analiticas en todo « # +1, los puntos singulares de la ecuacién de Legendre son precisamente z = +1, Por lo tanto el punto x = 0 es ordinario y las soluciones (2) = TX gan", estaran definidas por lo menos para | x |< 1. Para este $(1r) tenemos #2) = Sonat! y 9a) = Luego x =2x¢! (x) = 37 -2nane" y n=0 (n+2)(n+ Vang" y —2?d"(a) == n=O n— lana”. Ma oe) = Reemplazando en la ecuacién de Legendre obtenemos (n+2)(m+ Vany2 = n(n = Dan = 2ay + (0 + Lay)]2” = = H Ma (n+ 2)(n4+ lanyst+(a+n4+1)(a-njayj2™” = 0. H Ma Por Io tanto, debemos tener para todo n > 0 (n+2)(n+ Danyot(a+n+1j(a-nja, = 0 Jo que implica asx _(a+ntV(a=n) me nF Din +1) Para n = 0.1.2,3 y 4 obtenemos respectivamente a. a= a3 = (at3)(a-2 - ays Sag = (a+4)(a-3 = as = NE ag _ (o+5)(a-+8)(a+1)a(a=2)(a—4) CRE ao « (a+5)(a=4) ag = Oy A partir de estos valores se puede demostrar por induccién que para todo n <1 yn (@#2n= lo + 2n=3)-++ (a+ Nala =2) ++ (a = In +2) 2, = (-1)" nyt a; _ pyr Ot 2n)(a + 2n = 2) + (a+ 2)(a = D(a = 3) ++ (a= 2n +1) ana = (Cp ES ee Definamos alr) = 1+ Spat 2n— Ila +2n—3) (at Nala —2)++(a—2+2) 9» yo yl en, oa) = at Qn+D! . nt Entonces ¢1(x) y ia(a) estan definidas por lo menos para | x |< 1 y son linealmente independientes (W(¢1. #2)(0) = 1). Luego la solucién general de la ecuacién de Legendre es g(t) = apdr(z) + argo(x). ao.a, ER. Observaciones 6.4.1. 1. Si a = 2m con m un ntimero natural, entonces ala) = 1+ jn(@+2n= Ila $2n—3) (a+ Nala=2++(A=I2N+2) on yen Qn} us nat es un polinomio de grado 2m que contiene solo potencias pares de x. Por su parte 2(.r) es una serie infinita de potencias. Por ejemplo: a=0 = g(2)= a=2 = Ale eae, ) a=4 = dy(0) = 1-100" 4B, 2. Sia =2m+1 con m un ntimero natural, entonces 44(z) es una serie infinita de potencias. pero ahora $2(x)es un polinomio de grado 2m + 1 que contiene solo potencias impares de x. Por ejemplo: a=1 > &(r)= a=3 => (x)= 14 21 a=5 => a) =a - Fa + Sa 3, Luego podemos concluir que para todo mimero natural m. existe un polinomio de grado m que es solucién de la ecuacién de Legendre (=2?)y" = 2ry/+m(m+1y = 0. Definicién 6.4.2 Sea n un neimero natural, Se lama enésimo polinomio de Legendre al polinomio P(x) de grado n que es solucién de la ecuacién de Legendre (l-2?)y" 2a +n(n+Dy = 0, y verifica P,(1) = 1. La siguiente férmula, que demostraremos como ejercicio, nos dé una definicién alternativa de los polinomios de Legendre. Ejercicio 6.4.3 Para todo ntimero natural n se tiene 1 do. Plt) = oat — " Demostracién. Fijado n consideremos el polinomio u(x) = (x? — 1)". ‘Tenemos entonces w(x) =n(a2=1)""!2n = (a? = 1ul(w) — 2nvu(x) = 0. Derivando consecutivamente esta tiltima expresién k veces se obtiene Lul"(x) + 2ru! (x) = 2neul(x) = Inu(x) = 0 Lul"(x) + dru" (x) + Qu'(aw) = Qnzul"(x) — Anu'(x) = 0 ( (a? = Du! (a) + ark (x) + k(k = Lu (x) — 2nzu (x) — InkulE(z) = 0 Ponienda & = n+1 en la dltima ecuacidén se tiene (x? = Dul™*?) (x) + 20(n + Du (2) + (n+ rw (x) = 2ncul"+Y (x) = In(n + Du” (x) = 0. y agrupando términos (2? = 1) (a) + 20u!™P(x) = n(n + Iu (a) = Poniendo gn («) = u(x) y multiplicando por —1 obtenemos (= 2)gl(ar) = 2g (2) + n(n+ Nan() = 0. Luego el polinomio gn(x), que es de grado n. es solucién de la ecuacién de Legendre (L-a?)y"— 20 +n(nt+ Dy = 0. Ademés a(t) = [a - yr” = (@ = 1)"(@ + 1)" (@-—)" +1)" + téminos que contienen a x — 1 como factor = ni(x+1)" + términos que contienen a x= 1 como factor. Por Jo tanto an(1) =n! y asi P 1 » - 12a Pr) = goatee —)"- Los siguientes tres ejercicios se dejan para ser resueltos por el lector. Ejercicio 6.4.4 Usando la formula anterior compruebe que Pha)= 1, P(x) =x, Pala) = 822-4, P(x) = $29 — 2x, Ejercicio 6.4.5 Sea f(x) una funcién n veces continuamente diferenciable en el intervalo [-1, 1’ y sea : [ferrari L, Integrando por partes demuestre que = oe fire (a)(a? — 1)"d Ejercicio 6.4.6 Demuestre que 2 1 2, 2n+1(g) - 2n lg [ow (9)db = mr1 (8)sen(4) = 4 cos El siguiente ejercicio nos dice que la familia de los polinomios de Legendre es ortogonal. Ejercicio 6.4.7 Pruebe que a ' 0. si m#n: I, Pte eye = { omer Demostracién. tants ejercicio- (6.4.5) con f(x) = P(x) y m 0 considere la ecuacién de Bessel de orden p wy! + ay + (=p jy = 0. Claramente su tinico punto singular es x = 0. Como co Qt = 7 2 mally PoP app son analiticas en x = 0, el punto « = 0 es punto singular regular de la ecuacién de Bessel. Ejemplo 6.5.4 También se puede comprobar que x = 0 es punto singular regular de la ecuacién de Euler Py! + pay! + ay = © Observaciones 6.5.5 Suponga que 9 es punto singular regular de la ecuacién y" +p(a)y' +a(x)y = 0. Entonces multiplicando la ecuacién por (2 — 179)? nos queda Ia ecuacién (x = 2¢)?y" + (w — 20) P(a)y' + Qla)y = 0, donde las funciones P(x) = (x — 1¢)p(x) y Q(x) = (x — x9)2q(x) son analfticas en T= 1% 6.6 Método de Frobenius En esta seccién desarrollaremos el método de Frobenius para resolver una ecuacién homogénea de segundo orden alrededor de un punto singular regular. Primero que nada observemos que las soluciones pueden no estar definidas en el punto singular, ya que la misma ecuaci6n no est definida alli. Note ademés que podemos restringimos al caso en que el punto singular regular es x = 0, haciendo una traslacién en la variable independiente si es necesario. y en que la ecuacidn es de la forma L(y) = 2°y!" + xP(a)y’ + Q(a)y = 0, con P(x) y Q(x) analiticas en x = 0. De esta forma ns nx = age y Qa) = Soha = = y las series son convergentes para | x |< R. R > 0. Buscaremos soluciones ¢ de la forma c a(x) = 2" Sea con om #0 is definidas inicialmente solo para « > 0, « < R, Tenemos x c Skt reat y p(x) = Bk 4 r(k+r— Vaart. c=0 k=0 2 S x & H Ge >a) ( Sat) i con fy = Loins : (kt r)eqa* (Sa ) Il a Mey a 8 2 = qe = & (se k=0 x k = OSA ye* con d= G+ rea; y k=0 j=0 26"(2) = wy (k+r)(k+r—Vexa*. k= Reemplazando en la ecuacién obtenemos L(p)(2.1) =r (kt r)(k+r— lex + Gx + Ayle® = 0. ix Entonces [ je = har h+rVer tan + he = 0, VR=O12e, Pero k k [ok = &4rhtr-Ya + VG + roan + Votes, jae 5 kel = [(ktn(k+r—-1) +b +r1)09 4 hola + SG + rons + Boies a rth + de, donde ket ar) = rr —V+oort Be. y de = SOG +r) 0n) + hej j=0 El polinomio cuadrético g(r) = r(r = 1) star +o es llamado ol polinomio indicial asociado a la ecuacién. Para k = 0 [ 5 =a)a = 0 y Iuego los tinicos valores que pueden admitirse para r son Ia raices de q. Ob- serve ademas que dj, es combinacién lineal de ¢¢, 15+ .Cka1 ¥ que si estos ya estén calculados. cj también puede calcularse por la formula _ -dk(r) ~ qr +k) cx(r) siy sdlo sf g(r +k) #0. Entonces bajo la condicién g(r + &) 4 0 para todo k > 1, tados los ¢(r) pueden calcularse recursivamente de modo que si la serie x la,r) = cox? +a" Yr eg(r)a® i converge para 0 1 esrafzdeq. Sean 1,79 las raices de q y supongamos 1,12 € R. 7, > 73. Entonces di+k) 40 Weel, Jo que implica que si definimos los cj (r1) por Ja formula —ak(r1) —S_ veel, an+hy ol) =l y atn)= tenemos que n(z) = 21 + Dain x") es solucidn si la serie converge. Si rp es distinto de ry y si se verifica que q(r2 +k) #0 para todo k > 1, entonces poniendo también (2) =1 y eg(r2) = Vk>1, a(r2 +k) resulta que ho) = (1+ Yatra *) es otra solucién si la serie converge, EI siguiente teorema nos garantiza que estas series convergen para | a: |< r, que Jas soluciones #1(z) y 2(x) estén definidas también para —r < x < 0 (poniendo | x |" en lugar de x") y que son linealmente independientes. Antes una pequefia observacién. Observaciones 6.6.1 arth) #0 Vk>1 = néryth Vk>1 <=} r—r2 noes un entero positive. Teorema 6.6.2 Consideremos la ecuacién xy" +xa(x)y +b(a)y = 0, donde a(x),b(x) tienen desarrollo en serie de potencias (centradas en x = 0) con- vergentes para | x |< pg. py > 0. Suponga que las raices 11.12 del polmomio indicial ar) = r(r 1) +a(0)r +600), son reales y distintas, digamos ry > ro. Entonces. para 0 <| x |< po. existe solucién $4(x) de la forma dla) =|2 |" 1+ Sexe’), =I donde la serie converge para | x |< py. Ademds sir; =r no es un entero positivo. existe una segunda solucién dala) =|"? (1+ > ee") =I tal que la serie converge para | x |< po. Tales soluciones son linealmente independientes para 0 <| 1 |< py, y los coeficientes cx y & pueden obtenerse sustituyendo en la ecuacién. Para desarrollar el siguiente ejemplo conviene tener en cuenta la siguiente iden- tidad que puede ser demostrada por induccién Qk+D! 1365-76 Qk+1) = Soy (6.5) Ejemplo 6.6.3 Consideremos la ecuacin L(y) = = ryt ey! +a2y = 0. Su polinomio indicial es - 3. = pal ek 1 ar) = r(r Y+5r = rtgr os (r+ 5). cuyas rafces son ry = 0 y 72 = 4. Busquemos una solucién de la forma a) = 2" Sqa*. = Entonces x rg(t) = a Eg ceuh tt av SV oqask, 1 3142) = aw OX (kt rjeqe® = 1 brey 42! 23k treat, 5 i=0) z 3 23 } vg"(x) = al DE lk t+ rk+r Veet = a'r(r— Veco + US kar (kA = Vena’. he Por Io tanto 20 L(p)(x) = cea" +2" Sar thx tea", it Jo que implica que para todo k > 1 g = os ** Wh = gq = — ** dr+Rar +k-1) = a= (=1)keo En r =r, = 0 tenemos g(r; + k) = g(h) = k(k + 4), lo que implica que, tomando co(r1) = 1, para todo k > 1, se tiene an) = 5 1+3.2- (<1) 24 a, 2 1 35.7 Qk+) (<1) 28 9 kt = Foor (usando (6.5)) (i Qk+1)! Por su parte, en r = rz = —# tenemos q(rz +h) = q(—4 +h) = (k= })k. Lego si ©(r2) = 1, para todo k > 1, se tiene cx(t2) = (=I) 2 2k! (k=! FlQk— 1! (Ik 2k hr” (usando (6.5)) De esta forma y la solucidn general es d(x) = agi(x) + bf2(z), abER. Finalmente observe que como las funciones a(x) = $ y (x) = a tienen desarrollo en serie de potencias convergentes para todo x € R, nuestra solucién general esta definida para todo x € R.x #0. Casos excepcionales. Llamemos $1 (z) la solucién encontrada para r = 1 segtin el pracedimiento anterior. ‘Veremos aqui como se encuentra una segunda solucién ¢9(z), linealmente indepen- diente con ¢;(x) en los casos 7 = T2 y 71 — T2 igual a un entero positivo. 1) n=r En este caso a(n) = 7 (1) =0. Observe que existe 6 > 0 tal que sir € Jr: — 6.71 + 6[ entonces g(r + k) # 0 para todo k > 1, Luego para estos r’s y cada k > 1 existen coeficientes c, = ¢;(r) tales que si entonces L)(a.r) = a'q(r). También para todo r € (ry — 0.7, +0), tenemos 1(2) car) = Zeceyeer) = te) + ieee => 1(3) (er) = Luego wala) = (nn) x x = oS (rye") + In(w)a" + Sa(r)a*) ia hal = = a Sk(ri)a®) + In(x)dr(e). a es solucién de nuestra ecuacién diferencial. 2) ry = rg +m con m un entero positivo. En este caso tenemos que ar) = (rn) = 72), y poniendo c¢(r) — 1 y obtenemos di(r2) = aa(r2) Amai(t2) = Cm=i(r2) =0 y dm(r2) = 0 : Luego + = (0 = 12)dm(r) int) =(F=12)dm(P)¥- Galt) = I) _ dm(r) = (= T2)dm(0) Ym = Cart maT) y por lo tanto . . =d,,(7) m(T2) = lim ¢,(r) = =, Cmn(t2) = Tim em(7) m De esta forma todos los cx(r'2) y los ¢,(r) con 7 cercano a rz pueden calcularse de modo que si x Yer) con o(r)= k War) = 2 entonces L(y(a.7}) = co(r)a(r)a” = (r = re) q(r)a". Pero como ¢9(r2) = c1(r2) = +++ = ¢m-1(72) = 0. no es dificil de demostrar que la solucién W(x) = V(a.r2) = 2%a"o(x) = x"o(x) es linealmente dependiente con ¢i(z). Para encontrar una solucién linealmente independiente con ¢1(z). derivamos L(v)(x.r) = (r — r2)q(r)x" con respecto ar, obteniéndose £(S2) ter) = Preyer) = alr + (r= raidte) + nant Por lo tanto = 8 asa) or c c gaa) = 2" Sch(ra)x* + In(a)2" $7 ex(ro)a*, es decir k= k=0 dala) = aS ehlra)a* + cln(e)di(a). = para cierta constante ¢ € R ( ¢ podré ser cero). Resumiendo tenemos el siguiente resultado: Teorema 6.6.4 Consideremos la ecuacién wy" +2a(r)y +b(x)y = 0, donde las funciones a(x) y b(x) tienen desarrollo en series de potencias (centradas en cero) convergentes para | x |< py. po > 0. Suponga que las ratces ry.r2 del polinomio indicial ar) = r(r—1)+a(0)r +.8(0) son reales y verifican Tr > 12. Siri = 12, existen dos soluciones (2), (x) definidas y linealmente independientes para 0 <| x |< po, las cuales son de la siguiente forma wa(r) |x|" ox). x(x) = |x|" o2(x) +In(| x |)pilz). donde o1(x) y ¢2(x) son series de potencias (centradas en cero y convergentes para | 2 |< po) con o4(0) 40. Sit, — 12 es un entero positivo, existen dos soluciones $1(x).$2(x) definidas y li- nealmente independientes para 0 <| x |< pe, las cuales son de la siguiente forma a(t) = |x|" oz), go(x) = |x|? o2(x)+cln(| x |)pr(z), donde o1(x) y o2(x) son series de potencias (centradas en cero y convergentes para | 2 |< po) con o1(0) #0 4 02(0) yc € R es una constante (que puede ser cero). Finalmente si las rafces del potinomio indicial son complejas, digamos 7; = a+ 13, 7) =a—713, 8 £0, evisten soluciones linealmente independientes de la forma g1(2) |. |* cos(In{| a: |) )(1 + $7 egar®) ist pola) = |x | sen(Bln(| x |) ae Saal, Ejemplo 6.6.5 Considere la ecuacién Ly) = ay!" +3ay'+(+a)y = 0. Tenemos a(x) = 3 y B(x) = 1+. Luego el polinomio indicial es ar) = r(r-1)4+3r4+1 = (+1). cuyas rafces son ry = ry = 1. Sea (2) = o(a,7) = 2" rega®. i= Por Io tanto = & U1 a Me att! = a Saath ‘ ke B(k+r)qa*. Ppa) = x LV (k+r(k+r-Ver*. 7 Reemplazando en la ecuacién obtenemos L(9)(a.r) = egx’a(r) + S0 alr + bey + cyaile*. i= Para todo r cercano a ri = —1. g(r +k) #0 para todo k > 1, Luego poniendo aCe = TH obtenemos L(¢)(@,r) = eox"alr). Para estos t's, poniendo co(r) = 1. dela formula de recurrencia de los ck se puede deducir w= (<1)! (

0, consideremos Ja ecuacién de Bessel de orden p xy" +ay'+(2?-pPjy = 0. Se llama funcién de Bessel a toda solucién de esta ecuacién. Estas aparecen en problemas sobre vibraciones de una eadena colgante (Daniel Bernoulli), vibraciones de una membrana circular (Euler), movimiento de planctas (Bessel). propagacién de ondas. elasticidad, movimiento de fluidos, teoria del potencial, etc. Ya sabemos que esta ecuacin tiene a x = como tnico punto singular y que este es regular, Para resolverla alrededor de «x = 0 usando el método de Frobenious calculamos primeramente su polinomio indicial ar) =rr-lt+r-p cuyas rafces son ry = p y 12 =—p. Buscamos entonces un solucién (x) de Ja forma o (a) = 2" Peak, k=0 Tenemos entonces x =P ole) = 2!(-po- Pax + S> pep"). f Siege? = oc ap'(x) = 2 Sk +r qak = x" (req + (r+ Vert + Sk + ree"), k=0 k=2 Chaat, edn) = U(r = Veg + (r+ rea + Sk + rk +7 = Dex) iad Reemplazando en la ecuacidn de Bessel obtenemos c a" (ceq(r) + erg(r + Va + SO [g(r + bce + cx-s]n* = 0. i Asumiendo para r los valores +p. con p # 4, tenemos que q(r +1) es no nulo, Por Io tanto asumimos c; = 0, inclusive para p = $. Para anular los coeficientes correspondientes a k > 2 debemos tener ) — rek-2(r) =Chaa(r) =Ch-2(r) 00) = etm = GER > UERenG ea Los primeros cx (7°) son: y= ool) al) = TSF G=PFD clr) = 0 alr) = -alr) _ co(r) wy (+pt+Dr=pt4)— (+pt2Qr+p+Hr—p+ Der —p+4) e(r) = 0 clr) = =ca(r) ~ (r+p+6)\r—p+6) -alr) (r+ ptr +pt Hr +p FH = pt 2(r—p+ Ar =p +6) de lo que se deduce caalr) = Oy (=1)'co(r) T¥pt Deepa +p tyr — p+ D0 —p+a) = (r—p+ oh) Ok para todo k > 1. Luego para rr = 11 =p y todo k > 1 se tiene (=D¥co(n) (2p + 2)(2p+ 4) ( 0 est definida por la férmula T(q) =[ et 7 dt. 0 y tiene la siguientes propiedades lim Pq) = +00 (6.6) q+ md) = 1 (6.7) Ta@+1) = 409) (6.8) T(k+1) = &l para todok € ZW {0} (6.9) Para —1 < g < 0 tenemos 0 < q+1<1y por lo tanto [(q+ 1) est bien definido. Usando esto y (6.8) se define para—1 0. Para r = rz = —p y todo k > 1 se tiene (=D¥co(72) 2k(—2p + 2)(—2p + A) +++ (—2p + 2h) (=1)keo(r1) . PRM —p + 1)(=p + 2) +++ (=p +h) earls) = Para que todos los ¢ (72) estén definidos imponemos la condicién p ¢ Z*. Asf tenemos la solucién 4 (=1)keo(r1) 2k 7 wa) = 2° 3) appt Poniendo a (r2) = tf 2) =F (=p +1)’ obtenemos (=D PIP RT(k— p+)’ y la solucidn go(x) que se denota por J-p(2)'y que asume la forma Cxn(r2) = Dk (ER J-ole 123 anes p+) se denomina funcién de Bessel de primera clase de orden —p. Esta funcidn de Bessel Jp(s) esta definida para todo r €R, x >0. Ademés J,(0) = 0. lo que implica que J,(2) es acotada para x > 0 cercano a cero. Por otra parte el primer témino de Jip(x) es 2)-p } T(-p+1)’ Jo que implica que Jap(z) no es acotada para x > 0 cereano a cero. Luego bajo nuestra condicién: p no es cero ni un entero positivo, las soluciones J,() y J_p(x) son linealmente independientes en R* y la solucin general es Hx) = adv) + Jep(z). abeR. Caso p es un entero no negativo, Sea p > 0 un entero, Tenemos 2 (=I) (g)PRHP = oer El er =0 Observe que (6.12) implica lim |T(-g+k+1) |= +90 pata k=1.2....,p-1 Por lo tanto en Ia serie que define a J-(:r) los coeficientes correspondientes a k = O,1....,p—1 son cero. Ast Japa) = (=1)k+P (Z)%+9)-P (k+ py (k-+p—p)! Fy (=D! (ye “oe WR + Dy! = (-lP Joa). Por Io tanto Jp(:r) y J-p(x) son linealmente dependientes. Para encontrar una funcién de Bessel linealmente independiente con Jp{:x). definimos para q > 0 que no es un entero \ , \ ¥,(x) = 2 cos(gm) = Jg(t) | sen(q7) Esta funcién de Bessel es Hamada funcién de Bessel de segunda clase de orden @y es linealmente independiente con J,(:x) para todo q > 0 que no es un entero. Si en Ia fraccién que define a ¥,() ponemos q = p, tanto el numerador como el denominador se anulan, Usando L’Hopital se puede demostrar que ¥,(a) = limY,(x) op esta bien definida y que es una funcién de Bessel linealmente independiente con Jp(x). Por lo tanto, en este caso, la solucién general es g(t) = aJ,(x) + bY, (2), abeER. Ejercicios. 1) fle? s,(a)] = a? J-1(2) 2) Bley] = -2-F Jala) 3) M(x) + BAp(a) = Jp-a(2) 4) Sia) = 2Ap(2) = Spal) 5) Sha) = $[J-1(@) — Jnys() 6) Ip) = £[Jpar(x) + Jpsa(a) a . 4 . Demuestre que el cambio de coordenadas y = 277 transforma la ecuacidn de Bessel de orden p en Ja ecuacién & Demuestre que J, (x = Senta) a 9) Demuestre que J_y(x) = vz Soluciones. 1) Tenemos (yk gps ” Sa WTR + p+ 3, (ck eM D(k +p +1) wAp(e) = 4 a SS (ER Ep) ent We) = 0 Serpe (=1)k 22(b4P)=1-P poem py LD * RE = 2? Jpni (a). 2) Se demuestra de la misma forma que 1) y se deja de ejercicio. 3) De 1) tenemos WS a) + px Ip(x) = 2? Jpa(2). y nuestro resultado se obtiene multiplicando por .~?. 4) Se demuestra de la misma forma que 3) y se deja de ejercicio. 5) Se demuestra sumandbo las ecuaciones 3) y 4). 6) Se demuestra restando las ecuaciones 3) y 4). 7) Tenemos ya) = ya) = yx) = Por lo tanto -Py(2) = -pPa7tu(z), wy(x) = xtu(z), ry(2) = —3r tala) +axtul(c), y"(x) = Sa-tu(n) = rhe) +rtu"(2). Reemplazando en la ecuacién de Bessel se obtiene Fr-tu(n) —xhul() + 22u"(e) = da-bu(n) + ahul(a) + 22u(0) —pPamu(n) = 0. Reordenando Bula) + (or! + 2} ul(a) + Gat — ard tad px Hula) = 0, © Jo que es lo mismo ohul(a) + (0b + xa = 4p?)x“P)u(x) = 0, Finalmente multiplicando por x~? obtenemos ul(a) +(1+ ta —4p?)x~)u(x) = 0. . +4 . -. 8) Para p= 4 el cambio de coordenadas y = x~?w anterior nos da la ecuacién uw’ +u=0, cuya solucién general es u(x) = ey cos(x) + exsen(z). Por lo tanto las funciones : en(r) y ga(z) = 27# cos(x} Ala) = © son soluciones de nuestra ecuacién de Bessel de orden 4. Observe que como BO) = 0 = Bade) yp Lyla) = toe = Jin dala). tenemos que existe constante A tales que Jy(a) = Api(a). Ademés hogy 2 Se (EDR (2k+ 4) (PES Ko = Sara — vB (DF Ok + D DB y R(k+E Dest y ) 1 h(a) = Feos(x) — 52 2sen(x) = at(cos(e) — $2"tsen(2)) 9) Como y en el desarrollo en serie de potencias centrada en cero de la funcién sen(e) (resp. cos(x)) aparecen solo potencias impares de « (resp. solo potencias pares de «r) existe constante B tal que (t) = Bga(x). 1 3 Si Tamamos (=I) (2) N@) = 20 Te ix AMPR+5)? debemos tener Por lo tanto 6.8 Ecuacién Hipergeométrica de Gauss Para a,b,c constantes reales. se llama ecuacién hipergeométrica de Gauss a la ecuacién a(1—a)y" +[e— (@+b+ Daly’ - aby = 0. Supondremos desde el inicio que c # 0 y c#a+b+1. Bajo estas condiciones las funciones c= (atbt lr _ ab aP= : aaa) ¥ S@P= Fay solo no son analfticas en los puntos x = 0 y x = 1. Por lo tanto estos son los tinicos puntos singulares de nuestra ecuacién. Como las funciones Pla) = a(r) = 1P(2) = SEEHOE yy) = QM = son analfticas en x = 0, este es un punto singular regular. Ejercicio. El punto x = 1 es punto singular regular. Busquemos primero la solucién general alrededor de x = 0. El correspondiente polinomio indicial es alr) = (r= 1) +a(0)r +00) =r(r-1) ter = r= 140). Examinemos las condiciones que deben cumplirse para que tengamos una solucién asociada a r = 0, es decir, para que tengamos una solucién de la forma oa) = Saat. i= Estas son ry = 0 (es decir 0 > 1-¢ 3 ¢ > 1), 0m =0 (es decir e < I y 7-12 = 1—c no es un entero positivo (es decir € no es cero ni un entero negativo). Luego la condicién es c¥{0}UZ- (6.13) Bajo esta condicién y con (x) definido como arriba. tenemos 20 =abp(e) = S° —abeye i=0 Me -(a+b+Va¢'(x) = ha +b + Ueqa* g = H Mae chee! = SO c(k + Vega i= Hkh = Ucqar® \ 4 & H Me Mae K(k = Dege!! = Sk + hens. & = U1 Me Reemplazando en la ecuacién obtenemos c ((k+ Dk + c(k + leer + [= A(R 1) — (a +64 Dk —able)x* = 0. k=0 Luego para todo k > 0 debemos tener [(k+ Dk + clk + Dena + (-kk-D-(a+b+Dk-ablq = 0. © lo que es Jo mismo (kA (K+ 0) — (+ ak + bck = Esto implica _ (at k)b+h) ( ) = FT) a ) — @th(at+k-Yb+hb+k-1) (k+DMet+hyet+k—-) “tt (at h)atk—1)++-(a+Vabb+AG+k—1)-++ (4 1)b © Fike DCF EHR Der De Por lo tanto si ponemos cq = 1, tenemos para todo k > 1 = Wat Dat kA FD OFA) ~ Flce+ly(e+k—1) y nuestra solucién es reyes “@Q+k—-Dbb+1)-G+k-V a Tc(e+ ly (eh k= e ) Esta serie se conoce como la serie hipergeométrica y se denota por at+k=Vbb+1) + G+k-V ,, Ret) (e+k—-1) * atatl F(a.b,c,x) a) Observaciones 6.8.1. 1. Cuando a= 1 y b=c tenemos 2 F(1,b.b.2) = 1+ Soa! Por lo tanto para todo | a |< 1 y= F(1,b.b.2) = <= 2. Si ao b son un entero negativo.o,cero. la serie hipergeométrica se reduce aun polinomio, y por lo tanto converge para todo « € R. 3. Si ao b no son un entero negativo ni cero. la serie hipergeométrica converge slo para | x |< 1. 4, Luego para | x |< 1, F(a,b.c,x) es una funtidn analitica, que se denomina funcién hipergeométrica. 5, Tenemos F(a.b.c,x) = F(b.a, 0,2). Determinemos ahora bajo que condiciones tenemos una solucién asociada a la raiz 1 — c #0 del polinomio indicial. Esto sucede cuando 71 = 1 — ¢ (es decir, cuando 1 —c > 0 = ¢ < 1) 0 cuando ry = 1=c (es decir ¢ > 1) y 7) —7 = cC—1 no es um entero positivo ni cero (es decir, © no es un entero positivo). Luego la condicién es ceZt. (6.14) Luego asociada a la condicién (6.14) tenemos una solucién de la forma do(x) = 2 Sark, a #0. donde los @ pueden encontrarse sustituyendo en la ecuacién. Otra manera de encontrarlos, es hacer el cambio de coordenadas yous, obteniéndose yo = (l-oa*z + alee y! = -c(l—oa~ 2 + (1 — o)a~tz! + ate". Reemplazando en la ecuacién tenemos 0 = a(l—2)[—e(1 —c)a-e tz + 21 = o)amee! +a" + [e= (a+ b+ Ua = cjar*z + ate!) = abr! z x a(1 = x)2" + [x(L — 220 — ca“? + (c— (a+ b+ Var)a 2 + [=e —c)a(L = a)a7! + (1 = o)a“(e = (a +b + Vx) = abe! ]2. Multiplicando por x? se obtiene 0 x(1—a)2" + [{(1-c\(1-2) +e-(at+b+1ja2 + = [-cl-o(-2)27! + 1 - der - (1-0)(a +b +1) -abz, 0 bien 2(1—a)z" +[2—c—((a—e +) + (b-c +1) + Na)! —(a—c + b—-c+ Vz = 0 Observe que esta tiltima ecuacién es la ecuacién hipergeomértrica con las constantes a,b,c sustituidas por a— c+ 1,b—e+1,2—c. respectivamente. Ademas como la condicién (6.14) para c implica la condicién (6.13) para 2 — c. tenemos que a(t) = F(a-c+Wb-—ct+1,2-c¢,2) es solucién alrededor de x = 0 de la ecuacién transformada. Por lo tanto do(t) =| a | F(a—c+1b—c+1.2-¢,2) es solucién de la ecuaci6n original alrededor de x = 0. Asf bajo la condicién ceZ (que es la condicién (6.13) mas Ia condicidn (6.14) tenemos que la solucién general alrededor de x = 0 es P(x) = aF(a.b.c.2) + |x |e F(a-c+1,b-c+1.2-¢2) aeeR, Ja cual esta definida para 0 <| x |< 1. Para resolver alrededor de x = 1. hacemos el cambio de coordenadas ¢ = 1 = x. Observemos que este cambio leva z = 1 en ¢ = 0 y que # = -1. Luego reemplazando en la ecuacién nos queda 10-054 +a so—e41— (ator yne — ie aby = 0 Luego si ¢ — a — b no es un entero, la solucién general de la ecuacién transformada alrededor de ¢ = 0 es O() = F(a, b.atb—c+ 1.0) +09 | t [°° 8") F(c—b.c—a.c—a—bt 1.1), 1.0 ER. y est definida para 0 <| ¢ |< 1. Por lo tanto Ia solucin general alrededor de x = 1 de la ecuacién hipergeom(¢trica original es (2) =F (a,b,a+b—c+1, 1-2) +02 | 1a |"? F(c—b.c—a.c—a—b+ 11-2). con c1,c2 € R. y esta definida para 0 <| 1— x |< 1, es decir para x en JO, 2/ - {1}. 6.9 Ejercicios resueltos Ejercicio 6.9.1 Usando series de potencias encuentre la solucidn general de la ecuacién (1—22)y’ — Gay! —4y = 0, alrededor de a = 0. Determine su intervalo maximo de definicién. Solucién. Como los puntos singulares son. x = 1 y x = —1, la solucin general altededor de x = 0) esta definida por lo menos para | x |< 1. Sea x y(2) = Pana”. n= Luego —4y(x) = ~Gry'(x) = =e y"(0) = ula) = Sonn = Aaya? = S70 + 2)(10-+ Vay y20” « nme m0 Reemplazando en la ecuacién se obtiene 8 [(—4 = 6r = n(n = Van + (2+ 2)(M+ Vanya]2” = 0. 0 lo que es lo mismo SPL MF 4(n4 Vay + (N+ 2}(NF Vanyz]x” = 0. a0 Por lo tanto debemos tener n+4 Onin = zm VND. Asf para n = 0,1,2.3.4 y 5 obtenemos a = 4a a = fa a = $a = $a a = ta =ha as = Fa = $a a = $05 = far De aqui podemos concluir que para todo n > 1 se tiene am = ODay = (m4 iya yt = MES, Entonces la solucién general es 2 ula) = a [1+ S2(n+1) re") + ay 8 ate panel ag.a, ER. nal nal Finalmente el intervalo ssi de definicién de la solucidn general es ]-1, 1[, ya que por ejemplo la serie 37% ,(n + 1)2?" diverge*para « + 1. Ejercicio 6.9.2 Usando series de potencia encuentre Ja solucién general alrededor de x = 0 de la ecuacién " — dx?y' — Gry = 0. Determine ademas el intervalo maximo donde la solucién esta definida. Solucién.Como esta ecuacién no tiene puntos singulares la solucién general alrede- dor de x = 0 estaré definida para todo x ¢ R. Sea oc Ga) = SP ana” . net Luego —G6r¢(x) = S- —6ayr"t! = »- 6ap2” = 20? g(a) = SO =2na,a"t = »- 2(n = Vay—12" ni . O(a) = Son(n= Vane" = SP (n+ 2)(n + Vany22". nat nao Reemplazando en la ecuacién se obtiene x 2az + B(ag = ag)% + $7 ((—4 = InJanar + (n+2)(n+ Langs n=2 Por lo tanto debemos tener 2 a2 = 0. 43 = do, Y Ony2 = —Say1 VYn>2. me nl" Como el salto en la recurrencia es 3. los coeficientes de Ia forma asp se escriben en términos de ag. los de la forma agn41 en términos de a y los de la forma agn42 en términos de a2. De esta forma tm = ety = ee ee a im Bn = 23 Bn = 2 Bn = 5 Bn a7 Jo que implica que para todo n > 1 A3n = También Qynsy = egg = ee ag amt = Bn 1"? ~ Bn Bn—43n—-7 52 lo que implica que para todo n > 1 gn ant = TET Finalmente como agn42 = 0, paratodo n>0. la solucién general es d(x) = con dp. a; € R arbitrarios. Ejercicio 6.9.3 Usanda series de potencias resuelva el problema de valores iniciales y' + ay -y=0, y(0) = 2. y(0) = a Solucién. Como las funciones x y —1 son analfticas en todo punto, buscamos soluciones de Ia forma . yz) = 2 a2”. n=0 Como n=2 Mr: x (x) = Yo nana y y"(2) = n(n=1) an nat sustituyendo en la ecuacién obtenemos SP n(n=Vanx? + nana” = YO ana” = 0. os mt wo 0 lo que es Jo mismo 2ar — ae + So [N+ 2)(n+ Yanys + (n=1an]a" = 0, m= Por lo tanto 2ag — a = 0 y (n+2)(n+1jany + (n-Lay = 0. para n = 1,2,.... Luego 1 a2 = gH -1 Ando = 7 4 M para n= 1,2... n+ 2)(n+1) De esta forma ag = ds = Qon1 = 0 para n = 1,2, y para n>2 3-5-T+++(2n—3) (2n—3)! = (yet = (yet ) am = (I) Timi a = (“I Sea lami Luego Ia solucién general es 2 -3)! y(t) =a,2 + a (: + ie + y 0" att) : Finalmente la condicién inicial v0) =2 = w=2 y Y= was. De esta forma la solucién particular buscada es alg: Loy n___2n=3)! on yl) =52 +2 ( +t x (-1) P2n—-Dien” |)" Ejercicio 6.9.4 Usando el métado de Frobenious encuentre la solucién general de la ecuacién ay! "aad + 20)y! += $= 20)y = 0, alrededor de a = 0). Determine para que valores de x estd definida Ia solucién. Solucién. E] dnico punto singular de la ecuacién es x Ademés. como las funciones a(x) = —(3 + 2x) y B(x) = $(1 — 2r) son analiticas en + = 0, x = 0 es punto singular regular. EI polinomio indicial es 1 1 1 )=rr-l)-art+= = (r-l)(r-5), a) = Ds + z= -DO-5) cuyas rafces son ry = 1 y rz = $. Luego estamos en,6l caso r1 — r2 = $ no es cero ni un entero positivo, Sea Por lo tanto 1 wl k 390) = aw ST seule) —x¢(x) = —50¢'(n) = 0 Soak tran". vy —207d'(x) = 2lMa) = Reemplazando en la ecuacién obtenemos ca(r)0" g(r) + 57 fale + kee(r) + (-1- 2k 1+ r)xa(ryje* in Ponemos entonces H(-1= 26-14 Nal) _ Art k= Pewalr) _— Qeaealr) ar +h) ~ (r+k-Dr+k=-8) — k+r-1 cnr) = Luego si co(r) = 1 tenemos Wear) Lepn2 a) = ToT = Garena =D ok (r+k-Dr+h=2)e(r+1)r ok r+) +k) Para 7 =r = 1 obtenemos y Gr(a) Para r =r = 4 obtenemos skh! Js = Ss ex(r2) 73. Ok Hl y Por Io tanto la solucidn general es (x) = adr(x) + bp2(x). abeR, Ella est4 definida para todo x € R, « #0. ya que « =0 es el tinico punto singular. Ejercicio 6.9.5 Usando Frobenius encuentre la solucién general alrededor de x = 0 de la ecuacién 2 3 1 wy" + sal + 20)y — 5(1- 12n)y = 0. Solucién.Tenemos 1 P(x) = Satan y Q(x) = ~5(1- 122). Luego el polinomio indicial es 31 1 = rire 8 oll Gag pe b ar) = rr) + Sr 5 = + YO - 5). cuyas raices son 1 =4,y 72 = —1, Ademés como ri —r2 = $ no es un entero, tenemos soluciones linealmente independientes asociadas a r1 y r2. Sea. Luego 1 —59@) = 6rg(r) = 37°¢'(2) = 3(n — 14 r)an1z” 3 dla) =< zee) = 2M) = Reemplazando en la ecuacién se obtiene x aga(r)a” + 2! $7 [alr + njan + (6 + 3(M=1+r))anv2” = 0. ra Por lo tanto debemos tener r =r 0 r= 12 y (G + 3(n—147)) =o > an Ganthetn-h vn>1 Simplificando obtenemos ay = ——2~ amr Yn > 1. r+n-4 Entonces poniendo ay = 1, tenemos: My, para todo n>1. +4) + L £ nos ques Para r=ry = a, = T Jo que genera la solucin Para r =r =—1 nos queda Qn = 2 “2 2" (=1)" 3 5-+(Qn—3) (ciyntt 30 920-2 ( Qn—3)! Jo que genera la solucin \nt1 gn 92n=2 (7) — 9)! at =er (14 3! AAyrtt gn 22-2 (n — 2) Cs) Por lo tanto la solucién general es con a,b € R arbitrarios. Ejercicio 6.9.6 Con los cambios de coordenadas ¢ = 2x4 y y = xz transforme Ta ecuacién . 1 y’ — 2y +8(s° + S)y =0 yoy +al}y en una ecuacién de Bessel. Use esto para encontrar su solucién general. Solucién. Usando los cambios de coardenadas sugeridos cbtenemos y la ecuacion Como # = 823, tenemos dz _ de dt _ ode dx — di dx ~~ dt Bz 5 edz | ge Ga = et + Glo Te. y la ecuacién &z dz ghz 5dz 1 —2)e= Gan Te + 82°F + (Ce? — 2 = 0. Dividiendo por 16x y haciendo el reemplazo 1 = 22‘, obtenemos finalmente Ia ecuacién de Bessel > Pz dz 1 P= +t + (P-s)2 =0, mtg tO cuya solucién general es a) =ah +aJa, aeeR. Por Io tanto la solucién general de nuestra ecuacién es ya) = 2 (a 42a!) +e J_4(2r") . aOeR Ejercicio 6.9.7 Usando el método de Frobenius eneuentre la solucidn general alrede- dor de x =0 de la ecuacién wy! + Sry! - Fe? + Dy =0. Solucién. Como Pl) = 3 y Qa) = Fer +1) son analiticas, «= 0 es un punto singular regular. EI polinomio indicial es 31 1 r) =r(r-l) + Sr 5 = (r + = 5). ar) = r@=1) + Sr - 5 = + DO - 5) Sus rafces son 7) = 4 y ry = -1. Ademés como 7 — rz = 2 no es un entero, podemos encontrar soluciones linealmente independientes asociadas a ry y r2 por el método de Frobenius. Sea Entonces 1 4 & S H y reemplazando en la ecuacidén obtenemos = 1 a (cra + alrjar +12 + O° [ar-+ Hal) - srial)] *) =0. ra Asf debemos tener ¢(r) = 0 (ya que g(r +1) 40 para r=ry y r=r2 y $cr-a(7) ara 22. TEE SES Pas tods b> 2 x(t) = Luego para todo k > 1 tenemos Coa(r) = 0 y asumiendo ¢(r) = 1, Gy (r+ 3) +5) (PF A+AD OT +H +$ Cox (7) = Para r =r = 4 se tiene Gy a) = TEE TT (gy a Tl (4k-+3) 2" Al ; Tl (4k+3) Hi y la solucin Para r =r, =—1 se obtiene con(r2) = y la solucién De esta forma la solucién general es (x) = agi(z) + bpa(zx) con a.b ntimeros reales arbitrarios. Ejercicio 6.9.8 Usando el métado de Frobenious encuentre la solucién general alrededor de x = 0 de la ecuacién wy" —2(3+ 52r)y! + (4+52)y = 0. Solucién. EI tinico punto singular de la ecuacién es « = 0. Ademas, como las funciones a(z) = -(3 + 5x) y b(x) = 4+ 5x son analiticas en x = 0, x = 0 es punto singular regular. EI polinomio indicial es r(r—1)-3r+4 = (r-2)?. cuyas rafces son ry = Sea d(x) = o(t.r) = 2" Sat, i= Por Io tanto 4g(z) = Sad(x) = -329'(x) = ~5179"(n) = Pg"(2) = a! Senierr-nai i=t Poniendo co(7) = 1 y reemplazando en la ecuacién obtenemos a q(r) +2" fale Rea(r) +51 — (kK-1+7))cnea(r2* = 0, ia Ponemos entonces Bk-+r = Dopoa(r a(e) = HEEB) Luego ) _ Bewalr) _ Bene2 alt) = oo a 5k ~ GER-DrtR-3) + Dr 1 5k (Dre t DG +k-2)- Para r = ry = 1 obtenemos 5(k — 1+ r)oq—alr)a’ k Para obtener una segunda solucién linealmente independiente con la anterior, cal- cullamos ¢(r) para r cercano a 2. Tenemos 1 T=Da0 sl) Gs k=d ) ORG) RD 1 t+ Garr th— 2) _ —5k rote ty yt — (ra Dr(rt leer +h=2Qer-1 ro rtl r+k-2 Por Io tanto ; #1 1 crs) = ll + ste tq) Luego nuestra segunda solucién es ce ee =i Por lo tanto la solucién general es a Re vi st pe + In(| x |)4u(«) = d(x) = adi(x) + bpr2(x), abeR. Ella esta definida para todo x € R— {0}, ya que = 0 es el tinico punto singular. Ejercicio 6.9.9 Usando el métado de Frobenious encuentre la solucién general de la ecuacién ry" — xy! +(1-2)y = 0, alrededor de x = 0. Determine para que valores de x esté definida la solucién, Solucién. El tinico punt funciones a() = —1 y (x) = regular. EI polinomio indicial es gular de la ecuacién es x = 0, Ademés. como las —« son analiticas en x = 0. = 0 es punto singular gr) = rr-1-r+1 = (r-1), cuyas rafces son ry = Sea Por Io tanto —ro(x) = -2¢'(n) = 2p"(a) = Reemplazando en la ecuacién obtenemos co(r)a" g(r +e (r + k)ex(r) — cea(ry]a* = 0. Ponemos entonces — aalr) _ al) = a Luego si co(r) = 1 tenemos ) Gilr) Ch-2 a) = (iret = Ware kar 2 1 (+k—-1(r Pk-22 (r+ ee 1 Pir+lp(r+k—lp- Para r =r = 1 obtenemos ex(r1) = oy Para obtener una segunda solucién linealmente independiente con la anterior, cal- culamos ¢,(r) para r cercano a 1. Tenemos 40) = agape Rcaet ape = —2c,(r) fe): Por lo tanto —2 1,1 1 ye dg] Gs) = Gale tg tet gl Luego nuestra segunda solucién es =2.1,1 1 r(x) = 2 tated get + In x Doi). PT Por lo tanto la solucién general es d(x) = agy(x) + bb2(), abER. Ella esta definida para todo x € R— {0}. ya que x = 0 es el tmnico punto singular. Ejercicio 6.9.10 a) Muestre que la ecuacién de Legendre (1=22)y" = 2ry! +m(m+ Dy = 0. se convierte en una ecuacién de Gauss con el cambio 2? = t. b) Usando parte a) determine dos soluciones linealmente independientes para m = 4. Solucidn. a) Como 2? = ¢, tenemos dy dy f= Boo, YO ed wo — &y ody | Py 0 ea ae Reemplazando en la ecuacién obtenemos ct) (2M 4 a Y) — a veo vi 4 mim + 1y=0, dt dt? dt o bien 4u(1— yee + (211-2) -— ag +m(m+ljy=0, de a y dividiendo por 4 =0. yb = VoD _ a nuestra ecuacién de Gauss es 3 y(t) = QF (3 - Por lo tanto la solucién general de la ecuacién original es 3.115 51 yla) = aF (3-337) + coeF Gi wie & XY Capitulo 7 Transformada de Laplace 7.1 Definicién y Propiedades Sea f : (0, -+00c[—> R y suponga que x R [ eo" f()dt = lim [ eo" f(t D Rx Jy converge para algunos valores de s. Para estos valores definimos una nueva funcién f, llamada transformada de Laplace de f, poniendo fis= | * = F(ab . 20 Notacién. ccsoy(0) = Fess = f emf (Oat 0 si esta integral converge. Teorema 7.1.1 a) Les lineal, es decir dadas f.g : (0, +>0/—+ R tales que f(s) y 9(s) existen, Va,b € R se tiene L(af(t) + bg(t))(s) = aL(F(t))(s) + bL(G(O)(s) b) Si f,9 : (0, +oc[— R son continuas y f(s) = G(s). entonces f = 9. c) Si f" es continua en (0. +90 y fi Le F(t) =0. entonces L(f'())(s) existe => L(f(t))(s) existe. Ademds se tiene L(f'(t))(s) = sL(f (t))(s) — F(0). ‘ 4) Para f : [0.+00[—+ R continua, definamos F(t) = [ flu)du. Si para s > 0 0 L(f(OMS) eniste y lim e~*'F(t) = 0. entonces se teene que L(F(W))(8) existe a0 y que Demostracién. a) Se deja al lector. b) Pendiente (vea seccién 7.8). c) Sea s tal que dim FO) =0, y R> 0. Entonces [orem = tno /t es [eeroa R = &*"#(R) — f(0) ssf eo" f(tdt 0 Luego Bast a 7 st i st 14) ist ir st £(4) iste. tim, | ce" fl(Hdt existe <> dim, [ ce" f(O}dt existe y en caso positivo L(F'H)(s) = sL(F(H)(s) - £(0). d) Tenemos que F’(t) = f(¢) es continua y F(0) = 0. Como £(F"(é)}(s) existe y tim. e~“'F(t) = 0. entonces L(F(t))(s) existe y L(F())(s) = LEF'(O)(s) = sL(F())(s) — F(0) = sL(F(d))(s)- Por lo tanto Ejemplo 7.1.2 £(1)(s) =! Ys >0. En efecto sis > 0 e 1 [ edt= -=(eP 1). A 5 lo que implica £(1)(5) = im [ eta=t ys>0. Jim], Observe que £(1)(s) no existe para s < 0. Asf £(1)(s) sélo est definida para s>0. Ejemplo 7.1.3 Pongamos f(#)=1 Ve > 0. Entonces F(t = [ f(ujdu = ty para s > 0 lim_e~“F() = lime“ =0. (toe (be Luego L(F(H))(s) = 0 % 8 i 5 => L\(s) tad vs>0. Ejemplo 7.1.4 L(#)(s) = 2. Ys >0. Sea ahora f(t) = 2t y pongamos F(t) = ff! f(ujdu = @. Entonces para s > 0 tenemos lim e7 1-340 De esta forma LEMs) = CFOs) = Le(HO)(s) = LeeAKs) 2 2 = 2L(()=5 ve>0. 1 Ejercicio 7.1.5 L(t")(s) = a Ys>Osin=12-. Vs>1. Ejemplo 7.1.6 Para R> 0 n [ telat = [ end = 5 5 R lim e“edt = R400 Jy Ejemplo 7.1.7 Considere el problema de valores iniciales y-y =1-t y(0) =2 Sea y(¢) la solucién y asumamos que L(y(t))(s) existe y , Jim, e~“ y(t) = 0 para toc todo s > so. cierto sp. Entonces como y/(t) es continua, L(y/(t))(s) existe Ws > so. Luego para todo estos s se tiene Liy'() = wlt)\(s) = LIL = H(s) sL(y(H)(s) — y(0)- LO)(s) ==-S (s-Dewy)-2 = 4- L£(y(t))(s) = 1g] __s-l+2s* 2 |- s%(s=1) = at = L(t}(s) + L(2e")(s) Liylt)\(s) = L(t+2WV(s) Ws>s0. y yt) = t+2c!. Definicién 7.1.8 Sea f : [0,+50[—> R continua. Decimos que f es de orden exponencial ) (orden exp. b) si existe M > 0 tal que e“"|f(Q| 0. Teorema 7.1.9 Sea f : [0.-+00[—+ R continua y de orden exp. b. Entonces a) L(f()(s) existe Vs > b. b) Si ademds f" : [0.+bo0[—> R es continua, entonces L(f'(t))(s) existe Vs > b y L(F'()(s) = sLF(O)(s) — F0)- ‘ c) Sib AD y FQ = [ F(u)du, entonces F es de orden exp. b y 0 LEF(WMS) = AEUW)(s). V5 >. Demostracién. —_ Ejercicio. Observaciones 7.1.10 Si f. f’. f",-, etc. son continuas y de orden exp. , se tiene para s > b: LF W)s) = sL(F(O)(s) - £0) LEF(OM8) = PLAY) - sf(0) - FO) LFMONs) = FLFO)() =f) — sf'0) - Ff") ete. Ejercicio 7.1.11 f(é) de orden exp. b= =e7*'f(#)_ es de orden exp. b-a y para s>b-—a_ tenemos Lee FD)(8) = L(F(O)(s +a). Sabemos e™|f(Q| 0. ert f(D] = E™|F(Q| SM EDO e~" f(t) es de orden exp. b—a. Ademés para s > b-—a R Cleatpyia) = lin [ere foe R—toe Jy Ro = lm [ et" F(t Rt Jy = LFW)Ms+a) Ejercicio 7.1.12 L(sen(bt))(s) = para s > 0, En efecto. para R > 0, tenemos R [ e"'sen(bl)dt = —= 0 R R en(bt) | +f e cos(bh)dt A = -henon) +4 +3 [-tew* cos(bR) -1] - z[" e7sen(bt)dt 2 BY fP_, Ioan 6 b (+5) [ etsen(bat =e ren be cost) + % Licon(t))(s)=—2— 4 vs 30 b \(s)=—e = Loen(tt))(s)= spe Ws 0 forat cl 8 Ejercicio 71.13 L(cos(bt))(s)= zy Vs > 0. Para s > 0) tenemos b L(beos(bt))(s) = L{(sen(bt))’)(s) = s£(sen(be))(s) — sen(0) = Saar’ Luego L(cos(bt))(s) = iercici enat ))(s) = _ Ejercicio 7.1.14 L(e~ sen(bt}}(s) = Gage Vs > a. Aplicacidn directa del ejercicio 7.1.11. 1 iercici (5) — Ejercicio 7.1.15 — L(te*)(s) = GiB Vs> 1. Usando ejercicio 7.1.11, tenemos Lite!)(s) = L(t)(s-1) Vs>1, Jo que implica = L(te')(s) = = vs>1. Teorema 7.1.16 Si f es de orden exp. b, entonces f(s) = L(f(t))(s) tiene derivada de todas las ordenes para s > b y (Als) = L-tFO)(9);, ANS) = LFV): (AM) = L(-EF(O)(s). ete. Demostracién. Zero) = -le~' f(t) = “8 (-1f (1). 20 Ademés la integral e~“tf(t}dt converge uniformemente para s > by donde by > Bes cualquiera {es decir Ve>0 3N>0 talque R>N => R 36 if ef (Qdt — “tf (t)dt|lo). ‘0 Tenemos entonces 2%) - 2[ -[ Le Rf) = Z[eeroa= [Zero ~ = [ P(t fOdt = L(-UF(0)(s). , Ejemplo 7.1.17 Tenemos L(1)(s) = t pata s > 0. Por lo tanto para >a_ (ver ejercicio 7.1.11). Pero i (; = 3) = — =L{-te")(s) para s > a. Lite"\(9) = Gage p¥s>a Ejercicios: . ni paras > ay en particular £(0")(s) = Gat para 1 cept) = 7 s>0. an 2. Si fi. fa son de orden exp bi. be respectivamente, entonces fr + fr es de orden exp b= max{by.bo}. 3. Si fi, f2 son de orden exp bi, b2 respectivamente, entonces f; « fz es de orden exp b = by + ba. Lemma 7.1.18 Sean q(t),q/(0) continuas y de orden exp. b y sea $(t)) solucién de yl — ry = a(t). Entonces $(t). $!(t), !(t) son continuas y de orden exp. YU = max{b, |r|} Demostracién. Sin pérdida de generalidad asumimos que b > |r|. a) Sir € Bla solucion es p(t) = eM(c+ [° €-"q(u)du). Luego basta demostrar 0 ‘ que U(t) = | “Mq(u)du, y sus derivadas U'(t), U"(t) son continuas y de orden exp. br. Pero q(t) de orden exp. b implica e~™|q(t)| 0 Entonces e—@="%Je-P%q(u)| = e~M|q(u)] 0- y luego ‘ “ig(udul < cent [ Meu qu) f 1 = Me (coe U(t)es de orden exponencial br. Ademés es claro que U'(1) = e~"g(t), y UM (t) = =rem" g(t) + e—"a"(0) son de orden exp. b-r. b ) Sir € CG, digamos r= ay 18. la demostracidn es un poco mas complicada y se omotird en estas natas, Teorema 7.1.19 Si q(t), q/(t) son continuas y de orden exp. b y d(t) es solucién de y!" + pay! + pay = a(t) 5 (7.1) donde pi,p son constantes, entonces: o(t),9"(t)."() son continuas y de algin orden exp. Demostracién. Sean 1,12 rafces de 1? + pyr + pp = 0. Entonces —(n +12) =P. Y M172 =Po+ Sea @(t) solucién de (7.1). Definamos x(t) = ¢'(t) — r2¢(t). Entonces AO -nAO = $") -r¢'O —r('O - r24(0) PD = (11 +12) 8" + rirad(D) = a) ) 4 Por lo tanto $1(é) es solucién de y’ — riy = 9(é). lo que implica (0 continuas y de algtin orden exp. 14 (0), of (t) son Pero como ¢(t) es solucién de 4/ — ray = dy(1) , obtenemos también que $(2), #(t) y $"(2) son continuas y de algtin orden exp. Corolario 7.1.20 Si a i WD son continuas y de orden exp. b y o(t) es solucién de (7.1), entonces L(g())(s). L(¢'(O)(s) y L(G"(t))(s) existen para todo s sufie cientemente grande. Teorema 7.1.21 El operndor £ es 1-1; es decir, si f yg son continuas en [0, +90 y L(P\(s) = L(g(t)(s) Ys > so, entonces f(t) = g(t) Vt > 0. Demostracién. La demostracién esta en la seccién 7.8. Usando este teorema podemos definir el operador inverso £-! de Ia siguiente forma: L7*(A(s))(0) = FOSLFW)5) = dls). Observaciones 7.1.22 £7! es un operador lineal: es decir, Lads) + bY8))(1) = aL*(H(s))(4) + BL*(W(s))(1) Ejemplo 7.1.23 Considere la ecuacién y" +4y =t+ sen(2t), y(0) =y'(0) =0. Aquf q(t) = + sen(2t) y ¢(Q) =1 42 cos(2d) son continuas y de orden exp. b cualquier b > 0. Luego, la solucién (fy sus derivadas y/(#) y y(t) son de orden exp. a para algtin @ suficientemente grande. Luego para s > a tenemos Liy"(t) + 4y(O)(s) = L(t #sen(2t))(s). 1, 2 2r(y()(s) 4 4L(y(D\(s) = FLUV(O)(s) + LOS) = Stay 2 1, 2 2 Lacs) < 2 (8 + 4)L(y())(s) = stu 1 2 LyO)(s) = Fer | Rape De esta forma Pero Afirmacién L(sen (at) — at cos (at}) = @aiee wrap En efecto : we a Leen (abs) = SB - 3 Leoslad\is) = 5 L(-at cos(at))(s) = a£(—tcos(at))(s) = Luego 1 C _ ))(s) =a |—— L(sen(at) — at cos(at))(s) =a E vat Por lo tanto aft \yioly — 2# cos (24)) co (=z) (0) = sa (on (28) = 2 c05 (20). De esta forma yt) = Fe = Fen (24) + £ (son (24) = 2¢ cos (21)) = Fe te cos(2e): Ejemplo 7.1.24 Resolvamos la ecuacién. y! + 2y! + 2y = 2e~*cos(t), (0) =2, y'(0) = -2. Pongamos Y(s) = £(y(t))(s). Tenemos C = #Y(s)—sy(0)—y'(0) = s°¥(s)—254+2, 2(sY(s) — y(0)) = 2s¥(s)-4, 2¥(s). Ast). (s+124+1 Entonces. aplicando transformada de Laplace a nuestra ecuacién y reemplazando se obtiene (s+ 1) A 95-9 = . (3? +28 +2)¥(s) — 28-2 = Por Io tanto %s+1) s+ 1) YS) = Gyn? Gener Ast) d -1 G+ie+1 aerial p= s+l 1 -1 Ho = 20 (Taps) 0-1" (wees). = 2c cos(t) + te~'sen(t). Ejercicio 7.1.25 Sea (s) = £2 con P.Q polinomios. $i Q(s) tiene solo rafces simples (reales), digamos Q(s) = (s — r1)(s — r2)+++(s— tn) con ry #7; Sit F J. entonces Ejemplo 7.1.26 Descomponga en fracciones parciales Tenemos P(s) =2s+2. Q(s)=s'-s?-ds+4 = (s—1 y Qs) =(s+2)(s—2) +(s—1(s—2)+(s—1 Luego ocupando el ejercicio anterior obtenemos st -I5+2 +4544 (s+2(s—2) (s +2). st =Sel_ 3 s-1 Ejemplo 7.1.27 Resolvamos la ecuacién y" — 2! —3y = ef. y(0)=y/(0) =1. Poniendo Y(s) = £(y(¢))(s) y aplicando transformada de Laplace a nuestra ecuacin obtenemos #Y(8) — sy(0) (0) — Ass) - y)) -3¥(8) = — Luego (2—2s-3)¥(s) = 2 s—1. lo que implica 1 s-1 ye YS) = Gonespe-3 she _ 1+(s- 1) ~ (s=1(s+1(s-3) 1 (51 ,5 1 1 “Gs-1 7 8541! 8s Por lo tanto 7.2 Funciones Discontinuas Definicién 7.2.1 f : [0,+-0c_ —+ R se dice continua por partes si f tiene solo un niimero finito de puntos de discontinuidad en [0.+oc[ y estas discontinuidades son de tipo salto. Ejemplo 7.2.2 Considere la funcién escalén (0 salto) 0 st t<0 u(t) = (tiene salto en t = 0) 1 si t>0 Luego si para a > 0 definimos f(¢) = u(¢ — a), tenemos 0 si ta 'sOdt +f oe ldt = dew, Por lo tanto 9 L(u(t—a)) = para s>0 Ejemplo 7.2.3 Considere la funcién 3 t<2 f@=4 -1 2 R una funcién continua, Entonces para a > 0 se tiene L(f(t—a)u(t — a))(s) = K“L(F(O)(s) + En efecto Lilt a)ult—a)) = [ c-" f(l-a)dt = [ eras f(u)du os I e™ F(u)du = eB L(F(D)(5) « Ejemplo 7.2.5 Calculemos Sea Entonces ct (=) () = £1 (€* LFW) (© = FE- Qu - 2) 0 si £<2 = t-2yitan)={ t-2 si t>2 Ejemplo 7.2.6 Calculemos la transformada de Laplace de t O 0). Asi £(f(t))(s) existe para todo s Suficientemente grande y como para todo niimero natural y s > 0. £(¢")(8) = #4. tenemos que . SS (H1k (2k)! LFOMS) = LP Ys) = yl sie yu Qk+ Di se = ett, il, a kT s 383 5s? 7s? Recuerdo: Para |x| < 1 tenemos 1 _ 2444 pep = ictte x 1 gak+t arctan(t) = fo TR +1 Por lo tanto L(F())(s) = arctan (:) (s>1). 7.3 Funciones Periddicas Definicién 7.3.1 Se dice que una funcién f :R — R es periédica de periodo T, si T es el menor neimero positivo que verifica ft+T) =f) WER Ejemplos 7.3.2 sen(t), cos(t) son periddicas de periodo 27 tan(é) es periddica de periodo 7. Observacién 7.3.3 Para especificar una funcién periddica, basta dar sus valores sobre un periodo. Asf la funcién F(t) 1 Q 0 ; cu) = ee | Demostracién. Para s > 0 tenemos: cua = [P eenoa f ena [ 7 FO) vars eA f(i)dt +o = [fe ew f(t) cours ["e HT) nernare ["e RD) Ft 4 OT dd +o 0 —# F(t)dt. = ts ars [7 post ware [7 st . = [ e" f(i)dt + & [« f(dt+ € [« f (dt + r = GeePeetan) [ ev" f()dt 0 1 T -,i f ey Oat. Ejemplo 7.3.5 Calculemos la transformada de Laplace de la funcidn periddica f(t) del ejemplo que aparece en la observacién 7.3.3. Para s > 0 tenemos CUMS) = oe [eset 7.4 Convolucién Consideremos el problema y" +y = g(t): y(0) =0, y'(0) =0. Poniendo ¥(s) =L(y(O)(s) -y_ G(s) = L(g()(s) - aplicando transformada de Laplace a la ecuacién obtenemos — e+l w=c (s 1 -G()) wo. Definicién 7.4.1 Sean f.g : [0.+90[— R continuas por parte, La convolucién de las funciones f(t) y g(t), €s una nueva funcién, que se denota por f x9. y que se define por (fale =f f(t=2) Ejemplo 7.4.2 2 ¢ 2 tsP = [ (t— v)e2du i Propiedades: Sean fg, h : (0.>0[— R continuas por parte. Entonces: I) feg=gef. 2) Fe(gth)=feg+ fah. 8Y(s)+¥(s)=G(s) => Y(s)= As). Por lo tanto t oot a © 38 4° 12° 3) fx(geh) = (fxg) eh. 4) fx0=0. Demostracién. Haremos la demostracién de 1). Los item 2), 3) y 4) quedan de ejercicio. [ 0-waeman= f° f(w)g(t = w)(-dw) = [ a(t w)f(w)dw = (9* NO Teorema 7.4.3 Scan f.g : [0,2c[—> R continuas por parte y de orden exp. a. Entonces (fF * g(t) Lf *9)())(s) = LF(O)(s) - LG)(s). 0 equivalentemente si F(s) = L(F(Q)(s) y G(s) = Llg(t)(s). entonces LUF(S)G(S)\( = (Fa M(O- Demostracién. Para s > a CF e arenes =f eer eatow [ em (f f(t- alert) dt [ et (f° u(t v)f(t- vast) dt = [ g(v) u e~ult - v) f(t = vit) du = | Gle)LF(t= v)u(t = v))(s)dv A x _ glen Fls (eyau= rs) [ c= g(v)de c c = )G(s). Entonces volviendo al hon inicial ¥(s) = G(s) = i st] - Jo que implica Ejemplo 7.4.4 (1+ f(t) = ff f(wdu. Ejemplo 7.4.5 1¥¢ [ (t= v)edu = 0 = Ltxt)(s) = L(t)(s) LE Ejemplo 7.4.6 Calculemos ¢ + sen(/), Tenemos t a a txsen(t) = [= eseneqae = [ sen(v)dv ff esenoyae 0 0 0 -t cos(n) /* —[-u cos(a) /' - [ cos(v)du = o = 1 cos(t) +4 + £ cos(t) — sen(t) = 1 = sen(t). Por otra parte L(t * sen(t))(s) = L(t) (s) » L(sen(t))(s) = Ejemplo 7.4.7 t xe! L(t *e')(s) a t Ejemplo 7.4.8 — sen(t)*cos(t) = [ sen(t—v) cos(v) de = [ (sen(t) cos(v) — 0 0 cos(t) sen(v}) cos(u) dv = +++ = 5 tsen(t). Por otra parte L(sen(t) * cos(t))(s) = 8 S41 S41” GFT?" Ejemplo 7.4.9 — Resolvamos la ecuacién y"+y = cos(t}, (0) = y'(0) = 0. Poniendo ¥(s) = £(y(t))(s)__y aplicando transformada de Laplace obtenemos 8 *¥(9) + (9) = s- Por Io tanto ee ee YQ) = Gay > Fer Fy = L(sen(t))(s) + L(cos(t))(s) = L(sen (¢) ¥ cos (1))(s) y asi y(t) = sen(t) *cos(t) = Ftsen(t). Ejemplo 7.4.10 — Resolvamos y"—5y'+6y =e, y(0)=0. y()=-2. Poniendo Y(s) = £(y(¢))(s)._-y aplicando transformada de Laplace obtenemos #¥(5) = sy(0) = (0) #ALSY (8) — (0) + 6¥ (9) = 5. Entonces (—5s+6)Y(s)+2 = 2 —s+6 (s+2(s—2(s—3) (8-2 Pero co 1 (t) = eat = [ 2-2) 63° hy s—-2 5-3 5 ! dy = af edu = Ae! = 1) 0 Ademés (Sa weae=5) - =o(fe tata yd) 5° 5 = le Lee, Lew = 5° € + 55° Por lo tanto yi) = tet — heat y Ayah — agar yg gat / 5 4 20 = 9 St i. /2t 1 —2t = 5° t + 0° . Ejercicio 7.4.11 Usando transformada de Laplace, encuentre la solucién de y" — 2y! + 5y=—8e*. (0) = 2, y/(0) = 12. 7.5 Ecuaciones Integrales Sean f(x).k(x) funciones dadas. La ecuacién : eo) = yto) + [wenn ; donde y(z) es la funcidn incdgnita, se Hama ecuacién integral. Solucién. Aplicando transformada de Laplace a ambos lados obtenemos L(F(@)\(s) = L(y(a)}{s) + Lh * y)(a))(8) = LF (@))(8) = LYy(a))(s) + L(R(a))(s) Ly@))(s) LF @))(s) T+ LER@)(5) LAOS] () + L£(K(a))(s)] = LYy@)(s) = Ejemplo 7.5.1 Resolvamos Tenemos t 7.6 Funcion de transferencia Considere el problema de valores iniciales y+ ay’ +by = FO), yO) = Este puede ser pensado como un sistema mecdnico 0 eléctrico en reposo que es excitado por una entrada f(t). Aplicando transformada de Laplace obtenemos x Y(s) + asY(s) + bY(s) = L(F(O)(s) = Fls) y despejando F(s) >—— _ = P(s) F(s). s+astb (8) F(s) La funcién P(s) = (s?+as+b)7? se lama funcién de transferencia del sistema. En el dominio de las frecuencias el sistema puede ser descrito por el diagrama de la Figura 35, donde el rectdngulo representa la accién del sistema en reposo sobre la entrada, lo que en términos de transformada de Laplace consiste en multiplicar por Ja funcién de transferencia. Y(s) = Tone Output Aplicando transformada de Laplace inversa obtenemos Wl) = LN) = LUPO) FS), y luego y(t) = pt) * FO) donde p(t) = £-1(P(s))(#) se Hama la funcién peso del sistema, e y(¢) se llama respuesta de estado-cero. En el dominio del tiempo el sistema puede entonces ser descrito por el diagrama de la Figura 36, donde el recténgulo representa Ia accién del sistema en repaso sobre la entrada, lo que consiste en hacer convolucién con la funcién peso, | Garostate cntpat put Figura 36 Ejercicio 7.6.1 Demuestre que (0) = 0 y que si los coeficientes a y b son positivos se tiene i y= im p'()p= fim p()}=0 y jim pO Escribamos y(t) = p(t) * f(é) en su forma integral ui) = [rt fe-n ar. Como 0 < 7 < ¢. la integral puede ser interpretada diciendo que la entrada evaluada en 7 unidades en el pasado, a saber f(t—7), es ponderada con el valor de la funcién peso evaluada en el tiempo 7. Para sistemas del mundo real los coeficientes a y b son positives y la entrada f(1) es una funcién acotada. El hecho que a y 6 sean positivos implica que la funcién peso p(T) tiende a cero cuando 7 se aproxima a infinito. Luego existe T > 0 tal que p(T) es despreciable para 7 > T. Entonces, para t > T, tenemos wo = [ vio) =n)ar + [pt f(t-7)dr. y el valor de la segunda integral se puede considerar despreciable. Una manera de decir esto es que los valores de la entrada para mas de T unidades en el pasado tienen muy poco efecto sobre la respuesta; es decir, para cualquier ¢, las contribuciones de f(t—7) sobre la respuesta son despreciables para 7 > T. Ejercicio 7.6.2 Demuestre que la solucién del problema de valores iniciales y'+ay+by = FO, yYO=u:. YO=Hn, es yt) = p()*fO + (yu + wp + ver). 7.7 Impulso unitario Dado un sistema mecénico o eléctrico, deseamos modelar una entrada de gran mag- nitud que ocurre sobre un pequeno perfodo de tiempo. Para describir este compor- tamiento primero consideremos para cualquier mimero positivo ¢ la funcién 0 si t 0, sea 5-(t—¢g) la funcidn 6.(¢) trasladada fy unidades a la derecha: 0 ‘si th&+e. Usando la funcién escalén unitario se puede escribir de la forma 1 d-(t—to) = u(t — to) — u(t — to — y su grafico es como el de la Figura 37. y =d2(t = to) Claramente, cuando ¢ se aproxima a cero. el impulso rectangular J-(¢ — fc) es cada vez més alto y angosto y el Ifmite no existe. Aunque e] limite lim.49 5-(t—t9) no existe en el sentido usual, uno puede derivar algunas interesante propiedades de la funcién. Primero, como 6.(¢ — to) = 0 para todo ¢ fuera del intervalo tp < ¢ < tp + ¢, se sigue que oc tote y [ O-(t — to) dt = [ =dt =1. (7.2) 3c to © Si 6.(t— to) es pensado como wma fuerza. entonces 7.2 dice que el impulso total es unitario, Segundo, dada cualquier fmcién continua f(t) definida en el intervalo -oc < t < oc, tenemos - tote fot [ 5-(t— to) f() at =[ Lhaa = i[ flat. € Recordemos que el teorema del valor medio para integrales establece b [ f(Bjdt.= (b-a) f(r), A para algtin 7 en el intervalo a < t < b. En consecuencia, x 1 [ att) fiat = Leyte) = fen). (73) . para algiin 7: en el intervalo fo < 1 < to +. Tercero. la representacién de 5-(¢—¢o) en términos de la funcién escalén unitario implica que para fy > 0 1 £(t—h)(6) = 3 ensto easitots) 8 s | =e ne” = OA) Supongamos ahora que se permite apraximar ¢ a cero en cada una de las propiedades 7.2. 7.3 y 74. Si 5(t fo) denota el resultado de hacer ¢ tender a cero, entonces 7.2 implica x [ S(t to)dt = 1, (7.5) | En la propiedad 7.3, como 7; debe permanecer en el intervalo fo < ¢ < ty +e. tenemos lim-49 7 = to y [ott Fiat = Flt. (76) para cualquier funcién continua definida en el intervalo —90 < ¢ < oo. Realmente, pasta que f(/) sea continua solamente en alguna vecindad de ¢ = fo. y puede estar bast t ti Jamente en al lad de t= to. y puede est: definida como cero fuera de ese intervalo. En la propiedad 7.4. que define la transformada de Laplace de 6-(t— fo), observe que por la regla de Hopital lim = By Por Io tanto, L(5(t — to))(s) = e*®. (7.7) De esta forma haciendo fe > 0 obtenemos L(5()(8) = 1. (7.8) Este resultado nos previene que d(f) no es una funcidn bien comportada, ya que si Jo fuera, su transformada de Laplace serfa una funcidn que va a cero cuando s va para infinito. La “funcin” 5(2) construida anteriormente se lama funcién impulso unitario o funcién delta de Dirac. No es una funcién en el sentido tradicional pero es un ejemplo de lo que se suele lamar una “funcién generalizada” o “distribucién” . Ejemplo 7.7.1 Encuentre la respuesta al estado-cero para la ecuacién y" — y = d(t-1). El lado derecho puede ser mirado como un impulso unitario aplicado en t = 1, Si Y(s) es la transformada de Laplace de y(t). entonces usando la formula 7.7 tenemos sY(s) = ¥(s) = Y(s) = | 1 | Por Ia tanto, Sa1 y luego y(t) = [senh(t— 1)]u(t-1). wo) = 0 si O so. Debemos demostrar que f(t) = g(t) para todo t > 0. Considerando la funcién h(t) = f(t) — g(t). tenemos que H(s) = L(h(t))(s) = 0 para todo s > se. Luego. basta probar que h(t) = 0 para todo t > 0. En particular. para todo entero no negativo n se tiene x 0 = H(so+n+1) = [ Or Dleo! Hitt. (7.9) 0 Sea v(t ape e~"h(wjdu. Por lo tanto, v es continua en [0, +oc[, 0(0) = 0 y jim. u(t) = Als; Inteorando en ot por ves obtenemos R 0 = Also +n+1) = jim fe omer) bors) f emt) AL b 450 Jo que implica 7 [ eM My(idt =0. Yn=0,1 0 Hagamos el cambio de variables u Por Io tanto, e"™D! = un + 1), t= —In(u) y dt = —4du. Entonces si ponemos g(u) = v(—In(u)), tenemos x 1 0 -[ ey ()aL = [ ullg(u) b lb 1 [ ig(u)du = 0, VYn=0,1.+ 0 Lema anterior implica g(t) = 0 para todo t € (0. 1]. Pero g(t) = v(—In()). y por lo tanto u(¢) = 0 para todo ¢ > 0. t Tenemos entonces 0 = u(t) = [ e*"h(u)du, y derivando h(t) = 0 para todo t > 0. ° que implica 7.9 Ejercicios resueltos Ejercicio 7.9.1 Usando transformada de Laplace encuentre la solucién de Ta ecua- cién y! —2y +5y = -8e (0) = 2 (0) = 12. Solucién. Llamemos Y(s) = L(y(é))(s). Entonces Lly'())(s) = s¥(s)—y(0) = s¥(s)- Liy"(H)(s) = #¥(s)- sy(0)-y'O) = #¥(s)-2s-12 y L(-8eY(s) = -8L(e“Y(s) = Luego aplicanda transformada de Laplace a nuestra ecuacién obtenemos =8 2 0545)¥(s)- 2-8 = (s? = 28 +5)¥(s) - 2s = lo que implica 2s? + 10s — =Is+5 e+ 1D Por lo tanto na (38-1424 1 wo = 08 (Coo) 0 = 3c" (Gye) O+ (Gare) 0- 1 £"(—)tt (ao = Be! cos(2t) + detsen(2t) - e~*. Ejercicio 7.9.2. Usando transformada dé Laplace encuentre la solucién de y! + 4y' + 13y = 2t + 3e-*' cos(3t). (0) =0, yO) =- Solucién. Sea ¥(s) = L(y(t))(s). Al aplicar transformada de Laplace a nuestra ecuacién obtenemos 2 3(s+2) ay Ay 2 SY(9 H1+457(9) +1) = S+ EEE Io que implica 1 2 3(s +2) Y(s) = ————_ 5.§ —— > __ ,§ — 2 Fs) (s) Pass! PS basis | +t ep Tenemos Lo 103 s+4s+13 (s+2)24+9 3(s+2)24+9 y luego -1 1 —1 ( = -2¢7-%sen(3t). £ (=) 0) ze sent!) Ademés si ponemos 2 ALB, O34 s(st+d4s+13) 5 82) st 445413 As(s? + 4s +13) + B(s? +4s + 13) + (Cs + D)s? se +45 + 13) obtenemos 8 6 C= ig = i65 Entonces —_— +. 8st 6 32(s2 + 4s + 13) 169? 4s +13 42. st2 __10_ 8 169° (S422 49 3-169" +2249" 2 -8 2 8 10 _, 1 = 8 2, 8 cmt coaiayy — 22 6-2 sont). (aevet mH) = Tay at Tag CO) Bape send Finalmente, como 3(5 +2) (2+ 48+ 13) 3 1 2 ( +454 a) 1 2 a ds a(__ 3 ds \(s +2)? +9 cr 3(s +2) ( i —2t wee) () = ste sen(3t) obtenemos Por Jo tanto 179 8 1 2 8 yy La p-2t g, t) + —e7* cos tent t) + yt) = ar sen(3t) + To0 © cos(3t) + ge en(3¢) + 137 169° Ejercicio 7.9.3 a) Si y(t) es una solucién de la ecuacién de Bessel de orden p : Py") + O+( Py = 0. muestre que la transformada de Laplace Y(s) = £(y(t))(s) satisface (1+ s3¥"(s) + 3sY"(s) + (1-p°)Y(s) = 0. b) Resolver esta tiltima ecuacién para p = 0, expresAndola en la forma Liaiyr'ts) + B(s)¥(s)] = 0 para algtin A(s) y B(s). Solucién. a) Tenemos L(-py(t))(s) = —PY(s). Li y(t)\(s)_ = Y"s), Lity(Oy(s)_ = A(-LYW)(s)) = -Z(6¥(s) - yO) = -¥(s) - s¥"(s). LEy"O\s) = B(Ly"(O)(s)) = Hl (s) - sy(0) - y'(0)) = 2(s)+4s¥"(s)+°¥"s). Aplicando transformada de Laplace a nuestra ecuacién obtenemos LP y"(b)(s) + Llty'®)(s) + LP yb)(s) + L-Pyb)}(s) = y reemplazando se tiene 2Y (s) + 4s¥"(s) + 8°¥"(s) — ¥(s) — s¥"(s) + ¥"(s) -p*¥(s) = 0. es decir (1+s7)¥"(s) + 3sY"(s) + (1-p)¥(s) = 0 b) Poniendo p = 0 obtenemos la ecuacién (1+5°)¥"(s) + 35Y"(s) + ¥48) = 0. que podemos escribir de la forma a a Integrando esta tiltima ecuacién tenemos (1+s°)¥"(s) +sY(s)] = (1+s*)¥"(s)+s¥(s) = a1, que es una ecuacién diferencial lineal de primer orden, La correspondiente ecuacién homogénea es Ys) _ ss Y(s) 1 +8? que tiene solucién y,(s) = —Se. vies Haciendo variar Ja constante c = c(s) y reemplazando obtenemos Jo que implica ds Wo) = af Faw te a ln(s + V1+s?) +e. Por Io tanto 3) . Ye) = (stv) | _e vi+s? Vi+s Ejercicio 7.9.4 Resuelva usando transformada de Laplace la ecuacién y" + 4y = FO. y(0) = y(0) = 0. donde f(O= 4 153 { 0 00. Entonces si Y(s) = L(y(¢))(s). aplicando transformada de Laplace a la ecuacién diferencial obtenemos 3s41 5 (s? +4) ¥(s) = z e* Esto implica Bs+1 sy "0 = 3TH y luego 3st] ye ct 3s) (p) y(t) = £ ay Je Escribiend scribiendo Sit A,B, Cobo s?(s? +4) s s std? se obtiene 3 1 3 1 A=5, B=5, C=-j, D=-3 Asi Stl 31,41 8s 1 4s 4s? 4s?+4 444 (2 +4) ~ = (St) y 2 3 tr BS cosean - bs g(t) = L (<5 TH () = Zt gem Zoos) — g sin). Por lo tanto y(t) = g(t-3) H(t-3) 3 1 3 1. = G + 7-3) = F cos(2(t-3)) = sin —3)) H(t-3). Ejercicio 7.9.5 Resuelva usando transformada de Laplace la ecuacién y+ uf -y=0. yd) =0. ¥@ =1. Solucién. Sea ¥(s) = L(y(t))(s). Entonces cave) = evo) = Liv) - 0] = -Y) - s¥") LY"W)(9) = F¥(s) — sy(0) - YO) = 2¥(9) - 1. Luego, aplicando transformada de Laplace a nuestra ecuacién obtenemos #Y(s) — 1 — Y(s) — s¥(s) — ¥(s) = 0, es decir la ecuacién lineal de primer orden Y'(s) + (2-5) Y(s) = -=. Su solucién es Finalmente como 1 lim L(y())(s) = 0 y lim seF = 00. a debemos tener c = 0. Asf y por Io tanto Ejercicio 7.9.6 a) Demuestre que d + hola) = —A(2) b) Sabiendo que 1 ys) < LUMO) = Se usando transformada de Laplace demuestre que (Jo*A)(z) = Jo(x) - cos(x). Solucién. a) Como rem ( _ (CI pry xeet ES 0 (+ De —1)* 2k: an kt (5) . b) Tenemos Pero Por lo tanto L((Jo* A)())(s) lo que implica (Jo * A) (2) Wl 5 a —— Capitulo 8 Ecuaciones en Derivadas Parciales y Formas Can6énicas 8.1 Introduccién Una ecuacién en derivadas parciales (E.D.P.) es cualquier expresién del tipo FCG. Yy oo oy Ue thay taps o 6 Maas Mays +») = 0 (8.1) que contiene Jas variables independientes 2.Y,.... una funcidn incégnita u y sus derivadas parciales sucesivas tly, Uy. tras « Sin es el niimero de variables independientes. definida en un cierto dominio D C R". Queremos encontrar funciones u(x, y....) que verifiquen (8.1) en D. Estas fun- ciones, si existen, serAn Ilamadas soluciones de la E.D.P. (8.1). Llamaremos orden de (8.1) al mayor orden de las derivadas parciales que aparecen en la ecuacién. Por otra parte (8.1) se dice lineal, si la funcién f es lineal en u y en todas las derivadas parciales de u. Nos concentraremos en las E.D.P. lineales de segundo orden, A este grupo pertenecen las Hamadas ecuaciones de la Fisica Matemética: ecuacién de ondas, ecuacién del calor y ecuacién de Laplace. Cuando, ademds de lineal, la ecuacién esta definida en D C R?, su expresién general es Ta ecuacién (8.1) se consideraré Ajty: + Apthey + Agthy, + Aatty + Astty + Asin = 9(st,y) (8.2) donde Aj = Aj(x,y).i = 1,...,6 son Hamados cceficientes de la ecuacién y g = g(x,y) término independiente. Si g(x, y) =0 en D, la ecuacién (8.2) se llama homogénea. En caso contrario, diremos que la ecuacién es no homogénea o completa. 8.2 Principales diferencias con E.D.O. 1, Para una ecuacién diferencial ordinaria (E.D,O.) lineal de orden n, la solucién general depende de n constantes arbitrarias. En el caso E.D.P., en lugar de constantes, ella depende de funciones arbitrarias. Por ejemplo, para Ury = 0, Ia solucion general es u(z.y) = (2) + h(y), con g y h funciones derivables arbitrarias, Para obtener de ella una solucién particular, hay que agregar condiciones que permitan determinar g y h explicitamente. Esto en general puede ser més dificil que encontrar la solucién general. Por ello se consideran procedimien- tos que permiten encontrar directamente las soluciones particulares que nos interesan. re Para el caso E.D.O. lineales homogéneas de orden n, el conjunto solucién es un espacio vectorial de dimensién n. Para el caso E.D.P., el conjunto solucién es también un espacio vectorial, pero de dimensién infinita. Ejemplo 8.2.1 Consideremos|la ecuacién Ur — ty = 0. Para encontrar sus soluciones, hagamos primero el cambio de variables r=r+y, s=r-y. Usando Ia regla de la cadena. obtenemos Up = UpTy + UsSr = Up + Usy Uy = UpTy + UsSy = Up = Us + y reemplazando en la ecuacién us = 0. La solucién general de esta tiltima ecuacién es u(r.s) = f(r). y por lo tanto, la solucién general de nuestra ecuaci6n es u(t,y) = f(r+y). con f una funcién diferenciable cualquiera. Asf, para cada n € N. tenemos las soluciones: 1, (vty). (ct+y, .... (cw ty)", sen(r+y). sen(2(%+y)), » sen(n(x + y)), cos(r + y). cos(2(x + y)), cos(n(x + y)). ott, erty), enlaty) y todas ellas son L,I. 8.3 Clasificacién de las E.D.P. de Segundo Orden Consideremos la ecuacién AiUrr + Aglry + Aathy, + Ags + Ast + Ast = g(2,Y) (8.3) donde cada A; = Aj(,y),i = 1,...,6, es una funcién continua definida sobre un dominio DC R?. Haciendo una analogfa con Ia ecuacién de una cénica en el plano: Aa? + Apry + Agy? + Ast + Asy + As = 0 diremos que (8.3) es. en D: 1. hiperbdlica si (Az)? — 44,43 > 0 (en D) 2. parabdlica si (A)? — 4A; Ag = 0 (en D) 3. elfptica si (Ap)? = 4A; Ag < 0 (en D) Veremos que, en cada caso, (8.3) se puede reducir a una forma candénica por medio de cambio de coordenadas. 8.4 Formas Canénicas Consideremos Ia ecuacién (8.3) AjUre + Aztry + Agttyy + Agr + Atty + Agu = g(x,y), (8.4) donde A; = Ai(r.y),é DCR. Sean r=r(r,y) y 8 =s(x.y) , funciones de x,y dos veces derivables tales que 6 son funciones continuas definidas en una regién Tr Sx Ty Sy J #0 ed. Supongamos también que la aplicacién (r,s) : D — (r,s)(D) es invertible, y que su inversa (x,y) : (r,s)(D) — D tiene funciones componentes x = 2(r, s).y = y(r,s) dos veces derivables, Entonces Ur UpTy + UsSy Uy UpTy + UsSy Ure Upr (Tz)? + BystaSe + Uss(Sz)? + yer + UsSaz (8.5) Uny Ure ely + Urs (Tey + PySr) + UssS2Sy + Url ry + UsSEY Uy Upp (Ty)? + WpsPySy + Uss(Sy)? + Url yy + UsSyy + Reemplazando en (8.3) se obtiene Bytyy + Botts + Byttss + Byty + Bsus + Bsu = g(r-s) . (8.6) donde g(r, s) = g(x(r,5).y(7,8)) y Byrys) ~ Ay(12)? + Aaraty + Aalry)? Bo(r,s) = 2Arress + Aa(resy + es) + 2AarySy By(r,s) = At(sr)? + Arsesy + Aa(sy)? Bays) = Atrar + Aaray + Aaty + Aare + Asry Bs(7,8) = Aisre + Apsry + AaSyy + Assr + Assy Bo(r,s) = Ae. donde las funciones Aj. ¢ = 1,2 derivadas parciales de ry s en (7.5). Observe que la ecuacién (8.6) es del mismo tipo que la ecuacién (8.3). ya que .6, estén evaluadas en (x(r.s),y(r+s)). y las (Bz)? — 4ByBy = J?((Az)? — 4A1Az) - Supongamos A; # 0 en D y tratemos de determinar r,s como funciones de x e y de modo de obtener, en lo posible, .B; = 0 y Bs =0 en (r.s)(D). Tenemos las ecuaciones By = Aj(ry)? + Aarery + Ag(ty)? = 0 Bs = Aj(8,)? +Ag5,5, + Aa(sy)? = 0. Ambas son de Ja forma: Ailfe)? + Aafefy + Aslfy)? = 0, a(f) +a(g)+a=0 Sobre las curvas f = cte. se tiene o bién = = wy fe = hide+hyly=0 > FL = y obtenemos Ai(o/(2))? = Aa oa) + Ay = 0, (7) cuyas rafces son yoy) — Avt JGR = Tay \ ¥@ = (8.8) Ay - (Ay? = 441s 2A, . Definicién 8.4.1 Las E.D.O, de primer orden (8.8 - 8.9) se denominan ecuaciones caracteristicas de la E.D.P. (8.3). Sus soluciones y(2) = A(x) +e y(a) = do(a) + C2 son Iamadas curvas caracterésticas. Observacién 8.4.2 Si la ecuacién (8.3) es hiperbélica ((A))2 — 4A1Ay > 0), el cambio de coordenadas r(t,y) = y—A(x) { s(,y) = yale) (6.10) verifica J #0 y By = By =0. 8.4.1 Ecuaciones hiperbdlicas En este caso las ecuaciones caracteristicas son reales y distintas, y el cambio de coordenadas (8.10) nos da tys = —Bauy = Byus + Bou + Glr-s). Esta expresién es llamada primera forma canénica de las E.D.P. hiperbdlicas. Si ademas hacemos a=rt+s B=r-s Una — Ug6 = Cilla + cstis + cou + Gla.) « obtenemos que es Hamada segunda forma candénica de E.D.P. hiperbdlicas. Observacidén 8.4.3 Si A; = 0 en D, obtenemos en (8.7) dx dr\? ~4(%) + 4&(@) = y las ecuaciones caracteristicas son de de Bros ~ Ante (Z) =0 cuyas soluciones son r=a, 2 = AxXy)+e. Ponemos entonces v r s = r—oly), y la ccuacién transformada tiene la estructura de la primera forma candnica, Para pasar a la segunda, usamos el mismo cambio de coordenadas del caso Ai # 0. Ejemplo 8.4.4 Consideremos Ja ecuacién de ondas Ut = Clr + A(a.t), con c>0. Tenemos 0, Ap Ap=l. y (Ap)? — 4AiAy 42. Las ecuaciones caracteristicas son de _— dt Te y las curvas caracterfsticas 1 c= F-t+q. ¥ z ‘1 Hacemos entonces el cambio de coordenadas: 1 1 ror+-tl, sa=ar--l. c c Como ryo=t ™m% = 1 s — -b sy — 1 obtenemos y luego 8.4.2 Ecuaciones parabélicas En este caso hay sdlo una ecuacién caracteristica dy An 7 Ay): y una sola curva caracteristica Ponemos entonces con hy, ka constantes tales que el determinante -M(r) 1 (por ejemplo, ki = 1, ke =0). Asi obtenemos Bi = 0 y por lo tanto By = 0. y la ecuacién Uss = —Byuy — Bsus — Bou + G(r- 5) . Hamada forma canénica para la ecuacién parabélica. Observacién 8.4.5 Si A; = 0) en D, entonces Ap = 0 y la ecuacidn ya esta en su forma candnica, Un ejemplo de este tipo de ecuaciones es la ecuacién de difusién o del calor up = Kure + A(z, t). 8.4.3 Ecuaciones elfpticas Las correspondientes ecuaciones caracterfsticas son dy _ Aa, [(A2? = 4ArAs dx 2A, 4G y las curvas caracteristicas: y = az) +d(xjit a y = a(t) —B(xjit+c. Ponemos entonces y— (a(x) + B(a)i) s = y— (a(x) —W(ex)i). Luego a= 5(r+8) = y a(x) 1 B= ars) = —)(x) y la ecuacién nos queda Usa + Use = —Bytla — Bsug — Bou + 9(a,8). Esta expresidn es llamada forma can6nica para las E.D.P. elipticas. Un ejemplo de estas ecuaciones es la ecuacién de Laplace Ure + Uy = 0. Ejemplo 8.4.6 Obtener la forma candnica de Ia ecuacién Arr + Bly + Uy + Ur + Uy = 2. Como (42)? — 44,43 = 25-16 = 9>0 . Ja ecuacién es hiperbdlica en todo Las ecuaciones caracteristicas son dy _ dy dy 1 dy Vy ra y luego, la curvas caracterfsticas son 1 yatta y y= gta. Entonces el cambio de coordenadas: r=y-", 8s produce Fjemplo 8.4.7 Reducir a su forma candniea 2 1 Ure — At ty = gle, con @>0. Como (Ap)? — 44,43 = 16x? > 0. laecuacién es hiperbélica. Sus ecuaciones caracterfsticas son . I ya) = 22 oF TAA tq) = aon, a En consecuencia las curvas caracteristicas son: Hacemos entonces el cambio de coordenadas aye y= 2 r(t.y) = y-2?, s(v.y) = y+a". Entonces t= 22 ry = 1 s, = Qn Por Io tanto Une Uy Tm = —2 Try = 0 Tm = 0 Ser = 2 Sy = 0 Sy = 0 Up = UTrtUsSx = 2x(us — uy) Uy = Uply + UsSy = Up + Us. (Us — Ur} + 20(UsrTr + UssSx — Urrlx — UrsSx) (us — Up) + 220(—Deddgy + Wigs + QWeUUyy — Quy) (Us — Up) + 42 (Upp — Dey + ss) Uppy + UsrSy + Usey + UssSy Urr + 2Uns + Us « Reemplazando obtenemos 2(us — Ur) + 40? (tre = Qtbsr + Uss) AT? (Uy + Qtr + Uss) = i + 2a(us — Ur). es decir 1627 Us) = Pero como 2x? = s—r, se tiene 16(r — s)tyr = 0. 8.5 Ejercicios resueltos Ejercicio 8.5.1. Reduzca a su forma candnica la ecuacién Ure — 2C08(x)Uy, — (3+ ser?(x))tyy — yuy = 0. Solucién. — Tenemos A=, Ay =—2cos(x), Ag = -(3 + sen®%(2)) y como —4A\A3 = 4cos*(x) +4(3 +sen*(x)) = 16 Ja ecuacidn es hiperbdlica. Las correspondientes ecuaciones caracteristicas son dy dy = ~cos| 2, a te cos(a) + ie y las curvas caracterfsticas son —cos(x) — 2, y=-sen(r)+20+Gq. y y=—sen(r) — 27 +e. Por lo tanto debemos hacer el cambio de coordenadas r = ytsen(r)- 22, s = ytsen(x) +22. De esta forma ry = cos(t)-2 ry = 1 Sx = cos(z)+2 sy = 1 y Uz = Upts + UgSz = (cos(x) — 2)uy + (cos(x) + 2)uty Uy = Uply + UsSy = Up + Us Ure = —Sen(sr)(up + Us) + (cOs(x) — 2)? tty, + 2(cos? (x2) — A)urs + (cos(w) + 2)?u05 Ary = (cos() = 2)uyr + 2c08(x)tups + (cos(x) + 2)uss Uyy = Upe + 2ps + ss « Reemplazando en la ecuacién obtenemos 0 = =(y+sen(x)) (up + Us) + [(cos(x) =? 2Qcos(x)(cos(z2) — 2) — (3 + sen?(2))]tbpy + [2(cos*(x) — 4) — 4cos*(x) — 2(3 + sen?(x2))/urs + [(cos(x) nr = 2cos(x)(cos(x) + 2) - (3 + sen?(x))]UUss « Esta ecuacién se reduce a 1 = Flr + 8)(uy + us) = 160s Luego su primera forma canénica es (7 +5)(Ur + Us) +32tys = 0. Ejercicio 8.5.2 Reducir a su forma candénica y obtener la solucién general de la ecuacién W2qy — Way + yy + %y = Solucién. Tenemos A P, B=-&, C=1y B-440 =0. Luego la ecuacién es parabélica y su ecuacién caracterfstica es dy dx ~ La correspondiente curva caracteristica es y = -Wn(x) +c. Consideramos entonces el cambio de coordenadas t Por lo tanto y 1 2p = Sete + 25*S2 = = + Us, z 1 1 2 lee = ayn + Sl + At + Sas a a xv By = apeTy + 26+ 5y = ry 1 fy = Ti + Frey Reemplazando en la ecuacién abtenemos oy 1 1 2 1 O(- a5 + Ter + Fars + 2s) — 2u(s Arr + 27s) + ar + a = 0, que se reduce a Vis = 0. Luego la forma candénica es Zs = 0, y la solucién general en las coordenadas r,s es 258) = sf(r) + a(r): con f.g — funciones dos veces derivables cualquieras. De esta forma nuestra solucién general es Ary) = xf(y + I(x) + gly + In(x)), con f.g — funciones dos vecesjderivables arbitrarias. Ejercicio 8.5.3 Determine el dominio en R? en el cual la ecuacién diferencial es eliptica, En ese dominio obtener su forma canénica, Solucién. EI discriminante de esta ecuacién es BP — 4AC = 4y, y luego es elfptica para y <0. Consideremos entonces y <0), Las ecuaciones caracterfsticas son dy _ dx ~ y las curvas caracteristicas son y luego nuestro cambio de coordenadas es 1 2 _ i _ 3 a= gts) (-y)?, B= a s)= 52: y 1 Q = 0: a = —§ (-y)? 6b, = -$: 6, = 0 Tenemos 3 Ur = adr + Use = -F Us, = uss Ure = 7 Mass 3 1 Uy = Up dy + Us Sy = 5 (-y)? Ua. 3 -. 9 ty = 5 (-y) Fa + 5 (HY) toa 4 Finalmente reemplazando en la ecuacidn obtenemos 9 3B, 9 13,04 8 uss — 2 (-y)-? 2 Si (-ytu = Fy tos — 7 (-u) Ma + ZY Moa Hay 5 ( Y)? Uo = 0, que se reduce a 9 ZY Maa + Wa) = 0. Luego la forma canénica es Una + Use = O- Ejercicio 8.5.4 — Considere la ecuacién hiperbdlica: tune + fu =m, t>0. a) Redizcala a su primera forma candnica. b) Encuentre su solucién general. c) Encuentre la solucién particular que verifica 6 U0.) = 5. u(0.0) = 1. Solucién, a) Tenemos A=1, B=0, C=-t! y B=44C = 40, Luego sus ecuaciones caracteristicas son de —=+? dé y sus curvas caracterfsticas son 2 rote. 3 Asf el cambio de coordenadas es 2 2 Por Jo tanto ry = di me = Oy tee = 02 te = OF TH = 2 1 0 Sx = So = -Ps Se = Sot = 0; Se = —2t y Ur = Unt Te + st 82 = Ue Ll + uss l = Uy + Use Ure = (Upr + Uys) Te + (Ure + Use) +82 = Urry + Qtlys + Use» Uy = Ups ry + Us Sy = Up Pb us (-P) = Plu, — us), A (Urs — Uss) « ua = 2p — Us) + BCurr = thes) = Reemplazando en la ecuacién obtenemos Attu. = 0, y como ¢ > 0, la forma candnica es Ws = 0. b) En las coordenadas 1, s, Ia solucién general es u(r,s) = p(r) + q(s), con p,q funciones dos veces derivables cualquieras. Por lo tanto, en las coordenadas 2, ¢ la solucién general es u(x,t) = p(x + 5 + g(t — 3) con p.q_— funciones dos veces derivables cualquieras. c) La condicién u(0,t) = §, implica 3 oG) + ale y la condicién u,(0,t) = 1, implica o bién, multiplicando por é& 8 2 eye) + Pq) = B. P(g) + Fa'(-3) Integrando esta tiltima ecuacién con respecta a ¢ abtenemos Entonces poniendo v = 4, obteniémos el sistema no) + (0) = 8 pv) - g(-v) =u +e cuya solucién es ve) = gl" ++ av) = ERP = -c Por lo tanto. tenemos pe) = 38 +ete, y lo que implica que nuestra solucién es , 2 2 ur.t) = pe + 3) +4 - 5) App par ere + 2 3 3 1 i 2 3 3 Ejercicio 8.5.5 Reduzca a su forma canénica la ecuacién Trp — 2 Uy — Up = 0 7 >0, y encuentre la solucién que satisface las condiciones iniciales u(e,0) = G2+1)2, w(z-0) = er? Solucién. Tenemos A=r, B=0, C=-223 = B?-4AC=8r'. y la ecuacién es hiperbdlica. Ecuaciones caracteristicas : Curvas caracteristicas : Cambio de coordenadas ur = V2x(uy — us) UW = Ur tus Ux = V2(up = Us) + 22? (Upp — Qelys + My) Uy = Urr + Urs + Uss 5 y reemplazando en la ecuacién obtenemos V2exe( uy = Us) + 21 (py = Qttys + Uyy)) = 20° (thyy + Qys + Ugg) — V2r(uy = Us) = 0. Esta se reduce a 8 ups = 0, y luego su forma candénica es Ups = 0. La solucién general en la coordenadas r,s es entonces u(r.s) = p(r) + a(s). con p,q funciones 2 veces detivables cualquiera. Asf nuestra solucién general es (ay) = ply +2") + aly — con p,q funciones 2 veces derivables cualquiera. Debemos tener u(x,0) = pS. «2 2B) ta-Zat) = UF, y a 4 1,0) = we) + ==r. u(x, De) +d we Multiplicando esta tiltima ecuacién por _ x obtenemos Vay (S 2 a) + V2aq (Bey - = 4x3 e integrando Fa) = ‘Tenemos entonces el sistema p(x?) + o(-22?) pi (Ba?) — g(a) = cuya solucién es Por Io tanto si hacemos vu = 2x?. tenemos plu) = F(vdut 1? +204), g(u) = Five -1P-20. Luego nuestra solucién es u(r,y) [v2(y + ee) + 1P + Ay + Zaoy + V2 ve 2) — yp - 2 v2 a2 yr) que se reduce a u(x, y) = 1428 +294 Se . Ejercicio 8.5.6 = Reduzca a su forma canénica la ecuacién ul Use ~ Oty + F ty + 2 = Encuentre ademas su solucién general y la solucién particular que verifica u u ) = sinfy) = —,7 in (— u(0.y) = sinty), u(e,0) = me + sin (Fe) : Solucién. Tenemos u 7 => B - 4AC = 36 - 11 = 25, A=1, B=-6, C= y por lo tanto a ecuacién es hiperbdlica, Las ecuaciones caractetfsticas son ty A dy a aw > Y @ Te y las curvas caracterfsticas son y=-tr+a y= tree y= 3 1 y y= 2 2+ Luego nuestro cambio de coordenadas es - pete wey etl regtde, soyt ie, Como my = 4, m= 1 Sr sy = 1 tenemos Up = Uy Tr + UsSp = Flue + 11us) Uy = UTy + Ussy = Up + Us 1 Uae = qlur + 22urs + 121Uss) 1 Uy = Glee + 12t¢s + 1195) Uy = Upp + ps + Uss« Reemplazando cbtenemos 1 ~2Bire + Els = 1) = 0. Luego nuestra forma canénica es 1 me Integrando con respecta a s obtenemos 1 (3 , Up = a(S = 1s) + h(r), ¢ integrando a continuacién con respecto ar 2 urs) = a(S - S)e [io )dr + G(s Luego nuestra solucin general en las coordenadas r,s es Us = u(r.s) = rs(s- 7) + H(r) + G(s). 1 250 y en las coordenadas 2, y 1 1 il 1 a u(t.y) = ar(u+ $1) (v+ $1) + A(y + 31) + a(y + 2) : donde H,G son funciones derivables arbitrarias. La condicién u(0,y) = sinfy) => Hy) + Gly) = sin{y), wot} = Boe + sn(s) <= a(4e) +0 (Bs) = on(e), Por lo tanto G(x) = sin(x), y H(x) = 0, y la solucién particular es ul . ul u(ey) = a yt ge y+ za) +sn(y+ se). Ejercicio 8.5.7 Reduzca a su forma canénica la ecuacién Ure + 2ew Ug, + 27E7* Uy = (wey. Encuentre ademés su solucién general y la solucién particular que verifica u(O,y) = 2y + 2, u(09) = y + 1. Solucién. — Tenemos Az=1, B=%", C=re* => B-44C = 0, y luego la ecuacién es parabélica. La ecuacidn caracteristica es dy -2 qa y la curva caracteristica es y= -(l+aje™ +c. Consideramos el cambio de coordenadas r=yt+(+aje", saa. Entonces ry = —xem* Sy = 1 Por Io tanto Up = re uy + us uy = & Ur = (@— Vey, + 277 Uy, — Wwe Uys + Use Uny = UE Une + Us Uy = Ure Reemplazando en la ecuacién obtenemos la forma canénica Uss = 0. De esta forma la solucién general en las coordenadas r,s es u(r.s) = sf(r) + g(r), y en las coordenadas .r,y es u(x.y) = xf (y + (1 + ae) + o(y + A + ae). donde f,g son funciones dos veces derivables arbitrarias. Finalmente imponiendo Jas condiciones obtenemos 2y+2 = u(0.y) +1) y+ ur(0.9) +1). Esto implica oy) = 2. fy) =y. Luego nuestra solucién particular es u(a.y) = (© + 2)(y + (+ xe"). Capitulo 9 Ecuacién de ondas unidimensional 9.1 Deduccién de la ecuacién de ondas Suponemos que una cuerda flexible se tensa entre dos puntos del eje x, digamos 2=0y 2=1, Lacuerda se deforma a una cierta curva u= f(x) en el plano cu y luego se suelta, Figura 39 Problema: —Determinar la curva u(z,f) en que se deforma la cuerda en un in- stante ¢ posterior. Hipédtesis Adicionales: ¢ La vibracién subsiguiente es transversal; es decir cada punto de la cuerda tiene coordenada x constante; de modo que su coordenada u depende solo de « y del tiempo. De esta forma el desplazamiento de la cuerda a partir de su posicin de equilibrio viene dado por cierta funcién w = u(z,é) y sus derivadas: du Ut velocidad de la cuerda Pu ae aceleracidn de la cuerda Consideremos el movimiento de un pequefio fragmento de cuerda de longitud Az. Si la densidad de masa lineal es m = m(zr), Ia masa de este fragmento es mAs. y la segunda Ley de Newton dice que la fuerza transversal F que acttia sobre él es ®u = eu, ) P= mace (9.1) Como la cuerda es flexible, la tension T = T(x) en cualquier punto esta ditigida a lo largo de la tangente y tiene componente igual a T sen (9). Figura 40 © Suponemos que el movimiento de la cuerda se debe solo a la tensién. Luego F es la diferencia entre los valores de T sen (8) en los extremos del fragmento, es decir, F = A(T'sen (6). As (9.L)queda de la forma: Pu Ol? Si las vibraciones son relativamente pequefias. de modo que 6 es pequefio, A(Tsen(6)) = m Ax (9.2) sen (9) es aproximadamente igual a tan(9) = om Reemplazando esto en (9.2) obtenemos: Ou AT) 2 De’ _ , Fu ) Bx "OP (93) y haciendo Ax > 0 a (du Pu 2 (pd) = . A) Ox (r =) mae 0.4) © Asumimos que m y T son constantes, obteniéndose Pu _ Fu a =\= “oe = oF = Vin que es la ecuacién de ondas unidimensional. Buscamos una solucin u(x, ¢) que satisfaga las condiciones de frontera u(0.t) = 0. u(l.t) = 0 (extremes fijos) y las condiciones iniciales u(a.0) = fle) y M(a,0) = g(r). Observe que si g(x) = 0. entonces la cuerda esta en reposo al momento de soltarla y su forma viene dada por el grafico de la funcién y = f(x). De esta forma nuestro problema es: ue = 2 Une O R funciones continuas por partes y dos veces detivables. Entonces cualquier funcién de la forma 1 wat) = 5 1 pete Feta) + Pen el+ x f G(nar 2c Samet con F(x) = f(x) y G(x) = g(x). para 0< a <1 verifica Un = Cle u(x.0) = f(x). w(x.0)=g(x) O RB. se dice periédica de periodo T, si T es el menor nimero positivo que verifica F(x -+T) = F(x) para todo «€R Observaciones 9.2.3 Los siguientes resultados son inmediatos. 1. Si F: RR es una funcién derivable, entonces que F sea impar (resp. par) implica que F’ es par (resp. impar). 2. Para definir una funcidn periédica de periodo 27, digamos F , basta definirla en el intervalo [-T,T] y especificar que verifica F(x +27) = F(x), para todo rE R. 3. Suponga que F :R > R es periddica de period 27°. Entonces F(-2) = -F(2) Vre(-T.T) = F esimpar, F(-n) = F(x) Vee (-T,T = F espa. Definicién 9.2.4 Una funcién F : R > RB se dice impar (resp. par) en k si la funcién trasladada en k F:ROR F(x) =F(e+h). €s ampar (resp. par) Luego F : R > Res impar (resp. par) en k si F(-r+h)=-F(e +h) (vesp. F(-r+h)=F(r+h)) VoeR Observacién 9.2.5 Si F es impar (resp. par ) y periddica de periodo 2/, entonces F también es impar (resp. par ) en L. En efecto, en el caso impar F(-x +1) = F(-x+1-2l) = F(-v-l) = -F(+)). y en el caso par F(-r+l) = F(-a+l-2l) = F(-x-l) = F(x+d. 9.2.2 Problema de ondas homogéneo con extremos fijos lremos el problema (9.5). Teniehdo en cuenta las proposiciones (9.2.1) y (9.2.2), debemos considerar fmciones FG: R > R continuas por partes y dos veces diferenciables. que verifiquen F(x) = f(x) y G(x) = g(x) para todo 0 0. Afirmacién. — Esta condiciones se verifican si F y G son funciones impares y periddicas de periodo 20. En efecto, en este caso 7 u(0.t) = 4{F(ct) + F-a)) + + [ Gar = 0, 3 % doa y 1 et whe) = SRC Fe) + Ua) + ef Glide = SFU+ a) + F(-l-ct) + “hee G(s+Jds = 0, ya que la funcién h(s) G(-s—1) = -G(s+0 G(s +2) también es impar (h(-s) = G(-s +1) = =h(s)). Definicién 9.2.6 Dada h : {0.1 + R continua por partes, se llama extensién impar de periodo 2/. de h, a la funcién peridédica de periodo 21 hy: R + R definida por: A(x) si O0. donde f, y gj son las extensiones impares deperiodo 2 de f y g respectivamente. Ejemplo 9.2.7 Considere una cuerda de longitud 7. fija por sus extremos, que parte del resposo con un perfil inicial dado por la funcién f(c) = 22. Obtener la forma de la cuerda en el instante ¢. Nuestra ecuacién es Uy = Cura u(e.0)=2(a—m). u(0,6 Entonces 1 ur) = slfile+e) + file-a)] donde f,(x) es la extensién impar de perfodo 27 de f(x) = 2?. Es decir -P si -a R tales que que las funciones F//-1,1] y G//=I,1] sean pares, y que F(x+2l) = F(x), Definicién 9.2.8 Dada h : G(r +2l) = Ga), VreR (0.2) + R continua por partes, se Wama extensién par de periodo 21. de h, a la funcién periddica de periodo 21 hy: RR definida por: hi(a +21) = h(x) O0, encontramos la solucién 1 petet ued) = difie+e) + fle-ay+ bf” oar, 3 2 Srna donde f;. 9; son las extensiones impares de periodo 21 de fy g, respectivamente. Los desarrollos en serie de Fourier de f, y 9; son: fle) ~ S> bysen (ME) con m=? [ fls)sen (8) ds ra aia)~ So Basin (HE) con By =F f olsison (HE) Reemplazando las funciones fj y gi. por sus respectivas series de Fourier tenemos I 2 & — = “ia 8 8 = Se boos (FA) sen (M2) + 5B, sn (4) sen (Mx) Por Io tanto u(2,t) = > [my cos (1) + By sen (“ery sen (42) . Cada témino de esta serie puede considerarse como una onda estacionaria. Por ejemplo: Param = 1, [by cos (“2t) + Bison (#0)] sn (Ex) . t a . . es una onda senoidal (sen Te ) multiplicada por una amplitud que varia con el tiempo. Z™ Figura 42 . Qn . . Para n = 2, tenemos una onda senoidal sen (Fe por una amplitud que varia con ‘ on l el tiempo, Aquf existe un nodo en x = que nunca se mueve, Figura 43 Para n = 3, tenemos dos nodos y la figura Figura 44 Bn general el n-ésimo término es una onda senoidal sen (4) por una amplitud que varia con el tiempo, con n — 1 nodos. Asf la solucién u(x, £) puede interpretarse como la superposicidn de infinidad de ondas estacionarias. También existen ondas viajeras asociadas con Ja ecuacién de ondas. Estas surgen en forma natural de la solucién de d’Almbert de Ja ecuacidn de onda para una cuerda infinita. Problema: — Resolver: Un = Cllre x.te Rt u(x,0) = f(x) w(x.0)=9(x) x eR Claramente la solucién de d'Ambert es 1 st) = wel) = 5 [f(@+a) - fe-ct)] Por ejemplo, supongamos que atl -1<2<0 yque f(x) = ¢ -«+1 0<@<1 0 |aj>T. g(a) = 0. Entonces u(z.¢) es la suma de las ondas viajeras 1 1 , af la + et) y afl - ct) Para ¢ =0 estén superpuestas = 3 T Figura 45 Cuando ¢ aumenta, las dos ondas se separan entre si con velocidad 2c. Las Figuras 46 y 47 corresponden a los casos t = + y ¢ = 2, respectivamente. s a ° tT Fs Figura 46 a Ber 7 3 ni 5 > Figura 47 9.3 HEjercicios resueltos Ejercicio 9.3.1 Una cuerda tensa de guitarra de longitud 4 esta fija en sus ex- tremos. Con los dedos la desplazamos una pequefia distancia 0.3 en el punto x = 3, la soltamos con velocidad g(x) = 2¢ y la cuerda comienza a vibrar. Encuentre el desplazamiento vertical u(z, 2) de cualquier punto x para cualquier instante ¢. También calcule u(5, $) para c = 2. Solucién. — La correspondiente ecuacién de ondas es Pu Pu aE OTR: O0 uO.) = ud.) = 0. t>0 _ _ f Ole 0<2<3 ux.0) = f(z) = { 0.3012 30, ycalcule u(2,8) Solucién. Sabemos que - 1 pet ule.) = Sthle+0 + hle-o) + 4 [ gplr)ar. rot donde f,,gp son las extensiones pares de periodo 2 de las funciones f(x) = 0 y g(x) = 23. respectivamente. Luego si S-1<2<0 a . si O 0. u(3 53) . Solucién, —Tenemos ula) = iF (+4) + F(c-4/) + con F,G las extenciones par en x = 0 e impar en x = 2 de periodo 8 de las funciones f(x) = 2° y g(x) = 2x, respectivamente. Por lo tanto y calcule -40 3 5 u(x.0) = 1+ et zoen(z). (2,0) = 2 ycalcule w Solucién. Hagamos un cambio de funcién de la forma u(x,t) = u(x,t) + H(z). Entonces H(x) debe verificar 4H"(x) + sen(x) = 0 3 1 H(0) = 1 = Ha) =14+ mit rent). HG) = 2 " Entonces u(x,t) debe ser solucién de =r Por Io tanto 1 1 pee u(x,t) = s[F(w+2l) + F(w—20)] + > G(r)ar. 2 4 Ji-2 con FG las extenciones impar de periodo 7 de las funciones f(x) = sen g(x) = 22, respectivamente. De esta forma (x) y Por Io tanto u(3,2) 2H (2) = 14 224220. 23 2 Ejercicio 9.3.6 — Demuestre que cualquier problema no homogéneo de la forma 2n2 4 Ue - Clr = 48) O2) + P(5- )| - (=) + °(@)] E63) (-2) = jp-4) = 0. Luego Ejercicio 9.3.8 — Resuelva Ue = Ure + te u(0,t) = u(m.0)=0 u(x,0) = sent ; u,(x,0) =5sen(2x) — 3sen(5z) . usando un cambio del tipo: u(x. 1) = v(x. 1) + Pa(x)é. donde P3(x) es un polinomio cibico. Solucién. Poniendo P(x) = Ar? + Ba? +Cr+D y u(x,t) = v(x,t) + Pa(x)t, obtenemos Ue = Ue Ure = Use +t(6Ar + 2B) = mie =ty te Para que la ecuacién sea homogénea ponemos B= 0 y A=—4. Entonces Pa(x) = -he +Cx+D, y como u(0, t) = u(0,t) + P3(0)t = v(0.t) + Dé a + cmt para mantener las condiciones de frontera ponemos D =0 y C= bn. De esta forma tenemos. u(m.t) = o(a, 0) + Palm t = Ula.) + Pa(e) = -he + Arr, Examinamos ahora las condiciones ini¢iales u(z.0) = v(a,0) u(t.0) = ty(a,0)+ P(x) = t¢(x,0) — $22 + bn2e, sen 5sen(2r) — 3sen(5.r) Luego v(x.) =senx y uy(1r,0) = Ssen(2z) — 3sen (Se) + doe? -). Por lo tanto nuestro problema se reduce a: Ce v(0,4) . v(x.0) = sen; ty(x,0) = 5Ssen(2r) — 3sen(5r) + Lar(a? = x?) Como sen(x),sen(2r),sen(5zr) son impares y de periodo 2m y la funcién g(2) = La(a? =n?) es impar, si consideramos la funcién G definida por G(a) = g(t).-m0 u(t.0) = 2-2 u(x,0) = 22 +1, 0 -g(4t2) = -4ta-1 ss -40 u(0.t) = u2(3,t) = 0, £>0 u(x.0) = 0 wla.0) = ay - at O0<2<3 ycalcule u (3.3). Indicacién. Haga un cambio de funcién del tipo u(x,t) = u(x.t) + tA(x). Solucién. Con el cambio sugerido tenemos Un = Ue Y— Alge + 2x? = Aver + 4tA"(x) + 2tx?. Luego debemos tener = Ala) = -4r 4 Bote. Ademés 0=u0.t) = 001) +1C = C=0. y 0 = w(3.0) = (3.0) + (73° + B) = p=}. De esta forma el cambio de funcidn es u(a.t) = o(x.t) +b Por otra parte u(x,0) = u(2,0) + v(z.0) = 0 uf(t,0) = 2. O<4 <3 Asi pete v(a.t) = = G(s)ds 4 Sono donde G es la extensién impar en 0 y par en 3 de periodo 12 de la funcién e/, e Por lo tanto , -(6+2) -6 <4 <=3 Giz) = 4 a -3<90<3 G(x +12) = Gia) VreR. 6-2 3<2<6 Finalmente 1 2 4 1,)_1 dds - 4 = (53) = 7) Gods = 7 / sds == 0. Ejercicio 9.3.11 Resuelva el problema de ondas wl = Sue, OSS2. 420 u(0.f) = ur(2,t) = 0 u(x.0) = 22, wlx,0) = c+2 y calcule u (48. 4). Solucién. Como c = 3 y J = 2. la solucién es wt 1+3t u(a,t) = 5 [F(w+3t) + F(x —38)] +l, G(r) dr, i 3 donde F.G son las extensiones impares en 0 y pares en 2 de periodo & de las funciones . 10) =H], 9 Wade respectivamente, Por lo tanto -(44+2) 6-2 -4<4<-2 2 _ =x ,_ J w-2 -2<2<0 F(x) 2 G) r+2 0<4<2 (4-2)? 6-2 2O0 y u(ld) + he up(l.t) =0 u(x, 0)= file); (2,0) = f(a). ii) Para 0 N(Q)(M(O) + hy M'(0))=0, WeER* = M(0) + M0) =0. Luego M(x) debe ser solucién de la E.D.O, M(0) + tu M'(0) M(a) + hoM'(a) = M(x) + AM(x)=0 y { (amado problema regular de Sturm-Liouville) donde a = / en el caso hiperb6lico y parabélico, y a = a en el caso eliptico. Observe que la correspondiente ecuacién caracterfstica es: k2++=0. Observacién 10.1.2 Para poder aplicar separacién de variables, existen requisitos que deben cumplir las condiciones de frontera y la regién donde est definida la EDP, i) Las condiciones de frontera deben poder definirse sobre variables independi- entes en forma separada. Por ejemplo, si la regién es el disco de radio a centrado en el origen. su frontera en coordenadas cartesianas es x? + y? Entonces las condiciones de frontera = ) FD os 10:0 no definen condiciones separadas. Sin embargo, en coordenadas polares (7.0), la regién es 7 0, n €N, las soluciones de estas ecuaciones son: " ° . Suponiendo Nat) = dn cos(c VM t) + Ba sen(cVXn t) (caso hiperbdlico) Nall) = an e7knt (caso parabélica) Naly) = an eV Y + by ewVrY (caso elfptico) . Entonces, para cada n € N, tp = My+ Np verifica la E.D.P. junto a las condi- ciones de frontera homogéneas. Para encontrar la solucién que verifica, ademas, las condiciones que faltan, consideramos formalmente la serie x x w= Vu = SoM, nat hat en las que determinamos las constantes aq, bn que aparecen en N;, de modo que w cumpla las condiciones requeridas. 10.2 Problemas de Sturm-Liouville mas frecuentes Proposicién 10.2.1 Para | > 0 considere el problema M"(x) + AM(w) = 0 M(0) = MQ) = 0. Entonces los correspondientes autovalores son \, = "= y las autofunciones M,(«) = sen(2# a), y estdn definidas para todo raimero natural n> 1. Demostracién. Si \ <0, la solucidn general de M"(x) + AM(x) = es M(x) = cpeV** + cge-V**, La condicién M(0) = 0 implica ce = -c1.y M)=0 3S a +e") -0 = a=0, por lo que no hay autofuncionés.asociadas a < 0, Tampoco tenemos autofunciones para 4 = 0, ya que en ese caso Ja solucién general es M(x) = cy + cox. y las condiciones de frontera implican cr = co = 0. Sea entonces A > 0. Ahora la solucién’general es M(x) = c1 cos (vxz) + esen (vXz) : y la condicién M(Q) = 0 implica cy = 0. Luego M(a) = ensen (VX) vyla condicién M(2) = 0 implica cosen (VX: =o = VAl=nt, n=Ld.. we = A= n= 1.2... De esta manera Ios autovalores son A, = % y las autofunciones M,(x) = sen (3) , y estén definidas para todo niimero natural n> 1. Proposicién 10.2.2 Para | > 0 considere el problema M"(x) + AM(x) = 0 M'(0) = Ml) = 0 Entonces los correspondientes autovalores son A, = "SE y las autofunciones My(x) = cos (£2), y estén definidas para todo niimero natural n > 0. Demostracién. —Veamos que no hay autofunciones asociadas a \ < 0. En efecto, en este caso la solucidn general es M(e) = ae? + @e™,” y lacondicién M'(0) = 0 implica c2 = c1. Por otra parte M()=0 = av=ar [eo - ev) =0 => q=0. lo que implica M(x) = Sea entances \ > 0. Ahora la solucién general es M(x) = ¢ cos (vez) + esen (v¥2) : y lacondicién M'(0) = 0 implica c: = 0. Luego M(x) = cr cos (VX 2) yla condicién M'(2) = 0 implica a VNsen (VXI) = 0 = VAP = nm. n= 120% 2 92 ner Ss =4R. n= 12... De esta manera Jos autovalores son \, = 4 y las autofunciones M,(2) = cos (42) . y estan definidas para todo ntimero natural n > 0. Proposicién 10.2.3 Para | > 0 considere el problema M"%x) + AM(x) = 0 M0) = MQ) = 0. Entonces los correspondientes autovalores son X= oe y las autofunciones Myx) = sen (Supt 0). y estdn definidas para todo nmero natural n> 0. Demostracién. Como en la demostracién de la Proposicién 10.2.1, es sencillo verificar que no hay autofunciones asociadas a A <0. Sea entonces > 0. Como la solucidn general es M(x) = ¢ cos (VN) + cgsen(V¥2) « la condicién M(0) = 0 implica c, = 0. Luego M(x) = cpsen (vx 2): yla condicién M'(J) = 0 implica exV¥ cos(V¥0) = 0 = VXI= Ete n= 01,2 (2n+ 127? 4 = )\= n= 0.1.2... (ony? 2 De estia manera Jos autovalores son \, = S&te= sen (SNe r) , y las autofunciones M,(x) = stan definidas para todo ntimero natural n > 0. Proposicién 10.2.4 Para | > 0 considere el problema M(x) + AM(x) = 0 M0) = MQ) = 0. (2n41)? 2 Entonces los correspondientes autovalores son Ay = S44E™ y las autofunciones M,(x) = cos (232) , y estan definidas para todo mimero natural m > 0. Ejemplo 10.2.5 Usando separacién de variables resolvamos la ecuacién de ondas un = Cuyx, O0. Ponemos u(z,¢) = M(x)+N(t) y obtenemos las ecuaciones M"(r) + \M(c) Nb) + 2XN(O) 0 0, con —A constante de separacién. Para que se cumplan las condiciones de*frontera, M(x) debe ser solucién de M"(x) + AM(x) = 0 M'(0) = MQ) = 0. De esta forma los autovalores son Mn = n=0,1,2.0 y las autofunciones son M,(x) = cos 2). n=0,1,2, Sustituyendo A por An, en la ecuacién en ¢ se obtiene para n= 0,1,2,... 2q? NNO) + re Nall) =0 cuya solucién general es No(t) = 5(@ + det). y i 2 Nall) = en cos + dnsen( 1 1), Vn=1Q..- Entonces para cada n = 0.1,2,.... los productos Une, 6) = Mn(x) + Nat) son soluciones de . Ue Clare u,(0.t) = ur(l,t) = 0. Consideremos Ja solucién formal x u(a.t) = > Un( rt) L , nn nn 3 (Co + dot) + Ya cos EH + dy sen(—— 7 <1) cos(=- 2t) Debemos tener entonces & y= a f(x) u(x, 0) Z + Yoon Es g(t) = w(x.0) = & + 3 dn 00 para todo O<40. Solucién. Pongamos u(z,t) = M(x)N (8). Reemplazando en Ia ecuacién, sepa- rando variables e imponiendo las condiciones de frontera se obtienen las ecuaciones N'(t) +(1+A)N() { M(x) + M(x) = 0 M'(0) = M(@) = 0 Entonces los correspondientes autovalores y autofunciones son A= (es) m4 =oe(fo9)) para todo n> 0. Reemplazando en la ecuacién para la variable ¢ obtenemos Ni + : + (5+) Nit) = 0. alt) = ne s+"). Tenemos entonces la solucién formal cuya salucién es (oe) u(z,t) = Vane cos ((5+») r) . n=0 Imponiendo la condicién inicial tenemos = 1 1 = woo) = Seam (240). me Por lo tanto = 2 f'an((+n)2) ar = 2-2 ai((L4n)2) /" m = Ff s((5tn)a jar = as stn) /* = 44 wl, = 4c" ~ Tem aT) > RIF an y la solucién buscada es won crcl) (Lane), n=0 Ejercicio 10.3.2. Para a > 1 > 0), utilice el método de separacién de variables para obtener una solucién de Pu. du 4 @u je tem tusoss O0, u(l.t) = 0.t>0 1, w(t,0) = 0, O0<2 1. Reemplazando en la otra ecuacién . obtenemos para cada n > 1, 0 0 2a? NM) + 2N4(t) + (1 +2 = ) N,(0) =0, cuya ecuacién caracteristica es 22g? +a (1475) Como el discriminante de estia ecuacién es rena? rena? 1= (14 z )=- Bz 0 u,(0,0) = u(t) = 0, t>0 u(z,0) = sin(t), O 0, Reemplazando en Ia ecuacién para la variable t obtenemos t\7) Ni(t) + ! + (+") N@) = 0, cuya solucién es NA) = nel? (s+0)). enemos entonces la solucién formal wins) = Seng CB) an (Lan). Imponiendo la condicién inicial tenemos sng} = wed) = Seyce (En) 2) sin(2) cos ((3 + n) :) de if =(G) ee L(G) (@+2n) (I-20) Por lo tanto a = Alm ATE Ate y la solucién buscada es wei) = BS pet (FY) on (Lan). n= Ejercicio 10.3.4 Usando el método de separacién de variables resuelva el prob- ema WW +u=0, O0 u(0.1) = u,(7,t) 1>0 u(.0) = 2(m—2), OSa 0. Reemplazando en Ia ecuacién para la variable ¢ obtenemos Ni + : + (+)] Nit) = 0. y,() = wel! (z+): ‘Tenemos entonces la solucién formal 1 \ x -(14(5+n} Je 1 u(a,t) = Pane sin ((5 + ") ) : Imponiendo la condicién inicial tenemos x(m—2) = u(z,0) = Yovsin((}+2)2) : = cuya solucién es Por Io tanto a ef wo (($+»).) a 2a ((fo0)) + rh [e200 (be) ae Papeete alee) el = gatas rat (9) 8 4 _2n(- 1)" (1+ 2n)r 1+42n (1+ 2n)? 8 4 7 ~ TF inpa (im ~ m1"). Luego la solucién buscada es = 1 4 < (12? [a ~ Capitulo 11 Ecuacién del Calor 11.1 Deduccién de la ecuacién del calor Para establecer la ecuacién del calor unidimensional necesitaremos los siguientes principios fisicos: a) El calor se desplaza de las zona calientes a las frfas. b) El ritmo a que fluye el calor a través de un érea es proporcional al area y al ritmo de cambio de temperatura respecto de la distancia en una direccién perpendicular al area. El correspondiente factor de proporcionalidad k es llamado conductividad térmica de la sustancia. La cantidad de calor ganado o perdido por una cuerpo cuando su temperatura cambia, es decir la variacién de energia térmica, es proporcional a la masa del cuerpo y al cambio de temperatura. El factor de proporcionalidad c se Hama calor especffico de la sustancia. 2 Consideremos una varilla cilfndrica delgada de area transversal A, cuya superficie lateral esta aislada térmicamente. OD “ Figura 48 Delgada significa que tenemos temperatura uniforme en cualquier seccién transver- sal del cilindro. Luego la temperatura w depende solo del tiempo y de la posicién de dicha seccién; es decir w = w(t.t). Examinemos el ritmo de cambio del calor en una fina rodaja del cilindro entre las posiciones x y 2 + Ar. Qo “ Petar Figura 49 Si pes la densidad de la varilla, es decir su masa por unidad de volumen, la masa de la rodaja es Am = pAdx. Si Aw es l cambio de temperatura en el punto x en tm pequetio intervalo de tiempo At. entonces por el principio c): Ia cantidad de calor almacenada en Ia rodaja durante ese intervalo de tiempo es AH = cAmAu = cpAArAu. Jo que implica que el ritmo de almacenamiento de calor es AH Aw = = pAdAr—. Be POST Si suponemos que no se genera calor dentro de la rodaja por procesos quimicos o eléctricos, por ejemplo, la rodaja gana calor solamente por medio del flujo que penetra por sus caras. Esto implica, usando b), que el ritmo a que el calor fluye en Ja rodaja por la cara izquierda es ow KAS EI signo se debe al principio a). En efecto. para que penetre calor por la cara izquierda debemos tener que este decrezca con x (afuera debe estar mas caliente que adentro) y por lo tanto # es negativo en x. De esta forma para tener un flujo positivo por la cara izquierda debemos poner signo negativo. Por las misma razones, el ritmo a que el calor fluye por Ja cara derecha es Ow MAT sae y el ritmo total del flujo es dw ow "AGel rac Ges Por Io tanto Ou Ou Devan Ge], = PAD" hk B/ a7 / _ dw @ Ar : g y haciendo Aa — 0 obtenemos Ia ecuacién parabdlica Ot Oa?” k con @=— cp 11.1.1 Difusién en una barra finita aislada Consideremos wna barra de longitud J y suponemos: - El calor se distribuye uniformemente sobre cada seccién transversal a Io largo del tiempo. - No hay intercambio de calor con el exterior (aislada). - Temperatura nula en los extremos. - Distribucidn inicial de temperatura dada por una funcidn f(x) para 0 < x < 1. En términos mateméticos kurv.) O0 0 (2.1) u(z.0) = f(z), 0<2 M(0) = M(l) =0. Entonces para cada variable x tenemos la ecuacién { Mig) +Mia = MQ) = MQ = oo 0 0 cuyo autovalores son: dy = “22, con n> 1 y cuyas autofunciones son My(z2) = sen(#2 x), para n> 1, Luego, para la variable t, nos quedan las ecuaciones : 2g? NO +kSE NO = 0 paran> 1. cuya solucién general es sigead (REL) N,(t) = dn’ sen(—) Yn>1 Construimos la solucién formal: = Sainte?) nT u(x,t) = SY ulx.t) = dwe Z ‘sen(-) na y calcularemos las constantes an de modo que u(t,0) = f(t) O0 sen*(r) O 1 tenemos los autovalores: Ay, =n? y las autofunciones: Mp(r) = sen(nz). También para n > 1, tenemos Np(t) = ane7*”* y la solucién formal en(nx) . Para calcular Jos a) imponemos la condicién inicial c sen?() = u(e,0) = SP ansen(ne) . Luego se tiene: a, = 2 fF sen?(s)sen(ns)ds = 2§7(1 = cos(2s))sen(ns)ds = 1 fF sen(ns)ds— £ [2 (sen((2+n)s) +sen((2—n)s))ds y paran #2 ty = HQ (ayy 2g [tegen 4. teem] (n242n—4) { 0 sines par eM) eg j aa sines impar. Por lo tanto, como también a2 = 0. 2< 4n? +8n—1 k t —k(2n+: ua,i) = 75 4 Gn + Gn? + In — 3) en((2n + 1)x) es la solucién formal que es en realidad la solucién, ya que f(x) = sen?(x) cumple las condiciones de la proposicién, Ejemplo 11.1.3 Resolvamos wm = Uat2, O<2<1,t>0 r-@, O0, v(v.0) = u(r,0)-14+2?2 = 2-22-1422 = 2-1 e(0,t) = v(t) = 0. Poniendo v(x.t) = M(zx)N(é) y separando variables obtenemos las ecuaciones diferenciales ordinarias M"(x) +XM(x) = 0 M'(0)=MQ@) = 0 Luego para n > 0 tenemos los autovalores \,, = #(L + 2n)?n? y las autofunciones M,(«) = cos(§(1+2n)x). También para n > 0 tenemos N'(t) +AN(t) = Na(O) = anert2m?e, Por Io tanto, ala Yom er team! feos FL 4 2n) 2). a y como - Ym cos(= $a +2n)x). tenemos 1 ay = 2f2(s—Vcos(¥(1 + 2n) s)ds Asi 1 va) =-3 > See hos Fl 4 20)2), n=0 y ebb eos (1+2n)2). Ejemplo 11.1.4 Resolvamos u(a.t) = kuz: + cos*(x), O0, u(v.0) = 3+ e-s u(0,t) = ur(a,t) = Hagamos el cambio de funcién u(x.) = v(v.t) + h(x). Reemplazando en la ecuacién obtenemos u(a.t) = wlr.t) = kuge(x.t) + cos*(x) = K(dy2 (2,0) + h"(x)) + cos?(x) . Luego debemos tener kh"(x) =—cos*(x) => De las condiciones de frontera obtenemos 0 = ul.) = o0,)+h(0) = h(0) =0 0 ur(t, £) u(t thr) => h(n) = 0 Luego nuestra funcién es Ademés al, +2 vot 4k 2k 2k 8k lo que implica 1 (2,0) = Ren). Luego nuestra ecuacién para v es (at) = u(a,0) = v(0.t) = Pongamos v(z,t) = M(x) + N(¢). Reemplazando en la ecuacién y separando variables obtenemos las ecuaciones: M"(x) + AM(x) M(0) = M'(n) 0 5. | M@+ARNO =0. Por lo tanto, para n > 0 tenemos loa autovalores A, = ($+7)? y las autofunciones M,(«) = sen(($+n):x). También para n > 0 N,(t) = agen Eth Ht, De esta forma x = . 1 u(0,t) = SSanen4 Msen((5 +n)x). = 2pm a 1 a = 2 [ S_eesl2oem((5 +n) sds = af <(2s)sen((4 + n) s)d = Hae [/ mesienllg +s = f sen((3 +n) s) +sen((-3 +n) s)ds A taill- cost +n)n)]+ a - cos((-3 +n)z)] 2 ~Skn GI * G9)! 1 1 1 = |. ~ 4k“ (5+ 2n) Zz (2n = 3) ) = 1 1 (4 n}2ktgon((t u(x,t) 2 +m) + m3)! a sen((5 +n) x) y <1 1 wed u(a.t) = RTT qcos(2e) + aR -& 1a Lan? 1 _—_ _—_ e~(d4nyPaty =4n)r). Te SSH . seas +=) 2) 11.2 Ejercicios resueltos Ejercicio 11.2.1. Resuelva du Pu Saott Ot Ox? con la condicién inicial u(x,0) = 3sen(x) cos*(x) —sen*(x) + 2sen(z) para O0. Solucién. Como las condiciones de frontera son no homogeneas, hacemos un cambio de funcién del tipo u(x,t) = v(v.t) + Ar +B. Entonces nos resulta Ue = Ue. Ure = Ure =u.) =v0)+B = B= ates 10 = u(m,t) = o(t.t)+An+5 => A= Ademas 3sen(x) cos? (zr) — sen(x) + 2sen(z) = u(z.0) = v(,0) +245 lo que implica v(a,0) = Bsen(x) cos*(a) — sen(x) + 2sen(x) — : —5. Entonces nuestra ecuacién es Up = re 5_e =-5 v(x.0) = 3sen(x) cos*(x) = sén°(x) + 2sen(sx) — v(0,0) = vm.) =0 Pongamos u(x,t) = M(x)N(2). Reemplazando en lé’ecuacién, separando variables e imponiendo las condiciones de frontera se obtienen las ecuaciones { M"(x) + AM(x) M(0) = M(x) =0 =0 Ni(t) + 2AN(0) = Entonces los correspondientes autovalores y autofunciones son An = 7, M,(x) = sen(nz)). para todo n> 1. Reemplazando en la ecuacién para la variable ¢ obtenemos Nit) + 2PN() = 0, cuya solucién es , Nq(l) = ane? Tenemos entonces la solucién formal Imponiendo la condicién inicial tenemos 3sen (x) cos?(x) = sen®(x) + 2sen(x) — alex =5 = v(x,0) = SY ansen(nz). nt Pero 3sen(x) cos*(x) — sen*(z’) = Ssen(2r) cos() — Been en (2) (1 = cos(2r)) ) = Asen(:r) + Eeen(r) cos(2) 3 1 = glsen(32) +sen(x) 5 = sen(3r) Luego tenemos 20 sen(3x) + 2sen(:x} — 2e- 5 = Sansen(nz). nm Jo que implica - x0 =H0 5 = (ay —2)seul(#)+ agsen(2r) + (ag — 1)sen(3r) +S" a,sen(nz) 7 n=4 Ademés 2 5. _ = fhe, = — Zt — 5)sen(nx)de => [GE prscntneyte _ 10 [2 cos(nx) /* Lf - =3 ee [+e [, costn)dr| + Wo scone) /” 10 10 = S(-1)" + 0+ S(-D" - 1) SHI" + 0+ S(-1" = 1) = Dpto 0 m = Woy) = Secy-1) Luego nore Be ger ® 7 7 5 @ = 2 7 a-1 = 2 = ayai- 1, x ay = —(2(-1)"-1) para n>4 De esta forma a 5 1 w(e,t) = (2— Qe-*een(x) + 2eMsen(20) + (1 — *Desen(32) + 7 z 7 = 10g —2n, y —(2(-1)" — De ‘sen(nz) n=4 ml y la solucién buscada es u(a.t) = Set5t (2— Byer%son(x) + 7 “sen(20) + (1 = Byerten(32} + ” — 1)e?""*een(ner) . = Ejercicio 11.2.2 Determinar la temperatura u(x, ¢) de un conductor con cons tante k=1 que verifica u(r.0) = $2? + 1, u,(0,0) = 0, m(1.Q =a. Solucién. — La ecuacidn diferencial es uy = Ure Consideremos un cambio de funcién de Ja forma u(a,t) = Ax? + Be + o(x.1). Entonces wy (xt) = B+ u(x,t). u,(w.t) = 2Ax + v,(z.t) Uge(a.t) = 2A + dpp(x,t). Reemplazando en la ecuacidn diferencial obtenemos B+ wat) = 2A + wp(2.t) => B= 2A. Por otra parte u,(0.t) =0 = 2,(0.t) = 0. y como. a = u,(1.t) = 2A + u, (1.0) nis Ademas v(a,0) = u(x,0) - $e = $e +1- $e =1. Por Jo tanto & R I se + at + (2,0). con (x,t) que verifica Ue = Ure u(a,0) = v,(0,4) = ve(14) = 0. Por separacién de variables, tomamos 0(2.f) = GW _ Fa) _ QO Fe F(x) G(t) y se obtiene Luego debemos resolver F"(v) + F(a) = (0) = FQ) =0 que tiene autovalores ,, = 1? x? y autofunciones F(z) = cos(nm), para n=l La otra ecuacidn es Gt) + Gt) =0 SS Gilt) = Ane? ™ Sea v(a,t) = > Ane” © cos(nm) = La condicién 1 = o(z.0) = > An cos(nm) n=0 implica 1 2 si n=0 =) 7 = A An 2[ cos(nas) ds 2 n(nns)/! ai nO _ f2 a n=0 =~ 10 si n¥0 Por lo tanto o(x.t) = 2, y luego a u(a,t) = 2+ 52° + at. Ejercicio 11.2.3 Resolver Up — Ur = a2, OXasm, t20 uz(0,t) = ur.) = 0. &>0 u(z,0) = 2 + O0 a0) =a, O 0. Luego la otra ecuacidn es y la autofunciones son M,(x) = Nit) + (BE) me =0 = NW = ance) Consideremos entonces la solucién formal v(t) = Ye owe yr cox (Hr), Como para 0 <2 <7 debemos tener x = 0(a0) = Sa cos (#4) . a = H she ~S a g — S g wit 8 ae = = Beg an (ME) (Pe fn (BED) a OF ne OS i “ma " 27.2 n+) a - 2 lata 2 x hh + 7 0s (2 2 OF 27 2 ; 4 ~ 2 lat omy Luego 2) pamgny?y (n+ ya ' @)= 200 [an (“1 Qn+2, © cos( 2 ) Ejercicio 11.2.4 Una barra homogénea de 3 metros de longitud y difusibilidad 0.3, se saca de un horno de mado que la distribucién de temperatura de la barra es 4x +5. Rapidamente se aisla su manto y sus extremos se mantienen a 5°C, Hallar la distribucién de temperatura u(x. t) de la barra para cualquier instante t > 0 de cualquier punto x de ella. Solucién. Nuestra ecuacidn es Up = O<2r<3. t>0 u(0, t) =5, t>0 u(z.0) = 4r+5, 0<2<3 Haciendo el cambio u(x,t) = u(x,t) + A, obtenemos UW = Up, Uae = 5 = u(0.) = (0,0) +45 = uB.) = vB.)+A, y 4e+5 = u(a.0) = u(z,0)+A Luego haciendo A = 5 nos queda el problema uy = 03u2. O<2<3, t>0 u(0.t) v(3,t) = 0, t>0 u(#.0) = 4a, O<9 M; es decir, si el valor maximo de u en A no se alcanza en la frontera OA, sea (2. Yo) € A un punto interior de A tal que max_u(:t,y) u(t, Yo) Sea R > 0 tal que X esté contenida en la bola de centro (ro. ye) y radio Ry consideremos la funcién [(e = 29)? + (y=). Por Io tanto, v(29, Yo) = w(e- yo) =Mo. y si (x.y) € DA, entonces M. v(0,y) < M+ oo Fah +M) 0 => Uy <0 => Vee + Uy <0, Jo cual es una contradiccién. Luego el valor maximo de wu tiene que alcanzarse en OA. Corolario 12.1.3 ( Principio del Minimo) Sea u una funcién arménica en un dominio bidimensional acotado \ que €s continua en KR. Entonces wu alcanza su valor ménimo en OA. Demostracién. Aplicar la proposicién anterior a —u. Observacién 12.1.4 Estos resultados son wélidos para dimensiones mayores. Consideromas el problema de Dirichlet Au = u/s, = f- donde A < R? es un dominio acotado. ° (12.1) Proposicién 12.1.5 La solucidn de (12.1). si existe. es tinica. Demostracién. Sean uy, 12 soluciones de (12.1). Por Io tanto, Au = Aw = 0 U/oy St2/oy = Fe Ahora como, tw y ug son arménicas en A, también w = uw, — uz es armdnica en Ay w/,, = 0. Por lo tanto w =0 en A. ya que w alcanza su maximo y su mfnimo en dA, Esto implica que w= uw. Proposicién 12.1.6 La solucidn de (12.1), si existe, es estable. Demostracién. Sean v; y v2 armdnicas en A tales que Uo =f YY 2/oy = fee Entonces w =v — v2 es arménica y w/5y = fi— fo» Si consideramos la norma del supremo, I fam fa I= ana | flea) = fale) | tenemos max. w(t-y) < \fr= fa) (Principio del Maximo) Gwe iain, w(0,y) > =| fi fal (Principio del Minimo) Por lo tanto Jw |< |la-f| Vay) er, lo que implica [eilry) = v2(e.0) |< If hl Woy) eA. Corolario 12.1.7 Sea (un)neti una sucesin de funciones arménicas en un do- minio acotado Ade R2, que son continuas en K, y sea (fa)neni sucesion de fun- ciones tales que: Un/ 5, = Ins Entonces, si fn —> f uniformemente en OA, tenemos que Un, —> u uniforme- mente en K, Demostracién. Como f, —+ f uniformemente en OA, entonces dado ¢ > 0.3N EN tal que si n,m > N, se tiene ||fn— fll <€- Por lo tanto, Yn.m > N tenemos que | tn — Um|| < €, lo cual implica que (un)n es de Cauchy y, por lo tanto, convergente, 12.2 Ecuacién de Laplace en el disco Consideremos el problema Au = 0, (a4) = f0); lim,s¢e u(7.9) << cc. O Dy= 0, néN. Fy= 0, néN. x c F(9) = u(a,6) = a + 2 Cha cos(n6) + > Ena"sen(nd) n=l nat Luego, Co = 1 S5" f(s)ds Cy = % IR" f(s)cos(nsjds = a-"en BE, = = f2" f(sjsen(ns}ds = a-"d, Se obtiene finalmente: 1.0) = 4G + ve yt én cos(n9) + dysen(nd)] (12.4) 12.3. Convergencia de la Serie de Fourier: Nticleo de Poisson Supongamos f(9) continua en (0.27). Si | f(9) |< £, entonces [Go| = [2 fls)ds|< ky y lea| = | LC" f(s)cos(ns)ds |< k. vn eN |dn| = |G" f(s)sen(ns)ds |< k, vn EN Consideremos entomeces la sucesin de funciones (tin(7,9))nene definida por: Co o Entonces para todo n € N* se tiene uo = un(r.0) = (5)" (cn c0s(aid) + dysen(nd)) . | un(06) |< 2k ea” para 0ST

0, existe 5 =0d(e) >0 tal que 9s <3 = |f@-f()Il 5. se tiene hn (arco sya) Por lo tanto. existe rp tal que, si 0 < ro , entonces (a =7°) wooo SE (@ = Dar cosO= 5) Fr) ~ 6 Luego para 0 < rp 1. Ademés cos(38) + sen(50) = u(3,6) = Dpln(3) + Ye — 3™)C, cos(nf) Ch = 0. vn #3, E, = 0, vn#5 Por lo tanto la solucidn es ur,0) = x = 17) cos(36) + 3-35 12.4 Ecuacién de Laplace en un Rectangulo Au = 0:;0 (Crcosh( y) + Dasenh( y)) sen( x), nt n= Para determinar C;yDn .conn € N, imponemos las condiciones de contorno en y. u!(2.0) = fala) = Y cine) 2° nas =C,=— Jsen( =? )ds = Cn al fa(s)sen( a )ds Luego, Cn cosh De donde, Asi tenemos que. L n= ul (ay) = donde »=L(6 nat by D,senn( ny E P cosh(4) + a ay nt 1) a )+ ) + Dysent 2) ) sent (a an cosh 2 nob y= can (ea Zh 2 — aft > cosh( a = z[ fals}sen(“=)ads 2 ms = cf fsysen ds, Observacién 12.4.1 El problema 2 se resuelve de manera similar. 12.5 Ejercicios resueltos Ejercicio 12.5.1 Resuelva la ecuacidn de Laplace Au u(1,9) Solucién. O 1, respectivamente Ny(0) = sen(2n9), An = An? Reemplazando el valor de A= ,, en la ecuacién en r obtenemos PMNr) + rMi(r) = 4n?Myl(r) = 0 cuya solucién general es My(r) = aur?” + Bar”. Nuestra solucién formal es x u(r, 6) = SP (ay 1” + bar") sen(2nd) « n=l Para que la solucién sea acotada cuando r > 0 debemos tener by = 0 Yn>1. Luego x u(r,6) = $7 ayr"sen(2n6)]. nat La condicién x £(9) = w(a.0) = $7 ay, a" sen(2n8) , = implica 4 fi on ana = 4 [ (=F) sen(36) sent2nd). Como : sen(30)sen(2nf) = 5 [cos((2n =3)8) ~ cos((2n +3)6). tenemos y 1 3 7 ana” = 21 (6 — ©) cos((2n — 336) d9 -[ (0 = 4) cos((2n +3)0) do z J; 3 A 3 8(-1)" on nm 4n2-9" Por lo tanto nuestra solucién es 8 Kner pry2_ -= X wa (2)" sent2n8). u(r,6) Ejercicio 12.5.3 Resolver el problema Pu Pu _ pet apn 8 O 1. De esta forma la ecuacién en y queda de la forma (y) - AN) = Ni) —r?Na¥y) =0, n>1 cuya solucién general es Nq() = dn cosh(ny) + basen (ny). Luego nuestra solucién formal es u(x, 4) = Slay cosh(ny) + bnsenh(ny), cos(nx) . t= Imponiendo las condiciones iniciales u(z.0) =x y u(x, 7) = 0, tenemos x Sa, cos(na) ta y x 0 = SC lan cosh(nm) + basenh(nz)) cos(nz) nat Por lo tanto 21 tm = de = = -((-1)"~1) y by = a, -coshlnm) on = ~iin* Sah(nim) * De esta forma 2 cosh(nm) y= 27 yy 1 ) y u(x.y) ay wal! 1" = Dleosh (my) — Gg) Senne) costn) o bien 281 senh(n(t = y)) == S05 ((-1)"- D) cos(nr) u(x, y) z > ((-1)"-1) cenh (nm) cos(nz) . Ejercicio 12.5.4 — Encuentre la solucién de la ecuacién de Laplace Au = Ure + ty =O O1, 0<9<2n u(r.2m). ug(r.0) = ug(r.2n), r>1 cos'(9) — $2. 0<0<2n permanece acotada cuando r > 9 Solucién. Poniendo u(r,6) = M(r)+N(9) y separando variables obtenemos las ecuaciones N"(0) -AN(9) = 0 2M"(r) W'(n) Mn) = rM"(r) + rM'(r) + AM(r) = 0 { N@) = NOn). NC) Nn) Por lo tanto las autofunciones y autovalores son. respectivamente 1 cos(n6) \ Mn = m,n SOA Qe sen(nf) Reemplazando el valor de A = ,, en Ja ecuacién en r obtenemos Mir) + rMi(r) — 2M, (r) = 0 cuya solucién general es Mor) = Cy + Doln(r), para n=0. y M,(r) = Gyr" + Dy". para n>1. De esta forma tenemos la solucidn formal u(r.9) = HC + Deln(r)) + S(Cor" + Dyr™) cos(nd) + . mA SE GEnr" + Far-sen(n8) « ra Pero Do lim u(r,8) <0o => 4 Cy n>1, ros En n>1 Luego = Fp -mepeing) 4 SOR ne u(r.) = > + dpe cos(n) + her sen(nf) up(r,9) = = S2nDpr=""" cos(n6) — S7nFyr~"tsen(nd). nt rt Luego debemos tener x 3 x cos!(9) — z= ~ De rPncos(rA) — YEnF,sen(nd). n=l Pero comt(@) = 2 = Fa besa? 2 = FU + 2cos(26) + c03?2H)) = 2 = t + Fcos(2s) + a + cos(49)) — 2 = Feos(26) + boo s(49) . Luego D, =0, -2D) = i, D3 = 0. -4D, = i. Dy, = 0, n>5, y Entonces nuestra solucién es 1, 1 > 14 =lo-t y- 40) W(7.9) = 5Co— GI eos(28) ~ 357 cos( 40) Ejercicio 12.5.6 Sea A una constante, Resolver la ecuacién de Laplace Urr tly =0 O 1. De esta forma la ecuacién en y queda de la forma Nay) — Nay) =0. n>1 cuya solucién general es N,() = an cosh(ny) + bnsenh(ny) . Luego nuestra solucién formal es o(x,y) => jay cosh(ny) + bysenh(ny)|sen (ne) « mm La condicién x Ax = v(2,0) = So ansen(ne), implica oo = Af swat (nz) 77 7 = 4 [-» cost) /. + iff eosin | = -4 (21 La otra condicién 0 = v(x,1) = Sion cosh(n) + bysenh(n)]sen(na2) , nt implica De esta forma x (-1)" cosh(n) Ly) = -A sh(ny) — senh(ny)|s : u(x.y) x = Leosh(ny) Sun(n) enh(ny)|sen(nx) o bien , _ “\ (=1)" senh(n(1 — y)) u(a.y) = -ayo! a senhtay S02) n=1

You might also like